You are on page 1of 473

igh-Yield study

guide that correlates


with the Doctors In
Training Step 1 Review
Course video series

Featuring:
Brian Jenkins, MD
Mike Mclnnis, MD
Chris Lewis, MD
Jennifer Shuford, MD
Hampton Richards, MD

Daily quizzes,
mnemonics, notes

i and diagrams to
promote active
learning

Bridges the gap


between basic and
clinical sciences

r
r
r DOCTORS IN TRAINING
r BETTER DOCTORS. BETTER WORLD.
r
AUSE?
UDED:
ND 7 additional days of online
video lecture access
The ability to pause your
online video lectures access
once for any length of time
within your designated
)urse year.
ansider watching all of the
deo lectures, pausing the
7 ADDITIONAL DAYS >urse, and reviewing them
II again.

1-WEEK EXTENSION
withcodeSPAC15
■ AMty to purchase a I -week cone mansion vrll expire 3 months after your aart date ol Part 2 video lectufe access.

NEEDANE
N HELP.

I
PHARMACOLOGY
Comprehensive pharmacology review Our Solid Pharmacology series features
with an emphasis on high-yield information lectures from our experienced physicians,
for USMLE and COMLEX exams entertaining learning components, mnemonic
and animations to encourage active learning.
Clinical applications throughout This lecture series includes valuable
information that will help you in classes,
Prepare early for third year rotations exams and your Step 1/Level 1 studies.

ALSO, CHECK OUT OUR OTHER SOLID SERIES COURSES:

©urn - «3«-*ii' ! /

LECTURES CAN BE PURCHASED INDIVIDUALLY OR AS AN ENTIRE COURSE WITH DIT'S


FLEXIBLE LEARNING CREDIT SYSTEM AND CAN BE WATCHED ON YOUR SCHEDULE.
USMLE Step I Review
Study Guide
2012 edition

DOCTORS IN TRAINING
BETTER DOCTORS. BETTER WORLD.

Brian Jenkins, MD
Doctors In Training.com: USMLE Step I Review, 2012 edition

Author: Brian Jenkins. MD

Copyright © 2012 Doctors In Training.com, LLC


www.doctorsintraining.com

Doctors In Training.com
1701 River Run, Suite #703
Fort Worth, TX 76107
All Rights Reserved. This text is protected by copyright. No part of this book may be reproduced in any form or by any means, including photocopying, or
utilized by any storage and retrieval system, without written permission from the copyright owner.

Selected images from Lippincott Williams & Wilkins Atlas of Anatomy "*^~ rr
©2008 Wolters Kluwer Health, Inc. All rights reserved. WHlicHTIS & Wl HCIPIS

a Wolters Kluwer business


DISCLAIMER: THE AUTHOR DISCLAIMS ANY UABILITY. LOSS. INJURY. OR DAMAGE INCURRED AS A CONSEQUENCE. DIRECTLY OR INDI
RECTLY. OF THE USE AND APPLICATION OF ANY OF THE CONTENT AND MATERIAL CONTAINED IN THIS TEXT. ALTHOUGH THE INFORMA
TION IN THIS TEXT HAS BEEN CAREFULLY REVIEWED FOR CORRECTNESS. THE AUTHOR CANNOT ACCEPT ANY RESPONSIBILITY FOR ANY
ERRORS OR OMISSIONS THAT MAY BE MADE. THE AUTHOR MAKES NO WARRANTY. EXPRESS OR IMPLIED. AS TO THE COMPLETENESS. CUR
RENCY OR ACCURACY OF THE CONTENTS OF THIS TEXT. THE INFORMATION CONTAINED IN THIS TEXT SHOULD NOT BE CONSTRUED
AS SPECIFIC INSTRUCTIONS FOR INDIVIDUAL PATIENTS. MANUFACTURER'S PRODUCT INFORMATION AND PACKAGE INSERTS SHOULD BE
REVIEWED FOR CURRENT INFORMATION. INCLUDING CONTRAINDICATIONS. DOSAGES. AND PRECAUTIONS.

For problems, questions, or concerns, you may contact the author at support@doctorsintraining.com.

Copyright © 2012, Doctors In Trainmg.com. LLC All Rights Reserved.


Key to Abbreviations
This Study Guide contains page references to several medical educational resources. Below is a key
to the abbreviations that accompany the various page number references appearing in this text.

FAII Le, T, Bhushan, V, et al. First Aid for the USMLE Step 1 2011. New York, NY:
McGraw-Hill; 2011.

FAQ Le, T, Bhushan, V, et al. First Aid for the USMLE Step 1 2011. New York, NY:
McGraw-Hill; 2012.

Phys Hall, JE. Guyton and Hall Textbook of Medical Physiology. 12th ed. Philadelphia, PA:
Saunders Elsevier; 2011.

R Kumar, V, Abbas, AK, et al. Robbins and Cotran Pathologic Basis of Disease. 8th ed.
Philadelphia, PA: Saunders Elsevier; 2010.

COA Moore, KL, Dalley, AF, & Agur, AMR. Clinically Oriented Anatomy. 6th ed.
Philadelphia, PA: Lippincott Williams & Wilkins; 2010.

H Longo, DL, Fauci, AS, Kasper, DL, Hauser, SL, Jameson, JL, Loscalzo, J, eds.
Harrison's Principles of Internal Medicine. Vol. 2. 18th ed. New York, NY:
McGraw-Hill; 2012.

GG Brunton, LL, Chabner, BA, & Knollman, BC, eds. Goodman & Gilman's The
Pharmacological Basis ofThempeutics. 12th ed. New York, NY: McGraw-Hill; 2011.

Neither the Doctors in Training USMLE Step I Review course, nor this Study Guide, is endorsed
by or affiliated with any of the medical education resources referenced in this text.

Course Viewing Options


Historically, the Doctors In Training live Step I review course was structured as 5 hours of lecture
per day (Monday through Friday) for 3 weeks. We recognize, however, that this schedule may not
meet the individual needs of every student. To make the course as flexible and helpful as possible,
the online course is broken into 148 lectures, which can be viewed in more or fewer than 15 days,
as desired.

30-day course 5 lectures/day

21-day course 7 lectures/day

18-day course 8 lectures/day

15-day course 10 lectures/day

12-day course 12 lectures/day

10-day course 15 lectures/day

Although you have the flexibility to view the lectures in any order, we strongly recommend that
you watch the videos in the order in which they appear in the Table of Contents, completing each
row before proceeding to the next one, regardless of how many lectures you view in a day.

Copyright © 2012. Doctors In Training.com. LLC. All Rights Reserved.


2012 Doctors In Training Step 1 Review Course Table of Contents

Lecture 1 Lecture 2 Lecture 3 Lecture 4


Foundations 1 - Principles Foundations 2 - Cellular Foundations 3 - Cellular Foundations 4-Cel
and Embrvolo Order Structure and Function Suffering and Death
The 21 Fundamental Cell Cycle and Protein Cytoskeleton and Other Apoptosis and Necrosis
Principles of Human Organelles Cellular Components Cellular Injury
Life Protein Processing Plasma Membrane
Embryology Part 1
Embryology Part 2
Foundations 5 - Cellular Foundations 6 - Cellular Neuro 1-Basics Neuro 2 - Hypothalamus
Adaptations Organization of the Brain and Slee.
Cellular Response 1- Cellular Response 3- Neurotransmitters Hypothalamus
Acute and Chronic Internal Adaptations Sleep
Inflammation Cellular Response 4 -
Cellular Response 2 - Atrophy and Aging
Control of the
Extracellular
Environment
Neuro 3 - Brainstem and Pharm Basics 1- Pharm Basics 2 - Cellular Endocrine 1 - Pituita
Cranial Nerves ParasvmDathetics Communication Basic Overview of
lefts, Arches, Pouches Parasympathetic Autonomic Nerve Endocrine
ranial Nerves Activation Terminals Pituitary and Prolactin
pecialty Centers in the Parasympathetic Inhibition G-protein-Linked 2 Other Pituitary Hormon
Brainstem Messengers
Endocrine 2 - Adrenals Pharm Basics 3- Gl 1-Oropharynx Endocrine 3-Thyroid
Adrenal Steroids Sympathetics Mouth, Tongue, Salivary Thyroid Basics and
D Cushing Syndrome Sympathetic Agents Glands Hyperthyroidism
Other Adrenal Pathology Sympathetic Blockers Nose and Sinuses Hypothyroidism and
Thyroid Cancer
Immunology 1-Basics and Immunology 2 -T Cells Biochem 2-RNA Biochem 3 - Protein
Antieen Presentation and Monocytes RNA Basics tRNA and Protein
Immunology Basics T Cells Transcription Synthesis
Antigen Presentation Monocytes and Protein Modification
Macrophag
Immunology 3 - B cells. Immunology 4- Micro 1 - HIV Gl 2-Esophagus
Antibodies, and Cytokines Complement, HIV Basics Basics of the Esophagus
B Cells and Antibodies Highly Active Antiretroviral Esophageal Pathology
Cytokines and Immunodeficiency Therapy (HAART)
Immunosuppressants Complement,
Hypersensitivity, and
Granulocytes
Immunodeficiencies
Pharm Basics 4- ir@M!"-=iiJfita:'i'i3iiMi
of the Stomach Pharmacokinetics Gastritis, PUD, and Antaci
Gastric Physiology Enzyme Kinetics and Other Gl Pathology Duodenum
Pharmacokinetics Pancreas
Pharmacodynamics
Endocrine 4- Pancreas Gl 6-Small Intestine Biochem 4-Glucose Biochem 5- Ener
Endocrine Pancreas and Enterocytes and Glycolysis and Potential Fates of Pyruvate
DM Absorption Gluconeogenesis Pyruvate Dehydrogenase,
DKA and DM Medications Diseases of the Small Glycogen TCA and ETC
Intestine
Biochem 6-HMPShunt Biochem 7-Fuel Use Endocrine 5 - Obesif Gl 7 - Liver and Cirrhosis
nd Other Sugars Ketones Physiology of Obesity Embryology of the Liver
MP Shunt Feeding, Fasting, and Metabolic Syndrome and Hepatocytes
uctose, Galactose, and Exercise Obesity Cirrhosis and Portal
Lactose Ethanol and Caloric Hypertension
eficiencies Liver Diseases
2012 Doctors In Training Step 1 Review Course Table of Contents
Lecture 1 Lecture 2 Lecture 3 Lecture 4
Gl 8 - Liver Pathology Gl 9-Gall Bladder Biochem 8 - Lipids Biochem 9 - Amino Acids
Hepatitis Biliary Anatomy, Lipid Transport and Nitrogen
J Additional Liver Diseases Physiology, and Fatty Acids and Cholesterol Amino Acid Basics
Jaundice Nitrogen Metabolism
Biliary Tract Disease
Biochem 10 - Amino Acid Pharm Basics 5-Dm Heme 1 - Clotting Factors
Disorders Metabolism Coagulation Basics Erythrocyte Basics
K Amino Acid Disorders Drug Metabolism Agents Affecting Clotting RBC Pathology
Alcohol Metabolism and Factors
Drug Suffixes
Biochem 1 - DNA Biochem 11-Minerals Heme 3 - Nonhemolytic Heme 4-Hemolytic
DNA Basics Calcium, Iron, and Zinc Anemias Anemias
DNA Interactions Trace Minerals and Toxic Microcytic Anemia Intrinsic Hemolytic
Metals Macrocytic and Anemias
Nonhemolytic, Extrinsic Hemolytic
Normocytic Anemias Anemias
Heme 5-Platelets IIIMiilIi of Blood Biochem 12 - Fat-Soluble Biochem 13-Water
Platelet Basics Lymph Vitamins and Antioxidan uble Vitamins
Platelet Disorders Leukemias Vitamins D, K, and A Folic Acid, Vitamin B12,
M Other Bleeding Disorders Chronic Myeloproliferative Vitamins C, E, and Other and Vitamin B6
Antiplatelet Medications Disorders Antioxidants Vitamins B5, Niacin,
Multiple Myeloma Riboflavin, Thiamin!
and Biotin
Gl 10 - Large Intestine 1 Gl 11 - Large Intestine 2 Micro 2- Micro 3 - Protozoa &
Large Intestine Basics, Distal Large Intestine Enterobacteriaceae & Helminths
Cecum, and Appendix Anus Diarrhea Protozoa
N Ascending, Transverse, and Enterobacteriaceae, UTI, Helminths
Descending Colon and Antibiotics
Diarrhea and Food
Poisoning
Oncology 1-Basics Oncology 2 - Neoplastic Oncology 3 - Cancer Oncology 4 - Cancer Drugs
Genetics of Cancer Progression Prevention and Host Cancer drugs
Cancer Risk Factors Neoplastic Progression Defense
Downstream Effects of Cancer Prevention
Neoplasia Host Defense Against
Cancer
Medicine in Society 1- Medicine in Society 2- Medicine in Society 3 - Medicine in Society 4-
Evidence Based Medicine Bias & Statistics Public Health Geriatrics
Studies and Diagnostic Bias and Error Disease Prevention: Physiological and
Tests Confidence Interval Pediatrics and Psychological Changes
Application of Test Data Obstetrics Diseases and Resources for
Prevention of Adult the Elderly
Diseases
IIMflirafl.-fflSyi.-g IIBHilflfifHTff llTPTflTFPJfli'rTTffi
Healthcare System Ethics and Phvsiolo
Healthcare System Part 1 Ethical Principles Diaphragm and Circulation
Healthcare System Part 2 Ethical Scenarios Respiration Lung Volumes and
Respiratory Tree and Hemoglobin
Alveoli Pulmonary Circulation and
Pulmonary
Hypertension
2012 Doctors In Training Step 1 Review Course Table of Contents
Lecture 1 Lecture 2 Lecture 3 Lecture 4
Pulmonary 4 - Extreme Pulmonary 5 - PE, COPD, Pulmonary 6 - Restrictive
Oxygenation and Environments and Asthma Lung Disease
Ventilation Altitude, Aviation, and PE, DVT, and COPD Restrictive Lung Diseases
Oxygenation Space Asthma and ARDS
Ventilation Deep Sea and Hyperbarics Pneumoconiosis and
Sarcoidosis
PulmonmfflMzmfi Cancer Micro4-Bas icro 6 - Gram Positives
and Infections Bacterial Structures Gram Positives and Staph. and Penicillins
Lung Cancer Exotoxins Streptococcal Species Other Gram Positives
Lung Infections Penicillins
Micro 7-Cell Wall Micro 8 - TB and TB Drugs Micro 9-Other Cardiovascular 1-
Inhibitors Mycobacteria and Nonstaining Bacteria
Cephalosporins Tuberculosis Spirochetes and Zoonotics Overview and Ventricle
Other Cell Wall Inhibitors Anti-Tuberculosis Agents Other Nonstaining Bacteria Development
Atrial Development and
Fetal Circulation
Cardiovascular 2- Cardiovascular 3 - Cardiac Cardiovascular 4 - Heart Cardiovascular 5 - Edema
DevelODmental Pathola Output Failure and Shock
U Right-to-Left Shunts Cardiac Output Variables Heart Failure Capillaries and Edema
Left-to-Right Shunts a ' Charting Cardiac Output Pathophysiology Capillaries and Shock
[■^■H f J i M i i H I f a l Heart Failure Medicatio
Cardiovascular 6 - Cardiac Cardiovascular 7 - Heart Cardiovascular 8- Cardiovascular 9 - ECG and
Cycle Murmurs Electrophysiology Arrhythmias
V Atria and Ventricles Systolic Murmurs Action Potentials Atria to AV Conduction
Heart Sounds and Jugular Diastolic Murmurs Antiarrhythmics Ventricular Arrhythmias
Waves
Cardiovascular 10 - BP and Cardiovascular 11- Cardiovascular 12 Cardiovascular 13 - M
HTN Antihvoertensives Atherosclerosis Ml Pathophysiology
W Physiology of BP Antihypertei therosclerosis an_ Diagnosis and Treatm
Regulation Choice of i H l i l ' J ' M l H l ff p H anginal Therapy and of Ml
Hypertensio ipid-Lowering Agents
Cardiovascular 14 - Other Cardiovascular 15- Neuro 4 - Regions of the Neuro 5-Brainstem in
Cardiac Pathology Vascular Disorders Brain Cross Section
X Cardiomyopathies and Vascular Disorders 1 Cerebral Cortex Anatomy of the Brainstem
Endocarditis Vascular Disorders 2 Cerebral Circulation Cranial Nerve Lesions
Other Cardiac Pathology
I Neuro 6 - Occlusion Neuro 7 - Vascular Events Neuro 8 - Movement Neuro 9-Basal Gan
Syndromes Arterial Events Movement Execution Basal Ganglia and
Brainstem Occlusion Venous and Ventricular Modifiers of Movement Hypokinesis
Syndromes 1 Events Hyperkinesis
Brainstem Occlusion
Syndromes 2
Neuro 10-Spinal Cord Neuro 11 - Brachial Plexus Neuro 12 - Lower Rheum and Derm 1- Bone
and Lesions and Upper Extremity Extremity and Skeletal and Bone Disorders
Spinal Cord Basics Nerves Muscle Bone Histology and
! Spinal Cord Lesions Brachial Plexus Anatomy Lower Extremity Pathology
Nerves of the Upper Skeletal Muscle Paget Disease and Bone
Extremity Tumors

Endocrine 6 - Calcium Rheum and Derm 2-Joint Rheum and Derm 3-Joint Rheum and Derm 4
Metabolism Basics Diseases Systemic Disorders
Basics of Calcium B a s i cand
Basics s aLower
nd Lower Osteoarthritis and Systemic Rheumatic
Metabolism E x t r e mJoints
Extremity ity Joints Treatment Diseases
iiiT?jmiiii»]i.-{^-.Mj.-
Hyperparathyroidism and Upper Extremity Joints wi'^BiuffMlSB'
Hypoparathyroidism
2012 Doctors In Training Step 1 Review Course Table of Contents

Lecture 1 Lecture 2 Lecture 3 Lecture 4


Neuro 13-Sensation Rheum and Derm 5 - Micro 10 - Mvcolo Micro 11 - Antifungals and
Skin Sensation Dermatolo Mycology Neuro Review
BB Other Demyelinating Derm Basics and Derm Arthropods Antifungals
Diseases Path Part 1 Neuro Review
Otic Sensation Derm Path Part 2
Neuro 14 - Eve Psvch2-ChildPsvch Ufffliyn'niril'liiUM
Eye Structures Psychology Basics Infants and Early
C Neural Aspects of the Eye Learning and Therapy Childhood Delirium, Dementia,
Adolescence Alcohol
Substance Abuse
Neuro 15-Dementia and Neuro 16-Tumorsand Neuro 17 - Seizures Psvch 4 - Psvchosis and
Headache Anesthetics Seizures Mania
Dementia Brain Tumors Anti-Seizure Medications Psychosis and
Headache Anesthetics Schizophrenia
Mania and Bipolar
Disorder
Psvch 5 - Depression Psvch 6 Renal 1-Basics Nephron and
Major Depressive Anxiety Anatomy and Embryology Diuretics
Disorder (MDD) Ego Defe.. Glomerulus ^^yni^yj ~d 11; -i W r>T:
Antidepressant and Personali :|p1--iii-Hi
Medication ■■ E K S a
Renal 3-Metabolic Renal 4-Glomerular Renal 5-Other Renal Micro 12 - Other Gram
Disorders Pathology Patholo Negative Bacteria
Renal Metabolic Basics Nephritic Syndrome Other Renal Pathology 1 Gram Negative Basics
pHandADH Nephrotic Syndrome Other Renal Pathology 2 Specific Gram Negative
Organisms
Micro 13-Protein Micro 14 - DNA Viruses Micro 15 - RNA Viruses Micro 16-Viral Bas
Synthesis Inhibitors Herpes Viruses RNA Viruses Part 1 Viral Basics Part 1
Protein Synthesis Other DNA Viruses RNA Viruses Part 2 Viral Basics Part 2
Inhibitors
Other Antibiotics
Review 1-Micro by Review 2 - Side Effects and Reproduction l-Anatom\ Reproduction 2 -
Systems Antidotes Androgens and Testes
Micro by Systems 1 Drug Side Effects Reproductive Anatomy Androgens
Micro by Systems 2 Antidotes Reproductive Embryology Testes Pathology
Reproduction 3 - Male 4-Estrogen Reproduction 5-Vagina, Reproduction 6 - Ovaries
and Progesterone I Cervix, and Uterus Ovarian Pathology
Female Reproductive Cycle Vaginal and Cervical Ovarian Tumors
Hormonal Birth Control Pathology
and Menopause Uterine Pathology
Reproduction 7- Reproduction 8- Review 3 - Genetic Review 4-Genetic
Pregnancy Chromosomal Disorders Disorders: AD and Diseases: AR and X-linked
Pregnancy Complications Trisomies and Mental Trinucleotide Repeats Autosomal Recessive
Labor and Delivery Retardation Autosomal Dominant Diseases and Lysosomal
Other Chromosomal Diseases Part 1 Storage Diseases
Disorders Autosomal Dominant X-Linked Recessive
Diseases Part 2 and Disorders
Trinucleotide Repeats
Biochem 14-Basic Reproduction 9 - Breast Review 5 - Pediatric Review 6 - Final Review
Genetics Benign Breast Conditions Review The Wisdom and.
Genetic Laboratory Breast Tumors and Cancer Pediatric Review Part 1 Medicine
Techniques Pediatric Rev
Inheritance
The 21 Fundamental Principles of Human Life
I. What are the 21 fundamental principles of human life?
1) Human life is a system of , Human life requires
sufficient
2) Human life is organized into
3) Every level of human life has
4) As a system of energy and matter exchange, human life.
A. The law of
B. Since establishing order requires energy,.
C. Energy-consuming reactions (AKA ) must be coupled with
energy releasing reactions (AKA ).
5) In order for human life to grow, must take place.
6) Each stakeholder in human life is
7) As specialization increases, the various stakeholders of human life can be categorized as being
more or more relative to other stakeholders.
8) Each stakeholder in human life has

9) Human life is organized into multiple


10) The specialized stakeholders of human life that are designed to establish order.
o
11) Human life is in a constant state of_

p
12) In order to maintain the harmony of stakeholders, ^

13) When a stakeholder of human life goes through a cycle of. o


zCO
14) Harmony among the stakeholders in human life requires a method of allowing.
i

15) The more stakeholders that -rj


25.
16) Every stakeholder of human life has which ultimately were n

17) Each stakeholder in human life makes efforts to $

18)

19) 3
cr
20) The only way to alleviate S2_
— <$
21)

Copyright© 2012. Doctors lnTraining.com. LLC All Rights Reserved. L J


Embryology Part I
Fetal landmarks (FAII pi 18) (FAI2 pi24)

Morula Trophoblast placenta


embryonic membrane
Inner Cell Mass
I Hypoblast
Endoderm
Eplblast
A gastrulation Pircntnymeof;
•liver • respiratory i

•thyroid •tonsils
• thymus
epidermis •pancreas
• bladder, urethra
hair, mill
• tympanic cavity
mmary gland
ant pituitary 'auditory tube
lent tract
Intermediate:
Inner tar nturulation
teeth enamel via notoi"1" 'urogenital
(Including kidney
and gonads) • (covers amnion) *
• lateral and ventral ' "•" of 9"t (with endode
N-ur.,IC..-,t(>lls: uetamai; body wall (with "cardiovascular and lymp
* Schwann cells vertebral bodies, ectoderm)
"-■ —tUs Dermomyotomes
mine lamina, cannulus
• post pituitary fibrosus of Inter-
B«»
* pineal body Myotomes" Eeimtrej
anocytes vertebral discs • intrinsic back muscles
OermoSsmej.;
Ha of adrenal us " pulposls • dermis
from the (erector spinae)
I septum
■I root ganglia Hyporrnrci
-facial structures • intercostal!
• obliques • dorsal appendages - extensor
.js
• tansverse ah intral appendages - flexor m
In A Inhibits ml
• rectus abdo

'.. What is the formula for Gibbs free energy?

What developmental structure matches the following description?


• Fetal placental structure that secretes hCG
• Maternal component of the placenta

Order the following molecules by how much energy they contain that can be made
available to fuel endergonic reactions: pyruvate, adenosine monophosphate, glucose,
adenosine, adenosine triphosphate.

lat are the stages of an embryo between conception and an inner cell mass?

Copyright © 2012. Doctors In Training.com, LLC. All Rights Reserved.


Embryology part 2
GG: Chapter 23, 66
Fetal landmarks (FAII pi 18) (FAI2 p!24)
Embryologic derivatives (FAII pi 19) (FAI2 pl26)
Neural development (FAII pi 18) (FAI2 pl2S)
Teratogens (FAII pl20) (FAI2 pl27) (GGpl84S)
Important genes ofembryogenesis (FAII pll8) (FAI2 pl24)
Fetal alcohol syndrome (FAII p!20) (FAI2 pl28) (GG p64l)

6. What are the neural crest derivatives of the following adult structures?

Peripheral Nervous System

Ear

Eye

Adrenal Gland

Mouth

Heart

Digestive System

Thyroid
o
c
Skin z
%
Homeobox (HOX) Genes o
• Blueprint for skeletal morphology z
• Code for transcription regulators
• Mutation in Homeobox HOXD-13 ■▶ synpolydactyiy (extra fused digit between 3rd and 4th fingers)
I
• Retinoic acid alters HOX gene expression
-rj
D
n
- &
fl>
to
CD
D
Q_
m
3
cr
-.

1o
95

Copyright © 2012. Doctors In Training.com. LLC. All Rights Reserved. [3]


other findings would you expect to see?

9. List as many drugs as you can that are contraindicated in pregnancy.

During what weeks of fetal development does organogenesis take place?


(FAI2pl24)

What is the relationship between the notochord, the neural plate, the neural tube, and
the neural crest cells? (FA 12 p 125)

What is the embryologic origin of the following adult structures? (FAI2 pl26)
• Anterior pituitary
• Cornea
• Lens
• Retina
• Olfactory epithelium
• Mammary glands
• Salivary glands
• Sweat glands

Copyright © 2012. Doctors In Training.com. LLC. All Rights Reserved.


uestion Warm-Up

origin of the tissue just below the anal canal? (FAI2 pl26)

What effect might the following teratogens have on a developing fetus? (FAI2 pl27)
• ACE inhibitors
• Aminoglycosides
• Diethylstilbestrol
• Tetracyclines
• Valproic acid

Which vitamin should not be supplemented in large amounts during pregnancy? (FA I
PI27)

Cell Cycle and Protein Organelles

Nuclear Localization Signals


• Amino acids sequences: 4-8 amino acids; rich in lysine, arginine, and protein
• Essential component of proteins bound for or residing in the nucleus (e.g., histones)
• Nuclear pores recognize these signals and transport proteins into the nucleus via ATPase
• A mutation in a single amino acid may prevent nuclear transport

5. Cell Cycle Basics (FA 11 p76) (R p86)


• Cyclins + cyclin-dependent kinases (CDK) phosphorylate target proteins to drive the cell cycle
• All cyclins are degraded by ubiquitin protein ligase when their cell-cycle specific job is complete
• p2l, p27, and p57 bind to and inactivate cyclin-CDK complexes (p53 controls the activation of p2l)
Gl - S
• Cyclin D binds/activates CDK4 ■* phosphorylation of Rb protein -> Rb protein is released from
transcription factor E2F ■* with E2F unbound, the cell is free to transcribe/synthesize components
needed for progression through the S phase (cyclin E, DNA polymerase, thymidine kinase, dihydrofolate
reductase)
• Cyclin E binds/activates CDK2 ■* the cell is allowed to progress into S phase

G2-*M
• Cyclin A - CDK2 complex -» mitotic prophase
• Cyclin B - CDKI complex activated by cdc25 -* breakdown of nuclear envelope (nuclear lamin
breakdown) and initiation of mitosis

Copyright © 2012. Doctors In Training.com. LLC. All Rights Reserved.


What molecules provide the structural framework for DNA and the nuclear envelope?

What must be present on a protein in order for that protein to gain entry into the nucleu

Which types of proteins are responsible for fostering the progression through the cell cycle

Which cydin-CDK complexes assist in the progression from Gl phase to S phase?

Which tumor suppressor proteins prevent the progression of the cell into S phase?

Which cyclin-CDK complexes assist in the progression from G2 phase to M phase?

Protein Processing
Phys: Chapter 2
Endoplasmic Reticulum and Golgi
S- Rough ER (FAII p76)
_£j Smooth ER (FAII p76)
C Golgi apparatus (FAII p77)
Q Cell Trafficking (FA/2 p80)
Enzyme Terminology (FAII p95)
_j Protein Degradation
-= - Proteasome (FAI2 p80)
— - Lysosome
<L>
U 12.1-cell disease (FA 12 p80)
' • Deficiency in mannose phosphorylation
( N • no mannose-6-phosphate to target lysosomal proteins ■* secretion out of cell instead of into lysosomes

—7 • death by age 8
C • (+) corneal clouding, coarse facies, HSM, skeletal abnormalities, restricted joint movement, +/- MR

13. Chaperones and Heat Shock Proteins:


Chaperones assist in the proper folding and transport of polypeptides across the ER, Golgi, and
beyond. Some chaperones are synthesized constantly and are involved in normal intracellular
protein trafficking. Others chaperones are induced by stress such as heat (heat shock proteins
hsp70, and hsp90). These chaperones "rescue" shock-stressed proteins from misfolding. If the
) folding process is not successful, the chaperones facilitate degradation of the damaged protein.
L- This degradative process often involves ubiquitin (also a heat shock protein), which is added to the
abnormal protein and marks it for degradation by the ubiquitin-proteasome complex.

Copyright © 2012. Doctors In Training.com. LLC All Rights Reserved.


End of Session Quiz
14. What molecule targets proteins in the endoplasmic reticulum for lysosomes?

15. What is deficient in l-cell disease? (FAQ p80)

16. What are the different methods that a cell uses to break down proteins (proteolysis)?

17. Which cell types are rich in smooth ER?

o
c
z

o
zi n
to

n
C
D

o
a
(D
.
~5

Copyright © 2012, Doctors In Training.com. LLC. All Rights Reserved. [7]


Cytoskeleton and Other Cellular Components
Phys: Chapter 2
Cytoskeleton
- Cytoskeletal elements (FAII p78)
- Immunohistochemicalstains (FAII p78)
- Microtubule (FAII p78)
- Cilia structure (FAII p78)
- Kartagener syndrome
Free soluble cytoplasmic elements
- Free ribosomes
- Enzymes
- Water, elements (potassium), organic molecules (amino acids, glucose)
- Cytoskeleton
Insoluble cytoplasmic inclusions
- Glycogen
- Lipid droplets
Sarcoplasmic reticulum
Mitochondria (Phys pl6)
Peroxisome

4. Intermediate Filament Structures


i
f t Intermediate Filaments Structural Component of:

Vimentin Connective tissue


• Support cellular membranes (fibroblasts, leukocytes, endothelium)
• Keep certain organelles fixed in
13
U cytoplasm
I Desmin Muscle cells (smooth, skeletal, heart)
m
m Cytokeratin Epithelial cells (keratin in desmosomes and
Z hemidesmosomes)
\
Glial fibrillary acid proteins Astrocytes, Schwann cells, other neuroglia
efe (GFAP)

z
£.
Peripherin Neurons

D Neurofilaments (L, M, H Axons within neurons


0
_i_
(molectular wt)

Nuclear lamins (A, B,C) Nuclear envelope and DNA within

Copyright @ 2012. Doctors In Training.com, LLC All Rights Reserved.


lick Quiz
What drugs act on microtubules WMMKrii

What two fundamental substances are required to make most things work inside the

Which organelle is responsible for the breakdown of very long-chain fatty acids?

What are the intermediate filaments for the following tissue types and cellular structui
• Connective tissue
• Muscle tissue
• Epithelial tissue
• Axons

What are the defects seen in Kartagener syndrome? (FAII p78)

Plasma Membrane
Phys: Chapters 2, 4
R: Chapter 2
Plasma membrane
- Composition (FAII p78)
- Sodium pump (FAII p79)
- Sodium-mediated diffusion (Phys pS4)
Arachidonic acid (FAII p39l) (R p58)

Copyright 3 2012. Doctors In Training.com. LLC. All Rights Reserved.


10. Label the steps outlining the derivatives of arachidonic acid.

cell membrane phospholipids

140

16 18

I
Blocks receptors
to the agents on the left

17

C
o
t5
c 11. Tyrosine kinase receptor
LL_ • Transmembrane receptors that bind an extracellular ligand then intracellularly transfer a phosphate

"O group (phosphorylate) from ATP to selected tyrosine side chains on specific cellular proteins including
C itself (autophosphorylation). The first step in the signaling cascade that is initiated by tyrosine kinase
rd
receptors is autophosphorylation.
• PDGF and other growth factor receptors: single-pass transmembrane protein
D • Insulin and IGF-1 receptors:
tJ - 2a subunits (bound by disulfide bonds) - bind extracellular ligand
3
i_ - 23 subunits-tyrosine kinase activity
lo
i_

U
I
m
CO
Z
o
Iz
D
o
[10] Copyright © 2012. Doctors In Training.com. LLC. All Rights Reserved.
12. Clathrin

BBfiffla

|PElSffl|
rtctPtor clathrin coat

Vesicle formation I

-.: I*.

naked transport vesicle

'cargo molecule dynamln

A) Normal LDL Receptor


LDL binding site
' LDL receptor protein

clathrin
coated pit
binding site
A mutant LDL
B) Mutant LDL Receptor receptor lacks the
coated-pit binding site but
retains a functioning LDL-
binding site. As a result,
cells with mutant receptors
are able to bind LDL
normally but are unable to
LDL receptor protein with ingest it. Individuals with
abnormal coated pit binding site this mutation have a higher
risk of dying prematurely
from a myocardial infarction.

Copyright ©2012. Doctors In Training.com. LLC. All Rights Reserved.


" . Which arachidonic acid product causes the following effect? (FAII p39l)

• Increased bronchial tone


• Decreased bronchial tone
• Increased platelet aggregation
• Decreased platelet aggregation
• Increased uterine tone
• Decreased uterine tone
• Increased vascular tone
• Decreased vascular tone

14. What are the two most abundant substances in plasma membranes? (FAI I p78)

. What drugs act on the arachidonic acid product pathway? What enzymes do they affect!
(FAII p39l)

. What protein is involved in transporting an endocytosed vesicle from the plasma


membrane to the endosome?

Copyright © 2012. Doctors In Training com. LLC. All Rights Reserved.


Question Warm-Up

, /hat drugs interfere with microtubule functioning? (FAI2 p8l)

Which cell types are constantly regenerating themselves due to an absence of the GO
phase and a short Gl phase? (FAI2 p79)

Apoptosis and Necrosis


R: Chapter I
Apoptosis (FAII p220)
Intrinsic apoptosis vs. extrinsic apoptosis (FAII p220) (FAI2 p244)
Necrosis (FAI I p220)

4. Mechanisms of Apoptosis (FAI I p220) (R p27)


• Apoptosis is initiated when: (I) cells are deprived of important cell signals such as growth factors, (2) cell
stress is present, (3) DNA damage is present and the DNA repair process fails •» p53 triggers apoptosis,
(4) cytokines such as TNF trigger apoptosis, or (5) cytotoxic T cells insert granzyme B into cells ■*
activation of caspases
- If p53 is mutated or absent, it cannot induce apoptosis in the presence of severe DNA damage
• Caspase proteases execute apoptosis and have Cysteine protease that cleaves after aspartic acid residues
• Extrinsic Pathway (Death Receptor-Mediated) via activation of cell surface death receptor
- Type I TNF receptor I (TNFRI)
- Fas (CD95): Fas ligand binds Fas ■* grouping of 3 or more Fas molecules to form a binding site for
FADD ■* Fas-associated death domain (FADD) binds inactive caspase-8 (caspase-10 in humans) ■*
cleavage and activation of caspase-8 ■* cleaving and activation of other pro-caspases ■* apoptotic
proteolytic cascade (pathway used in selection of T cells)
- FLIP protein may bind to and inhibit cleavage of pro-caspase-8 thereby inhibiting apoptosis
• Intrinsic Pathway (Mitochondrial)
- Increased mitochondrial permeability ■* release of pro-apoptotic molecules into the cytoplasm
- Bcl-2 proteins regulate apoptosis.
- Bcl-2 and Bcl-x prevent apoptosis (Bcl-2 is homologous to Ced-9)
- Bak Bax, and Bim are pro-apoptotic
- Cells undergo stress or are deprived of important cell signals -* Bcl-2 and Bcl-x are lost from
mitochondrial membranes and are replaced with Bak, Bax, and Bim ■■> mitochondrial membrane
permeability increases ■* caspase activating proteins (i.e., cytochrome c) and AIF (apoptosis inducing
factor) leak out
- Cytochrome c binds cytosolic Apaf-I (apoptosis activating factor-1 which is homologous to Ced-4) ■*
this complex activates caspase-9
- Apaf-1 may be directly inhibited by Bcl-2 and Bcl-x
- AIF (apoptosis inducing factor) binds to and neutralize various inhibitors of apoptosis ■* apoptosis is no
longer inhibited
• During apoptosis, substances that recruit phagocytes are secreted and marker molecules are placed
on the cell surface so that phagocytosis can take place prior to necrosis and inflammation. CD31 is
expressed by healthy cells to prevent phagocytosis.
Copyright @ 2012. Doctors In Training.com, LLC. All Rights Reserved.
Cellular Injury
R: Chapter I
Causes of cell injury (Rpll)
Reversible vs. irreversible injury (FA 12 p245)
Mechanisms of cell injury (R pi 7)
- ATP depletion
- Mitochondrial damage
- Influx of calcium and loss of calcium homeostasis
- Accumulation of oxygen-derived free radicals (oxidative stress)
- Free radical injury (FAII p222)
- Defects in membrane permeability
- Damage to DNA and proteins
Ischemia-hypoxia vs. ischemia-reperfusion
- Infarcts: red vs. pale (and reperfusion) (FAII p22l)

c 9. What cellular byproducts might you detect in the serum when the following cell types
are injured?)
Cardiac myocytes
Skeletal myocytes

Hepatocytes
Salivary gland cells
Pancreatic exocrine cells
RBCs

. Ti fl P L
Which metals are known to facilitate the generation of oxygen free radicals?

What cellular particles are responsible for handling oxygen free radicals? (R p2l)

In order to prevent cellular death from cellular injury, what specific things would you nee
to optimize to prevent the cellular injury in the first place? (R pi 8)

Copyright © 2012. Doctors In Training.com. LLC. All Rights Reserved.


3 Question Warm-Up
1. What are the characteristic features of a cell undergoing apoptosis? (FAQ p244)

2. What are some of the substances that can trigger apoptosis?

3. What are some of the cell types that are derived from the neural crest? (FA 12 pi 26)
Peripheral Nervous System
Ear
Eye
Adrenal Gland
Mouth
Heart
Digestive System
Thyroid
Skin

Cellular Response I - Acute and Chronic Inflammation


R: Chapter 2

Inflammation (FAII p22l)


- Acute inflammation o
- Leukocyte extravasation (FAII p222) c
- Mediators of inflammation (R p56)
Patterns of inflammation (R p66)
z
- Serous
- Fibrinous
- Suppurative %
Transudate vs. exudate (FAII p223)
o
Chronic inflammation
- Role of macrophages (R p7l)
- Granulomatous diseases (FAII p223) (R p73)
zc/->
Markers of inflammation Ln
- Erythrocyte sedimentation rate (FAII p223)
- C-reactive protein (R p74)
- Systemic effects (R p74) o
CD

I
C
D

5'
CO.

Copyright @ 2012. Doctors In Training.com. LLC. All Rights Reserved. [15]


4. Elevated ESR (FAI I p223, FAI2 p248)
• Polymyalgia rheumatica
• Temporal arteritis
• Disease activity in RA and SLE
• Infection, Inflammation (e.g., osteomyelitis)
• Malignancy

5. C-Reactive Protein (R p498)


• Acute-phase reactant synthesized by the liver
• Part of the innate immune response: opsonizes bacteria and activates complement
• Can be secreted from cells within atherosclerotic plaques to activate local endothelial cells to induce a
prothrombotic state and increase the adhesiveness of the endothelium to leukocytes
• Elevations are a strong predictor of Ml. stroke, PAD, and sudden cardiac death
• CRP can be lowered by smoking cessation, exercise, weight loss, and statins

Cellular Response 2 - Control of the Extracellular Environment


R: Chapter 3
Collagen (R p95)
Collagen synthesis and structure (FAI2 p83)
Fibroblasts
Osteogenesis imperfecta (FAI2 p83)
Ehlers-Danlos syndrome (FAI2 p83)
Alport syndrome (FAI2 p84)
Elastin (FAI2 p84)
Mechanisms of angiogenesis (R p99)
Cutaneous wound healing (FAI2 p248) (R pl02)

10. Epithelial Wound Healing


• 0-3 hours - Hemorrhage and clotting
• 12-24 hours - Acute inflammation (PMN)
• 2-4 days - Macrophage infiltration and epithelial cell migration
• 3-5 days - Granulation tissue (especially at wound edges)
• Months - Collagen production (type III then type I)

Copyright © 2012, Doctors In Traimng.com. LLC All Rights Reserved.


What is the underlying dysfunction in Chediak- rome?(FAI2p8l)

How does having a high cholesterol content in the plasma membrane affect the function
of the plasma membrane? (FA 12 p8l)

What area of the colon is most susceptible to ischemic damage? (FAI2 p245)

Cellular Response 3 - Internal Adaptations


R: Chapter /, 3
Strengthening
- Hypertrophy vs. hyperplasia (R p6)
Replacement
- Stem cells (R p82)
- Liver regeneration (R p93)
Irreparable damage
- Metaplasia

4. Where can tissue stem cells be found in the following locations?


Bone Hematopoietic stem cells in the marrow
Liver Oval cells in the canals of Hering
Brain Neural precursor cells in the subventricular zone
and dentate gyrus of the hippocampus
Skin • Hair follicle bulge
• Interfollicular areas of the surface epidermis
• Sebaceous glands
Intestine Deep within the intestinal crypts
Skeletal muscle Satellite cells beneath the myocyte basal lamina
Cornea Limbal stem cells at the cornea-epithelium junction

Copyright © 2012, Doctors In Training.com. LLC. All Rights Reserved.


ick Quiz
What female hormone is known for

Does the compensatory growth of muscle fibers occur primarily as a result of hyperpla
or hypertrophy? (R p6)

Does myometrial growth in pregnancy occur primarily as a result of hyperplasia or


hypertrophy? (R p7)

What organelle becomes hypertrophied in hepatocytes with chronic phenobarbftal


use?(R p7)

Which four transcription factors appear to be essential in pluripotential stem cells? (R p84

What can happen to the cells of the lower esophagus in response to chronic acid
reflux? (Rp 10)

Cellular Response 4 - Atrophy and Aging


R: Chapter I

Atrophy (FAII p22l)(Rp9)


Cellular aging and telomerase (R p39-40)

Quiz
What is actually occurring at the cellular level during atrophy? (R plO)

What is a lipofuscin granule? (R plO)

What enzyme mitigates the aging effects of cellular division by maintaining chromoson
length? (R p40)

.. What is currently the known as the most effective way of prolonging life span? (R p4l]

15. What family of proteins is induced by caloric restriction? (R p4l)

Copyright © 2012. Doctors In Traming.com. LLC. AH Rights Reserved.


0. How can a stroke of the facial motor cortex be distinguished from Bell's palsy?

iat should you immediately think about in a patient with "bilateral Bell's palsy"?

How do the symptoms of a lesion to the cortical motor region of the face differ from
lesion of the facial nerve or nucleus?
Lesion in cortical motor face region-
- Lesion of facial nerve or nucleus-

3. A patient comes to your office and before you notice any other symptoms, you see th
the patient's uvula deviates to the right when she says "Ah." What neurological areas
might be damaged in order for this abnormality to be seen?

4. A patient has leftward deviation of the tongue on protrusion and has a right sided spastic
paralysis. Where is the lesion?

j. A patient cannot blink his right eye or seal his lips. What is the diagnosis and which ne
is affected?

Copyright'<) 2012. Doctors In Training.com. LLC. All Rights Reserved.


Which type of vasculitis fits the following description? (FAI I p277-279) (FAQ p304-305)
• Necrotizing granulomas of lung and necrotizing glomerulonephritis
• Necrotizing immune complex inflammation of visceral/renal vessels
• Young Asian women
• Young asthmatics
• Infants and young children; involved coronary arteries
• Most common vasculitis
• Associated with hepatitis B infection

What portion of the brain is supplied by the anterior cerebral artery? Middle cerebral
artery? (FAI I p404) (FAQ p442)

How does the presentation of a right parietal lobe lesion differ from the presentation of a
left parietal lobe lesion? (FAI I p403) (FAQ p44l)

Brainstem Occlusion Syndromes I


H: Chapter 370
Effects of strokes (FAII p405) (FAI2 p444) (H p3284)

4. What is the hallmark sign of a general brainstem lesion?

Alternating syndromes: with long tract symptoms on one side (i.e., hemiparalysis) and cranial nerve
Symptoms on the other

5. What causes and what are the symptoms of Weber syndrome?


Midbrain infarction resulting from occlusion of the paramedian branches of the posterior cerebral
artery:
- Cerebral peduncle lesion ■▶ contralateral spastic paralysis (AKA contralateral hemiparesis)
- Oculomotor nerve (CN III) palsy ■▶ ipsilateral ptosis, pupillary dilation, and lateral strabismus (eye
looks down and out)

6. What causes and what are the symptoms of medial medullary syndrome? Damage to
which areas cause these symptoms?
Caused by occlusion of a paramedian branch of anterior spinal artery (from vertebral artery) ■»
unilateral infarct of medial portion of rostral medulla (AKA anterior spinal artery syndrome)
- Contralateral spastic hemiparesis (pyramid/corticospinal tract damage)
- Contralateral tactile and kinesthetic defects (medial lemniscus damage)
- Tongue deviates toward side of the lesion (hypoglossal nucleus/nerve damage)
- (Note that pain and temperature sensation are preserved)

Copyright fc> 2012. Doctors In Training.com, LLC. All Rights Reserved.


7. What causes and what are the symptoms of lateral medullary syndrome (AKA
Wallenberg syndrome)? Damage to which areas cause these symptoms?
Caused by occlusion of one of the posterior inferior cerebellar arteries (PICA) ■» unilateral infarct of
lateral portion of rostral medulla (AKA posterior inferior cerebellar artery syndrome)
- Loss of pain and temp, over contralateral body (spinothalamic tract damage)
- Loss of pain and temp, over ipsilateral face (trigeminothalamic tract damage)
- Hoarseness, difficulty swallowing, loss of gag reflex (nucleus ambiguus: glossopharyngeal and vagus
damage)
- Ipsilateral Horner syndrome (descending sympathetic tract)
- Vertigo, nystagmus, nausea/vomiting (vestibular nuclei damage)
- Ipsilateral cerebellar deficits (i.e., ataxia, past pointing) (inferior cerebellar peduncle damage)

Brainstem Occlusion Syndromes 2


H: Chapter 370

MLFsyndrome (FAII p424) (FAI2 p464) (H p239)


Locked-in syndrome (H p2247)

10. What is the cause of medial inferior pontine syndrome? What are the symptoms? ",
Damage to which areas cause these symptoms? (
Caused by occlusion of a paramedian branch of the basilar artery ■▶ unilateral infarct of medial aspect of Q\
inferior pons |
- Contralateral spastic hemiparesis (corticospinal tract damage) /
- Contralateral loss of light touch/vibratory/kinesthetic sensation (medial lemniscus damage) (
- Paralysis of gaze to side of lesion (damage to pontine gaze center: PPRF and abducens nucleus)
- Ipsilateral paralysis of lateral rectus muscle (damage to abducens nerve fibers) |
- (Note that pain and temperature sensation are preserved) !

Copyright © 2012. Doctors In Training.com. LLC. All Rights Reserved


What is the cause of lateral inferior pontine syndrome? What are the symptoms?
Damage to which areas cause these symptoms?
Caused by occlusion of one of the anterior inferior cerebellar arteries (AICA) ■▶ unilateral infarct of
lateral aspect of inferior pons (AKA anterior inferior cerebellar artery syndrome)
- Ipsilateral facial nerve paralysis (facial nucleus and nerve fiber damage)
- Ipsilateral loss of taste from anterior 2/3 of tongue (solitary nucleus and nerve fibers)
- Ipsilateral deafness and tinnitus (cochlear nucleus and nerve fiber damage)
- Nystagmus, vertigo, nausea/vomiting (vestibular nucleus and nerve fiber damage)
- Ipsilateral limb and gait ataxia (damage to middle and inferior cerebellar peduncles)
- Ipsilateral loss of pain and temperature sensation from the face (spinal trigeminal nucleus and nerve
fiber damage)
- Contralateral loss of pain and temperature sensation (damage to spinothalamic tract)
- Ipsilateral Horner syndrome (damage to descending sympathetic tract)
- (No contralateral body paralysis or loss of light touch/vibratory/kinesthetic sensation)

superior pontine syndrome. What are the symptoms of this syndrome? Damage to
which areas cause these symptoms?
- Ipsilateral loss of taste from anterior 273 of tongue (solitary nucleus and nerve fibers)
- Ipsilateral limb and gait ataxia (damage to middle and inferior cerebellar peduncles)
- Ipsilateral loss of pain and temperature sensation from the face (spinal trigeminal nucleus and nerve
fiber damage)
- Ipsilateral loss of light touch and vibration sensation from face (main sensory trigeminal nucleus and
nerve fiber damage)
- Ipsilateral jaw weakness and deviation of jaw toward side of lesion (trigeminal motor nucleus and
nerve fiber damage)
- Contralateral loss of pain and temperature sensation from body (damage to spinothalamic tract)
- Ipsilateral Horner syndrome (damage to descending sympathetic tract)

A patient cannot abduct her left eye on lateral gaze and convergence is normal. She is
also having difficulty smiling. In what part of the CNS is there a lesion?

A lesion of what artery can cause a locked-in syndrome?

.-■■«.. is the clinical presentation of a patient with internudear ophthalmoplegia?


(FAI I p424) (FAQ p464)

Copyright © 2012. Doctors In Training.corn. LLC. All Rights Reserved.


On auscultation of a patient, you hear a pansystolic murmur at the apex with radiation
the axilla. What is the most likely cause of this murmur? (FAI I p259) (FAQ p285)

Your study shows that high LDL does not increase one's risk of CAD. What type of
error is this? (FAI I p56) (FAQ p57)

What medication inhibits alcohol dehydrogenase? Which medications inhibit


acetaldehyde dehydrogenase? (FAI I p246)

Arterial Events
R: Chapter 28
H: Chapters 370, 378

Aneurysms (FAII p405) (FAI2 p445)


Intracranial hemorrhage (FAII p406) (FAI2 p446) (R pl288, 1295) (H p3293)
- Subarachnoid hemorrhage
- Epidural hematoma (H p3379)
- Subdural hematoma (H p3379)
- Parenchymal hemorrhage (H p3294)
- Interventricular hemorrhage
Ischemic brain disease (FAII p407) (FAI2 p447) (H p327l)

4. Intraventricular hemorrhage in the newborn


• Hemorrhage into the ventricular system
• Most common in premature/very low birth weight infants (< 32 wks, <l500g) within the first 72 hours
of life
• Originates from the germinal matrix in the subependymal, subventricular zone that gives rise to neurons
and glia during development
• All infants born at younger than 30-32 wks gestational age should receive a screening US to detect

Copyright © 2012, Doctors In Training.com. LLC. All Rights Reserved.


with berry aneurysms?

A 28-year-old woman is involved in a motor vehicle accident (MVA). She initially feels fine,
but minutes later she loses consciousness. CT scan reveals an intracranial hemorrhage that
does not cross suture lines. Which bone and vessel were injured in the crash?

A 40-year-old man with a history of Marfan syndrome and hypertension presents with
a severe headache. A head CT is normal at presentation and examination of the CSF
reveals numerous red blood cells. What is the cause of the man's headache?

An 85-year-old man with Alzheimer disease falls at home and presents 3 days later with
severe headache and vomiting. What is the most likely diagnosis and structures were
damaged?

Venous and Ventricular Events


R: Chapter 28
Dural venous sinuses (FAII p407) (FAI2 p447)
Ventricular System (FAII p408) (FAI2 p448) (COA p878)
Hydrocephalus (FAII p408) (FAI2 p448) (R pi283)
Pseudotumor cerebri (H pi26)

9. Circulation of CSF (COA p88l)

Lippincott Williams 6 Wilkms Atlas of Anatomy ©200S Wolters Kluwer Health. Inc. All rights reserved.

Copyright © 2012, Doctors In Training.com. LLC. All Rights Reserved.


PSEUDOTUMOR CEREBRI (AKA Benign Intracranial Hypertension)
10. What are the characteristic features of pseudotumor cerebri?
• Young, obese female
• Headaches - daily (worse in the morning) pulsatile, possible nausea/vomiting, possible retroocular pain
worsened by eye movement
• Papilledema
• Most worrisome sequelae is vision loss
• CT scan: Absence of ventricular dilation, no tumor or mass

What treatment options are available for managing pseudotumor cerebri?


• Confirm absence of other pathology with CT and MRI of the head (r/o central venous thrombosis)
• Discontinue any inciting agents (e.g. vitamin A. tetracyclines, corticosteroid withdrawal)
• Weight loss in obese patients
• Acetazolamide - first line (start 250mg qid or 500mg bid ■* increase to 500mg qid to lOOOmg qid)
• Invasive treatment options
- Serial lumbar punctures
- Optic nerve sheath decompression
- Lumboperitoneal shunting (CSF shunt)

Where is CSF generated? Where is CSF reabsorbed? (FAI I p408) (FAQ p448)

14. What is the difference between communicating and noncommunicating hydrocephalus?


(FAII p408) (FAQp448)

clinical features characterize normal pressure hydrocephalus? (FAI I p408)


(FAI2p448)

Copyright © 2012. Doctors In Training.com, LLC. All Rights Reserved.


What are five hereditary thrombosis syndromes? (FAI I p356) (FAQ p388)

Which cancer drugs are known for being cardiotoxic? (FAI I p36l, 363) (FAQ p400)

Movement Execution
Phys: Chapters 54, 55
Lower motor neurons
-Reflex arc (P p658)
- Clinical reflexes (FAII p4l4) (FAI2 p454)
- Primitive reflexes (FAII p4l4) (FAI2 p454)
Upper motor neurons
- Motor cortex (Phys p667)
- Thalamus (FAII p399) (FA12 p437)

Copyright © 2012. Doctors In Training.com. LLC. All Rights Reserved.


4. Muscle Spindle Control (Phys P657, 661)
Muscle spindle: monitors muscle length
• Extrafusal muscle fibers: functional unit of muscle
• Intrafusal muscle fibers: regulate length
- Muscle stretch results in intrafusal stretch which stimulates la afferent ■* DH ■» alpha motor
neuron, causing reflex muscle contraction
- Hint: help you pick up a heavy suitcase
• Golgi tendons: monitor tension rather than length (perpendicular to intrafusal muscle fibers)
- Provide inhibitory lb afferent feedback
- Hint: cue you to drop a heavy suitcase
Gamma loop: regulates sensitivity of reflex arc
• CNS ■» the Y motor neuron ■» contracts intrafusal fiber (central part of spindle), increasing the
sensitivity of the reflex arc

Muscle Spindle, Gamma Loop, & Golgi Tendon Organ

Golgi
tendon

Descending
pathway
Intrafusal •
muscle
fibers

Muscle
spindle

Extrafusal
muscle
fibers

5. What portion of the thalamus relays the following information? (FAI I p399) (FAI2 p437)
• Somatosensory from body (via medial lemniscus and spinothalamic)
• Communications with prefrontal cortex; memory loss results if destroyed
• Cerebellum (dentate nucleus) and basal ganglia ■▶ motor cortex
• Trigeminothalamic and taste pathways to somatosensory cortex
• Retina ■▶ occipital lobe
• Basal ganglia ■» prefrontal, premotor, and orbital cortices
• Mamillothalamic tract ■▶ cingulate gyrus (part of Papez circuit)
• Integration of visual, auditory, and somesthetic input
• Dentate nucleus and basal ganglia ■* supplementary motor cortex
• (Auditory info) brachium of inferior colliculus ■» primary auditory cortex

Copyright .' 2012. Doctors In Training.com. LLC. All Rights Reserved.


6. What are the differences b
motor pathway?

7. What portion of the thalamus relays the following information? (FAI I p399) (FAQ p437)
• Somatosensory from body (via medial lemniscus and spinothalamic)
• Cerebellum (dentate nucleus) and basal ganglia ■» motor cortex
• Trigeminothalamic and taste pathways to somatosensory cortex
• Retina ■» occipital lobe
• Basal ganglia ■* prefrontal, premotor, and orbital cortices
• (Auditory info) Brachium of inferior colliculus ■▶ primary auditory cortex

What reflexes might you check for in a newborn exam to assess an infant for healthy
neurological function? (FAI I p4l4) (FAQ p454)

Modifiers of Movement
Phys: Chapter 56
Lateral corticospinal tract (FAII p4IO) (FAI2 p450) (Phys p669)
Motor neuron signs (FAII p4IO) (FAI2 p450)
Cerebellum (FAII p399) (FAI2 p437) (Phys p68l)
Tremor (FAII p402) (FAI2 p439) (H p3327)

CEREBELLUM (FAII p399) (FAQ p437) (Phys p682)


9. What are the longitudinal zones of the cerebellum starting with the most medial?
• Vermis
• Intermediate (paravermal) zones (right and left)
• Lateral hemispheres (right and left)
Lateral Intermediate
Eart of part
emisphere He of

Anterior Lobe
Anterior Lobe,
y Primary Fissure
.fco. Midbrain
'Culmen'
I I ■ Posterior
i i Lobe
I Decllve i
Posterior
Lobe I Folium I

Flocculonodular
Medulla Lobe

Posterolateral Fissure
" Flocculonodular
Flocculus
Lobe

Copyright © 2012. Doctors In Training.com. LLC. All Rights Reserved.


10. Simplified Transverse Section from the Cerebellar Cortex

Stellate
v-r*—i e eTT^ ■ v
Molecular Layer
Basket
~^M-

Purkinje Layer Purkinje w±

Granule Layer Golgi #-» <# Granular

IPSP to deep Climbing fibers Mossy fibers


cerebellar nuclei
and vestibular nuclei

11. Describe the general flow of information through the cerebellum.


Inputs (mossy and climbing fibers) ■▶ cerebellar cortex ■* Purkinje fiber ■*• deep nuclei of cerebellum
■» output targets

oo

Copyright © 2012. Doctors In Training.com, LLC. All Rights Reserved. [29]


12. What structure provides the major output pathway from the cerebellum?
Brachium conjunctivum (AKA superior cerebellar peduncle) ■▶ contralateral VL of thalamus

13. Based on the primary source of information brought into the cerebellar cortex, which
cerebellar regions are referred to as the vestibulocerebellum, spinocerebellum, and
cerebrocerebellum? (To which deep nuclei do these regions project?)
Vestibulocerebellum - Flocculonodular lobe and vermis (■» fastigial)
Spinocerebellum - Vermis and paravermal regions (■▶ fastigial and interposed)
Cerebrocerebellum - Lateral hemispheres (■» dentate)

Spinocerebellum

Intermediate
hemisphere
.^brocerebellum

H
(lateral hemisphere)/' ,

Denate
nucleus
Interposed

Vestibulo
cerebellum

To m o t o r To l a t e r a l To m e d i a l
and premotor descending descending To vestibular
cortices systems systems nuclei

Motor Motor Balance


planning execution and eye
movement
14. Motor control on which side of the body would be affected with a lesion on one side
of the cerebellar hemisphere?
Motor control ipsilateral to the side of the lesion would be affected (because the cerebellum ■»
contralateral thalamus ■» cortex ■▶ corticospinal tract ■▶ body contralateral to cortex).

15. What neurological abnormalities can be attributed to damage of the spinocerebellum


(vermis and paravermis)?
• Postural instability
• Slurred/slowing of speech
• Hypotonia
• Pendular knee jerk reflexes

Copyright S3 2012, Doctors In Training.com. LLC. All Rights Reserved.


16. What symptoms are seen in anterior lobe (anterior vermis) syndrome? What is the
most common cause of anterior lobe syndrome?
The most anterior portion of the vermis belongs to the legs.
• Ataxia/dystaxia of legs (even when the trunk is supported) ■▶ broad-based, staggering gait
• Chronic alcohol abuse ■* thiamine deficiency ■▶ degeneration of cerebellar cortex (starting at
the anterior lobe)

17. What neurological deficits can be attributed to damage of the cerebrocerebellum


(lateral hemisphere)?
• Lack of coordination of voluntary movements (with respect to both the timing and rate of the
movement)
• Delays in initiating movements and trouble stopping movements
• Dysmetria (impaired ability to control the distance, speed, and power of a movement)
• Intention tremor

TREMOR (FAI I p402) (FAI2 p439)


18. What is the difference between essential tremor, resting tremor, and intention tremor?
(FAI I p402)
• Essential tremor- family history of tremor, occurs with movement and at rest
• Resting tremor- associated with Parkinson disease, disappears with voluntary movement
• Intention tremor- assodated with cerebellar damage, appears only with voluntary movements

19. What are the features of essential (AKA familial) tremor?


• Rapid fine tremor of head, hands, arms, and/or voice (think Katharine Hepburn)
• 50% of patients have a family history of tremor
• Occurs with movement and at rest
• Treated with beta-blocker (propranolol), primidone (an anticonvulsant), clonazepam, or alcohol (pts
often self medicate)

20. What neurological deficits can be attributed to damage of the vestibulocerebellum


(vermis and flocculonodular)?
• Disequilibrium: difficulty in maintaining balance
• Abnormal eye movements (such as cerebellar nystagmus that is more pronounced when patient looks Z.
to the side of the lesion) Q]

21. What is the most common cause of damage to the flocculonodular lobe? Q
Medulloblastoma in childhood CO
I
zo
$
3
C
D
13

Copyright ® 2012. Doctors In Training.com. LLC. All Rights Reserved. [31]


raHlRraRBwWTP!
to affect movement? (FAI I p4IO) (FAI2 p450)

What diseases specifically damage the area above? (FAI I p4l I) (FAQ p45l)

What are classic signs of an upper motor neuron lesion? Of a lower motor neuron lesion?
(FAII p4IO)(FAI2p450)

What is the difference between essential tremor, resting tremor, and intention tremor?
(FAI I p402) (FAI2 p439)

Copyright © 2012, Doctors In Trainmg.com. LLC. All Rights Reserved.


estion Warm-
What is the rate-limiting step in heme synthesis? (FAI I p354) (FAQ p386)

What is the structure of HbH? What disease results in HbH production? What is the
structure of Hb Bart's? What disease results in Hb Bart's production? (FAI I p349)
(FAQ p380)

What excitatory neurotransmitter is involved in pain?

Basal Ganglia and Hypokinesis


Phys: Chapter 56
H: Chapter 372

Basal ganglia (FAII p400) (FAI2 p438) (Phys p690)


Parkinson disease (FAII p40l) (FAI2 p438) (R pl3l9) (Phys p693) (H p33l7)
Surgical treatment for Parkinson disease (H p3324)
- Hemiballismus (FAII p40l) (FAI2 p439) (H p3332)
Parkinson medications
- Parkinson disease drugs (FAII p435) (FAI2 p478) (H p332l) (GG p6l4)
- L-dopa (levodopa)-carbidopa (FAII p435) (FAI2 p478) (Hp332l) (GGp6l4)
- Selegiline (FAII p436) (FAI2 p478) (GGp6l8)
Differential diagnosis of hyporeflexia

4. Basal Ganglia

Neostriatum r ■-iri.vif.iiH mi
pars reticula.
ct direct
Substantia Nigra
ay pathway pars compai

MMitirrpTrT
IflHjnaiEluEOaSa

Subthalamic
Nucleus

Copyright© 2012. Doctors In Training.com. LLC. All Rights Reserved.


What are the cardinal features of Parkinson disease? (FAI I p40l) (FAQ p438)

28-year-old chemist presents with MPTP exposure. What neurotransmitter is depleted?

How do the following structures normally impact movement globus pallidus internal
segment, subthalamic nucleus, and substantia nigra pars compacta?

Hyperkinesis
R: Chapter 28
H: Chapter 372

Types of hyperkinesis and dyskinesis


- Chorea (FAII p40l) (FAI2 p439)
- Athetosis (FAII p40l) (FAI2 p439)
- Myoclonus (FAII p40l) (FA12 p439)
- Dystonia (FAII p40l) (FAI2 p439) (H p3328)
Huntington disease (FAII p40l) (FAI2 p439) (R pi322) (H p3330)
Huntington drugs (FAII p436) (FAI2 p479) (GGp622)
Differential diagnosis of hyperreflexia

8. A man in his 40s begins to develop eariy dementia and uncontrollable movements of his
upper extremities. In what portion of the brain do you expect to see atrophy

A male patient presents with involuntary flailing of one arm. Where is the lesion?

What are the Cs of Huntington disease? (FAI I p40l) (FAQ p439)

What neurotransmitters are altered in Huntington disease? (FAI I p40l) (FAQ p439)

Copyright © 2012, Doctors In Training.com. LLC. All Rights Reserved.


What defect is associated with the following type of murmur? (FAI I p259) (FAQ p28 ,
• Crescendo-decrescendo systolic murmur best heard in the 2nd-3rd right interspace close to ■
sternum
• Rumbling late diastolic murmur with an opening snap
• Pansystolic (AKA holosystolic or uniform) murmur best heard at the 4th-6th left intercostal
spaces
• Continuous machine-like murmur (in systole and diastole)

What organisms are known for causing endocarditis? (FAI I p275) (FAQ p30l)

What four drug regimen is used to treat TB? (FAI I pl9l) (FAQ p2Q)

Spinal Cord Basics


COA: Chapter 4

Spinal cord and associated tracts (FAII p409) (FAI2 p449)


Spinal nerves (FAII p408) (FAI2 p448)
Spinal cord - lower extent (FAII p409) (FAI2 p448)
Spinal tract anatomy and functions (FAII p4IO) (FAI2 p450)

4. Label the cross-sectional view of the spinal cord.

Copyright © 2012. Doctors In Training.com. LLC. All Rights Reserved.


Which spinal tract conveys the following information? (FAI I p409) (FAQ p449)
• Touch, vibration, and pressure sensation
• Voluntary motor command from motor cortex to body
• Voluntary motor command from motor cortex to head/neck
• Alternate routes for the mediation of voluntary movement
• Pain and temperature sensation
• Important for postural adjustments and head movements
• Proprioceptive information for the cerebellum

Where does each of the following spinal tracts decussate/cross over? (FAI I p4IO)(FAQ p450)
• Dorsal columns
• Lateral corticospinal
• Spinothalamic tract

Spinal Cord Lesions


R: Chapter 28
H: Chapters 373, 374, 377, 380

Spinal cord lesions (FAII p4ll) (FAI2 p45l) (H p377)


Poliomyelitis (FAII p4ll) (FAI2 p45l)
Oligodendroglia (FAII p396) (FAI2 p434)
Multiple sclerosis (FAII p425) (FAI2 p465) (R pl3IO) (H p3395)
Internuclear ophthalmoplegia (FAII p424) (FAI2 p464) (H p239)
Werdnig-Hoffman disease (FAII p4ll) (FAI2 p45l)
Amyotrophic lateral sclerosis (FAII p4ll) (FAI2 p452) (R pi324) (H p3345)
Tabes dorsalis (FAII p4l2) (FAI2 p452)
Friedreich ataxia (FAII p4l2) (FAI2 p452)
Brown-Sequardsyndrome (FAII p4l2) (FAI2 p452)

7. Selected hereditary ataxias:


•• Friedreich ataxia (FAI I p4l2) (FAI2 p452) (H p3343)
• Hereditary vitamin E deficiency (mutations in the alpha-tocopherol transfer gene protein)
• Ataxia-telangiectasia (FAI I p2l4) (FAI2 p238) (R pl323) (H p3343)
• Metachromatic leukodystrophy (FAI I pill, 426) (FA 12 pi 16)
• Wilson disease (FAI I p333) (FA12 p364,466) (H p3334)

Copyright © 2012. Doctors In Training.com, LLC. All Rights Reserved.


What are the findings of Brown-Sequard syndrome? (FAI I p4Q) (FAQ p452)

vnat is the classic presentation of a syringomyelia? What malformation is associated


syringomyelia? (FAI I pi27,411) (FAQ pl3,451)

... What clinical presentation would lead you to suspect amyotrophic lateral sclerosis as a
diagnosis? (FAI I p4l I) (FAQ p452)

What are some of the more classic presenting scenarios for multiple sclerosis? (FAI I p425)
(FAQ p466)

Copyright © 2012. Doctors In Training.com. LLC. All Rights Reserved


What is pulsus paradoxus, and what,

With what hematologic disease would you expect to see the following?
• (+) Ham's test
• Heinz bodies
• Basophilic stippling
• (+) Osmotic fragility test
• (+) DEB test
• D-dimer
• Coomb's (+)
• Coomb's (-)
• (+) Ristocetin test

Outline the flow of fetal circulation (FAI I pQ5) (FAQ pi 32)

Brachial Plexus Anatomy


COA: Chapter 6

Brachial plexus (COA p72l) (H p347l)


Brachial plexus lesions (FAII p373) (FAI2 p408)
Erb-Duchenne palsy (FAII p375) (FAI2 p4IO)
Klumpke palsy and thoracic outlet syndrome (FAII p375) (FAI2 p4IO)

Copyright © 2012. Doctors In Trainmg.com. LLC. All Rights Reserved.


4. Brachial plexus in neck dissection

Lippmcott Williams 8 Wilkins Atlas of Anatomy ©2008 Wolters Kluwer Health, Inc. All rights reserved.

5. Label the following diagram of the brachial plexus:

Trunks Cords ^^^^^^^^^^^J

"
r%
2
1 { ] ■■ W i l l
3
1 I II
^^^^^^fll

Copyright 3 2012. Doctors lnTraining.com. LLC. All Rights Reserved.


Label the following diagram cniai plexus:

lat are the symptoms of a lesion to the C5 and C6 nerve roots? (FAI I p375) (FAQ p4IO)

What are the symptoms of a lesion to the inferior trunk of the brachial plexus? (FAI I p375)
(FAI2p4IO)

JL> Nerves of the Upper Extremity


COA: Chapter 6
rd
•p Upper extremity innervation (FAII p372) (FAI2 p407)
Cj Upper extremity nerves (FAII p374) (FAI2 p409)
Ct3 Distortions of the hand (FAII p375) (FAI2 p4IO)

Jq Long thoracic nerve (FAII p376) (FAI2 p4ll)


Hand muscles (FAII p376) (FAI2 p376)

Copyright © 2012. Doctors In Trammg.com. LLC. All Rights Reserved.


9. What does the RADIAL nerve innervate, and what is seen if this nerve is damaged?

• Triceps - Cannot extend forearm, loss of triceps reflex


• Skin of posterior arm - Loss of sensation of posterior arm
• Deep branch
- Supinator & Brachioradialis - Loss of brachioradialis reflex
- Extensor muscles of forearm - WRIST DROP (cannot extend wrist)
- Abductor pollicis longus - WRIST DROP
- Skin of posterior forearm - Loss of sensation of posterior forearm
• Superficial branch
- Skin of lateral, posterior hand - Loss of sensation of lateral, posterior hand

Hint: radial nerve innervates the BEST: Brachioradialis, Extensors of wrist and fingers, Supinator, and Triceps
Saturday Night Palsy: Radial nerve compression against the spiral groove of the humerus ■> weak wrist and finger
extension, weak brachioradialis reflex, but normal triceps.

Lippincott Williams 8 Wilkms Atlas of Anatomy ©2008 Wolters Kluwer Health. Inc. All rights reserved.

Copyright © 2012. Doctors In Training.com. LLC. All Rights Reserved.


10. What does the ULNAR nerve innervate, and what is seen if this nerve is damaged?
• Flexor carpi ulnaris - CLAW HAND*
• Medial !4 of the flexor digitorum profundus - CLAW HAND*
• Deep branch
- Hypo thenar muscles - cannot adduct the thumb
- Adductor pollicis - cannot adduct the thumb
- Ulnar two lumbricals - cannot abduct tor adduct fingers
- All interosseous muscles - cannot abduct tor adduct fingers
• Superficial branch
- Skin over medial hand - loss of sensation of medial hand

*CLAW HAND - Cannot flex DIP joint 4th and 5th digits, atrophy of interosseus muscles and inability to extend
interphalangeal joints when trying to straighten fingers.

Lippincott Williams 8 Wilkins Atlas of Anatomy ©2008 Wolters Kluwer Health, Inc. All rights reserved.

Copyright © 2012. Doctors In Training.com. LLC. All Rights Reserved.


What does the MEDIAN nerve innervate, and what is seen if the nerve is damaged?
• Pronator teres - Cannot pronate
• Flexor carpi radialis - Weak wrist flexion
• Palmaris longus -Weak wrist flexion
• Flexor digitorum superficialis- Weak wrist flexion
• Flexor pollicis longus - Cannot flex thumb
• Pronator quadratus - Cannot pronate
• Lateral 1/2 of flexor digitorum profundus - BISHOP'S HAND / HAND OF BENEDICTIONf
• Radial two lumbricals - BISHOP'S HAND / HAND OF BENEDICTIONf
• Skin over lateral palm - Loss of sensation of lateral palm -
• Skin of distal first 3 Yi digits - Loss of sensation of distal first 3 */i digits y^T
• Recurrent branch /
- Thenar muscles -APE HAND:): (cannot oppose thumb) / (

fBISHOP'S HAND/HAND OF BENEDICTION - loss of PIP flexion in digits 1-3


and loss of DIP flexion in digits 2-3 ■* when patient attempts to make a fist,
digits 2 and 3 remain partially extended (COA p768)

+APE HAND - thumb movements are limited to flexion/extension in the


plane of the palm

CARPAL TUNNEL SYNDROME - median nerve entrapment at the


carpal tunnel (enclosed by the inelastic flexor retinaculum ventrally,
and the carpal bones dorsally)

Lippincott Williams 8 Wilkins Atlas of Anatomy ©2008 Wolters Kluwer Health, Inc. All rights reserved.

12. An elderly woman with chronic osteoarthritis and diffuse pain now presents with
numbness and tingling over the lateral digits of her right hand that sometimes radiates
up to the elbow. Exam reveals wasting of the thenar eminence. What is the diagnosis?

Copyright © 2012. Doctors In Training.com. LLC. All Rights Reserved.


13. What does the MUSCULOCUTANEOUS nerve innervate, and what is seen if this
nerve is damaged?
• Biceps - Weak arm and forearm flexion, and forearm supination
• Coracobrachial - Weak arm flexion
• Brachialis - Weak forearm flexion
• Skin of lateral forearm - Loss of sensation of the lateral forearm

Lippincott Williams 8 Wilkins Atlas of Anatomy ©2008 Wolters Kluwer Health. Inc. All rights reserved.

14. What nerves run with the following arteries?


- Dorsal scapular artery
Lateral thoracic artery
• Posterior circumflex artery
• Suprascapular artery
• Thoracodorsal artery
• Deep brachial artery
• Ulnar artery
• Brachial artery
• Anterior interosseous artery
• Posterior interosseous artery

Copyright © 2012. Doctors In Training com. LLC. All Rights Reserved.


Session Quiz
15. What nerve is damaged when a patient presents with the following symptom (upper
extremity)?
• Claw hand
• Ape hand
• Wristdrop
• Scapular winging
• Unable to wipe bottom

• Loss of forearm pronation


• Cannot ab- or adduct fingers

• Weak lateral rotation of arm


• Loss of arm and forearm flexion

• Loss of forearm extension


• Trouble initiating arm abduction
• Unable to abduct arm beyond 10 degrees
• Unable to raise arm above horizontal

What nerve is most at risk of injury with the following types of fractures/injury?
• Shaft of the humerus
• Surgical neck of the humerus
• Supracondyle of the humerus
• Medial epicondyle
I* Loss
• Anterior
of armshoulder
abduction
dislocation
• Injury to the carpal tunnel

17. A patient falls off a motorcycle and lands on his right shoulder On physical exam you
notice his shoulder has an abnormal configuration. X-rays indicate an anterior dislocatii
of his shoulder. What artery and nerve are most at risk of being damaged?

v nigh-school athlete falls on his arm during practice. In the ER, a radiograph shows a
midshaft break of the humerus. Which nerve and which artery have the highest risk of
being damaged? What muscular actions are affected?

A patient presents with decreased pain and temperature sensation over the lateral aspects "<
of both arms. Where is the lesion?

Copyright © 2012. Doctors In Tr.nning.com. LLC. All Rights Reserved.


What drug categories do the following medications fall under? (FAI I p247) (FAQ p274)
• Losartan
• Vecuronium
• Ticarcillin
• Desipramine
• Enalapril
• Lorazepam
• Rosiglitazone

Which branchial arches develop into the following structures? (FAI I pi26) (FAQ pl36)
• Common carotid artery
• Aortic arch
• Right subclavian
• Pulmonary arteries

What is the classic clinical presentation of athyrogiossal duct cyst? (FAI I pl3l) (FAQ pi31

_£j Lower Extremity


-^<D COA: Chapter 5
-^ Sensory
^' Lower extremity nerves
of the lower and muscles (FAII p376) (FAI2 p4ll)
limbs
"O Arteries of the leg

Copyright © 2012. Doctors In Training.com. LLC. All Rights Reserved.


4. LE Nerves (FAI I p376)

Femoral nerve Superior gluteal nerve


Psoas and iliacus. Tensor fascia latae
Pectineus Gluteus medius
Sartorius Gluteus minimus
Rectus femoris
Vastus Inferior gluteal nerve
- Lateralis Gluteus maximus
- Intermedius
- Medialis Sciatic nerve (tibial branch)
Biceps femoris
Obturator nerve Semitendinosus
Adductor Semimembranosus
- Magnus Adductor magnus
- Longus
- Brevis Sacral plexus
Gracilis Gemellus superior and inferior
Piriformis
(Pectineus)
Quadratus femoris
zm
Deep peroneal nerve Tibial nerve C
Peroneus tertius
Tibialis anterior
Gastrocnemius
Soleus S
Extensor hallicus longus Plantaris
Extensor digitorum longus Poplrteus
i1
Flexor hallucis longus
ii —
Superficial peroneal nerve Flexor digitorum longus O
Peroneus longus Tibialis posterior £
Peroneus brevis fD

Superficial peroneal nerve


Peroneus longus
Peroneus brevis 3

&
a
c
fl>

Copyright © 2012. Doctors In Training.com. LLC. All Rights Reserved. [47]


5. Femoral Nerve 6. Obturator Nerve

It ,,

(<*

~J.l
I4 1

Lippincott Williams 8 Wilkins Atlas of Anatomy ©2008 Wolters Kluwer Health. Inc. All rights reserved.

Copyright i<3 20I2. Doctors In Training.com, LLC. All Rights Reserved.


7. Sciatic and Common Fibular Nerve 8. Sciatic and Tibial Nerve

Uppincott Williams 8 Wilkins Atlas of Anatomy ©2008 Wolters Kluwer Health. Inc. All rights reserved.

Copyright ©2012. Doctors In Training.com. LLC. All Rights Reserved.


9. Lower Extremity Dermatomes

Lippincott Williams & Wilkins Atlas of Anatomy ©2008 Wolters Kluwer Health. Inc. All rights reserved.

Copyright G? 2012. Doctors In Training.com. LLC. All Rights Reserved,


»jt/h:c«
Exam of a patient revea
leg. What muscular defect would you also expect to be present?

A 20-year-old dancer reports decreased plantar flexion and decreased sensation overth
back of her thigh, calf, and lateral half of her foot. What nerve is involved?

, patient fractures her fibula neck. What nerve is most at risk of being damaged?

Skeletal Muscle
Phys: Chapters 6, 7

-^ Types of muscle fibers (FAII p378) (FAI2 p4l2)


^ Muscle conduction to contraction (FAII p377) (FAI2 p4l2) (Phys p83)
D Skeletal and cardiac muscle contraction (FAII p378) (FAI2 p4l3) (Phys p73)
>_ Dantrolene (FAII p435) (FAI2 p477)

0 14. Label the following sarcomere:

Copyright © 2012. Doctors In Training com. LLC. All Rights Reserved.


iiiij nPI/X'J' LL
Which two muscle receptors are responsible for opening the sarcoplasmic reticulum in
response to depolarization? (FAI I p377)

What drug prevents the release of calcium from the sarcoplasmic reticulum of skeletal
muscle? (FAI I p435)

17. For what conditions is this drug useful? (FAI I p435)

Which type of muscle fiber would usually be dominant in the gastrocnemius muscle?
(FAII p378)

Copyright© 2012. Doctors In Trainmg.com. LLC. All Rights Reserved.


WK91' L=±~i "f £»J i li/< liuZ
-Mhroir?;
A young woman is found to nave snore ^;ilSI8BBB^8i8BBEi^fflSB8miCTrMHE
stature ana snortenea tin ani
What endocrine disorder is most likely responsible for these manifestations?

What is the rate-limiting enzyme of beta-oxidation of fatty acids? (FAI I p95)

What is the treatment for nephrogenic diabetes insipidus? (FAI I p295)

Skin Sensation
Phys: Chapters 46-48
Sensory corpuscles (FAII p397) (FAI2 p435)
Additional sensory nerves
Landmark dermatomes (FAII p4l3) (FA12 p435)
Clinical important landmarks (FAII p37l) (FAI2 p405)
Peripheral nerve layers (FAII p397) (FAI2 p435)
Spinothalamic tract and dorsal columns (FAII p4IO) (FAI2 p450)
Schwann cells (FAII p396) (FAI2 p435)
Guillain-Barre syndrome (FAII p426) (FAI2 p466) (H p3473)

SENSORY RECEPTORS (FAI I p397) (FAQ p435)


4. Compare Merkel receptors, Meissner's corpuscles, Ruffini endings, and Pacinian
corpuscles based on location in the skin tissue and on whether they are slowly or
rapidly adapting.
Slowly adapting Rapidly adapting
Superficial layers

Deep layers (dermis)

5. What is the difference between a slowly adapting receptor and a rapidly adapting
receptor?
Slowly adapting: sends a continuous electrical signal throughout a continuous stimulus
Rapidly adapting: sends an electrical signal only at the beginning and end of a continuous stimulus

<U 6. What sensory receptor communicates with the following information? (FAI I p397)
°° (FAI2p435)
• Pricking pain (fast, myelinated)
_ • Burning or dull pain and itch (slow, unmyelinated)
-\ • Receptor for cold sensation
•s • Receptor for warm sensation
D • Vibration and pressure
Hz • Dynamic/changing light, discriminatory touch
— • Static/unchanging light touch
• Proprioception information - muscle length monitoring
54 1J r * r Proprioception
C o p y r i g h t - W O U informationTmuscleJensionmonitoring,.
. L J o c t o i s I n T r a i m n g . c o m . ' e L C . A l l R i g h......
t s R eD
served.
7. What sensory receptor matches the following description?
• Resembles an onion in cross section
• Robust spindle-shaped structures found particularly on the soles of the feet
• Found only in areas of skin without hair (fingertips, lips, eyelids, etc.)
• Simplest sensory receptor thought to be pain receptor or thermoreceptors
• Touch receptor that is tough to distinguish from melanocytes

8. What sensory receptor communicates with the following information? (FAI I p397)(FAQ
• Pricking pain (fast, myelinated)
• Burning or dull pain and itch (slow, unmyelinated)
• Vibration and pressure
• Dynamic/changing light, discriminatory touch

• Proprioception information - muscle length monitoring


• Proprioception information - muscle tension monitoring

E" Static/unchanging light touch


9. What is the landmark for a pudendal nerve block? (FAI I p37l) (FAQ p405)

What CSF changes are present in Guillain-Barre syndrome? (FAI I p426) (FAQ p466)

.at are the classic manifestations of Guillain-Barre syndrome? (FAI I p426) (FAQ p4^

Other Demyelinating Diseases


R: Chapter 28

Other demyelinating and dysmyelinating diseases (FAI I p426) (FAQ p466) (R pi309)

Copyright & 2012. Doctors In Training.com. LLC. All Rights Reserved.


Otic Sensation
Phys: Chapter 55
H: Chapter 21
COA: Chapter 7
Tympanic membrane (COA p967)
Inflammatory ear diseases (R p754)
- Acute otitis media (COA p978)
- Otitis externa (COA p978)
- Cholesteatoma (H p249) (R p754)
Ossicles (COA p970)
Cochlea (COA p973)
Vestibular apparatus (Phys p674)
Vertigo (FAII p427) (FAI2 p467) (H pl78)

12. Weber Test


Normal ^Midline
Conductive hearing loss ■» lateralizes to the side of the affected ear
Sensorineural hearing loss ■▶ lateralizes to the side opposite the affected ear

13. Rinne Test


Normal ■» Air conduction > Bone conduction (AC>BC)
Conductive hearing loss ■» BC > AC

14. Given the following exam findings, what is the diagnosis?

Weber Rinne (left) Rinne (right) Diagnosis


Patient A Midline AOBC AOBC
Patient B Right AOBC BO AC
Patient C Left AOBC AOBC
Patient D Midline BC>AC BO AC

15. Cholesteatoma
• Overgrowth of desquamated keratin debris within the middle ear space that may eventually erode the
c ossicular chain and external auditory canal
o • Causes: negative middle ear pressure (chronic retraction pocket) from eustachian tube dysfunction or
to direct growth of epithelium through a TM perforation
c • Commonly associated with chronic middle ear infection
<L>
CO • PE: grayish-white "pearl/' lesion behind or involving the TM, conductive hearing loss, vertigo
• Treatment surgical removal usually involving tympanomastoidectomy and reconstruction of the
ossicular chain

o
D

[56] Copyright © 2012, Doctors In Training.com. LLC. All Rights Reserved.


Session Qu
■;s

Which organisms are most commonly responsible for acute otitis media7 (R p754)

What organism is most commonly responsible for otitis externa7

Chronic otitis media can sometimes result in a cystic lesion that is lined by keratinizing
squamous epithelium which can be metaplastic that is filled with amorphous debris. Wie
the name of this condition? (R p754)

Copyright O 2012. Doctors In Training.com. LLC. All Rights Reserved.


internal acoustic meatus. What is the diagnosis?

2. What are the different mechanisms by which heart contractility can be increased?

What medication is used to treat the following parasitic infection? (FAII pl60-l63)(FAI2pl75-l77)
• Trichomonas or Gardnerella
• Plasmodium vivax or ovale
I ( FA• IPediculosis
I p 2 5 5 ) ( FA
capitis
I 2 por
2 8pubis
l)

Eye Structures
COA: Chapter 7
H: Chapter 28
Eye and retina (FAII p420) (FAI2 p459) (COA p889)
Eye pathology (FAII p420) (FAI2 p460)
Aqueous humor pathway (FAII p420) (FAI2 p460)
Glaucoma (FAII p42l) (FAI2 p46l) (COA p9l2) (H p234)
Glaucoma drugs (FAII p430) (FAI2 p47l) (GGpl785)
Cataract (FAII p42l) (FAI2 p46l)
Papilledema (FAII p42l) (FAI2 p46l)

Glaucoma (fai i d42IWFai:


4. Outline the flow of aqueous humor.
Formed in a capillary bed in the ciliary body ■» secreted into the posterior chamber ■» flows between
the angle formed by the lens and the iris diaphragm ■▶ into anterior chamber ■» reabsorbed by the
canal of Schlemm

5. What is the pathogenesis of glaucoma?


Blocked canal of Schlemm ■▶ aqueous humor not reabsorbed ■» increased pressure ■» atrophy of optic

6. Open Angle Glaucoma


• Common, insidious form; almost always bilateral
• Risk factors: older than 40, black, diabetes
• Early stage- asymptomatic
• Late stage- areas of reduced/absent vision, contraction of visual field (peripheral ■* central)

7. Acute Angle-Closure Glaucoma


• Emergency
• Abrupt onset of pain, nausea, colored halos, rainbows around light
• Red, teary eye with hazy cornea and fixed, mid-dilated pupil (not reactive to light) that is firm to palpation

8. Diagnosis
• Cupping of the optic disk (cup:disc ratio greater than 1:2)
• Tonometry

Copyright <Q 2012. Doctors In Tramtng.com. LLC. All Rights Reserved.


Cataracts (faq p42i) (FA12 p46i)
9. What is the classic presentation of a patient that has a cataract?
• Painless, progressive decrease in vision manifested with difficulty driving at night, reading road signs, or
reading fine-print
• Usually bilateral, but often unilateral
• Near-sightedness is often an early manifestation
• Possible disabling

60-year-old male has a hard time driving at night because of worsening vision and the
appearance of halos around oncoming headlights. What is the diagnosis?

11. What 5 drug dasses are used in the treatment of glaucoma? (FAI I p430) (FAQ p47l)

12 How does the optic disc appearance differ in glaucoma when compared to hydrocephalus?
(FAII p42l) (FAQp46l)

(FAIIp42l)(FAI2p46l)

Neural Aspects of the Eye


H: Chapter 28
Retinal detachment (FAII p423) (FAI2 p463) (COA p9IO)
Age-related macular degeneration (FAII p423) (FAI2 p463) (H p235)
Pupillary control (FAII p422) (FAI2 p462)
Pupillary light reflex (FAII p422) (FAI2 p462) (COA p9ll)
Cranial nerve III in cross section (FAII p423) (FAI2 p463)
Visual field defects (FAII p423) (FAI2 p463) (H p227)

Pupillary Light Reflex (FAII p422) (FAI2 p462)


14. Outline the pupillary light reflex pathway.
Cells of retina ■* optic tract ■* optic tract/nerve ■* pretectal nucleus ■* bilateral Edinger-Westphal
nuclei ■* preganglionic parasympathetic fibers in oculomotor nerve ■» ciliary ganglion ■▶ postganglionic
parasympathetic fibers ■* pupillary sphincter of iris ■* pupillary constriction (miosis)

15. Describe what light reflexes will be seen in both eyes if the right optic nerve is
damaged prior to the pretectal nucleus (AKA afferent defect).
• No constriction of either the left or right eye when light is shined in the right eye
• Both pupils constrict if the light is shined in the left eye

16. Describe what light reflexes will be seen in both eyes if the right oculomotor nerve is
damaged (AKA efferent defect).
• Right eye will not respond to light shone in either the right or left eye
• Left eye will constrict when a light is shined in either eye

Copyright © 2012. Doctors In Training.com. LLC. All Rights Reserved.


What eye abnormality is seen in a p
(FAI I p424) (FAQ p464)

A patient cannot adduct her left eye on lateral gaze but convergence is normal. What
structure is damaged?

Light stimulus in patient's right eye produces bilateral pupillary constriction. When the
light is shown in the left eye, there is a paradoxical bilateral pupillary dilatation. What is
the defect?

A woman presents with headache, visual disturbance, and amenorrhea. What is the
diagnosis?

hat is the treatment for dry age-related macular degeneration? (FAI I p423) (FAQ
p463)

Copyright © 2012, Doctors In Training.com. LLC. All Rights Reserved.


What neoplasm is most commonly responsible for the following paraneoplastic syndrom.
(FAII p229) (FAQp255)
• ACTH ■* Cushing syndrome
• Erythropoietin ■> polycythemia
• ADH ■» SIADH

What are the endogenous agonists to the different opioid receptors? (FAI I p430) (FAQ p47l

Which medication fits the following description (FAI I p430) (FAQ p47l)?
• Opioid cough suppressant commonly used with the expectorant guaifenesin
• Opioid used in the treatment of diarrhea
• Opioid used in the treatment of acute heart failure
• Opioid receptor antagonist
• Non-addictive weak opioid agonist
• Partial opioid agonist that causes less respiratory depression

Dementia
R: Chapter 28
H: Chapter 371
Dementia (FAII p425) (FAI2 p465) (H p3300)
Alzheimer disease (FAII p425) (FAI2 p465) (R pl3l3) (Phys p727) (H p3305)
Alzheimer drugs (FAII p436) (FAI2 p479) (GG p6l9)
Pick disease (FAII p425) (FAI2 p465) (R pl3l8) (H p33IO)
Lewy body dementia (FAII p425) (FA/2 p465) (R pi321) (H p33l2)
Creutzfeldt-Jakob disease (FAII p425) (FAI2 p465) (H p33l2, 3441)
Prions (FAII pl75) (FAI2 pl95) (R pl308) (H p344l)
Other causes of dementia

4. Identify the neuronal pigment/inclusion that matches the following statement:


• Intranuclear inclusions seen in herpes simplex encephalitis
• Cytoplasmic inclusions pathognomonic of rabies
• Neuronal inclusions characteristic of Parkinson disease
• Cytoplasmic inclusion bodies associated with aging
• Dark cytoplasmic pigment in neurons of the substantia nigra and locus coeruleus, not seen in patients
with Parkinson
• Eosinophilic, rod-like inclusions in hippocampus of Alzheimer patients
• Diagnostic of Alzheimer disease
• Filamentous inclusions that stain with silver, do not survive neuronal death
• Filamentous inclusions that stain with PAS and ubiquitin

5. What diseases are associated with Lewy bodies?

Copyright © 2012. Doctors In Training.com. LLC. All Rights Reserved.


What are the usual components of a "dementia work-up"?

Which neurodegenerative disease matches the following statement?


• Lou Gehrig disease (ALS)
• Senile plaques, neurofibrillary tangles
• Presents at birth as "floppy baby"
• Lewy bodies
• Dementia often associated with frequent falls and/or syncope
• Atrophy of caudate nucleus ■▶ chorea
• Depigmentation of substantia nigra
• Both upper and lower motor neuron signs (spasticity and weakness)
• Dementia + visual hallucinations
• Dementia + personality changes and/or aphasia

What allele is associated with Alzheimer disease? Why is Alzheimer disease so common
in patients with Down syndrome? (FAI I p425) (FAQ p465)

What is the mechanism of action of the drugs used in the treatment of Alzheimer
disease? (FAI I p436)

^ Headache
~j2 H: Chapter 14
rd
^ Headache (FAII p427) (FAI2 p467)
•.p - Tension (H pi20)
'<~ - Migraine (H pi 14)
- Cluster (H pi22)
- Other headaches (H pi24-128)
Headache medications
\ - Sumatriptan (FA11 p436) (FAI2 p479) (H pi 19)

^ HEADACHE (FAI I P427) (FAQ p467)


10. What is the typical presentation of a tension headache?
O • Constant, chronic pain (lasts hours/days/weeks/months)
r~T- • Occurring in the frontal or occipital regions (most often bilateral) or as a band around the head

j-jl • No associated symptoms such as light/loud noise sensitivity, visual changes, nausea/vomiting, or focal
neurological changes

Copyright© 2012. Doctors In Training.com, LLC. All Rights Reserved.


11. What are the main diagnostic criteria for migraine without aura?
• At least 5 attacks
• Headache lasting 4-72hrs (2-48hrs in children)
• At least 2 of the following:
- Unilateral location
- Pulsating quality
- Moderate to severe intensity (inhibits or prohibits daily activities)
- Aggravated by dimbing stairs or similar activity
• At least I of the following: nausea and/or vomiting; photophobia and/or phonophobia

12. What are the typical symptoms of a cluster headache?


• Strictly unilateral
• Severe aching/boring pain in the retro-orbital/periorbital region
• (Doesn't throb like a migraine. No aura.)
• Duration of 30 min-3 hours, occurs daily (often at the same time), and continues for an interval of 4-8 weeks
• May be assodated with partial Horner syndrome (ptosis and miosis), ipsilateral nasal congestion or eye
redness, rhinorrhea, or tearing

13. What is the most likely cause of headache based on the following description?
Made worse by foods containing tyramine

Obese female with papilledema

Jaw muscle pain when chewing

Periorbital pain with ptosis and myosis


m
Photophobia and/or phonophobia C
Bilateral frontal/occipital pressure
S
Lacrimation and/or rhinorrhea
Ln
Elevated ESR I
"Worst headache of my life" u
fl>
Headache + extraocular muscle palsies 3
a>
Scintillating scotomata prior to headache
{D
Headache occurring either before or after orgasm (D
D
CL
Responsive to 100% oxygen supplementation
I
Trauma to the head ■▶ headache begins days after the D
f
PJ
event, persists for over a week and does not go away Q_
JD
O
ZT

Copyright © 2012. Doctors In Training.com. LLC. All Rights Reserved. [63]


tension headache? (FAI I p427) (FAQ p467)

What is the mechanism of action of sumatriptan? (FAI I p436) (FAQ p479)

What are the contraindications to sumatriptan use? (FAI I p436) (FAQ p479)

What is the usual treatment for migraine headaches in pregnancy?

What would suspect as a cause of headache in a patient using topical retinoic acid for
acne?

A 20-year-old woman that has migraine headaches each proceeded by an aura should
never be prescribed which medication?

Copyright © 2012. Doctors In Training.com. LLC. All Rights Reserved.


est ion Warm-Up
An ER trauma patient is found to have multiple masses scattered throughout the bo
and brain as seen on a trauma protocol CT scan of the head, neck, chest, abdomen,
and pelvis. Assuming that this represents multiple metastatic lesions, what do you
suspect as the primary source of the cancer given that metastases are found in the b
parenchyma? (FAI I p230) (FAQ p256)

abel the following diagram of lung volumes.

Copyright O 2012. Doctors In Trainmg.com, LLC. All Rights Reserved.


Brain Tumors
R: Chapter 28
H: Chapter 379

Primary brain tumors (FAII p428) (FAI2 p468) (R pl330) (H p3382)


Herniation syndromes (FAII p429) (FAI2 p470) (H p2248)
Uncal herniation (FAII p429) (FAI2 p470) (H p2248)

uick Quiz
What are the 3 most common primary brain tumors in adults? What are the 3 most
common in children?

Which primary brain tumor fits the following description?


• Pseudopalisading necrosis
• Polycythemia
• Neurofibromatosis II
• Associated with von Hippel-Lindau syndrome
• Foamy cells, high vascularity
• Hyperprolactinemia ■* galactorrhea, amenorrhea, anovulation
• Psammoma bodies
• Fried-egg appearance
• Perivascular pseudorosettes
• Bitemporal hemianopia
• worst prognosis of any primary brain tumor
• child with hydrocephalus
• Homer-Wright pseudorosettes

Anesthetics
GG: Chapter II, 17, 19,20
Barbiturates (FAII p432) (FAI2 p474) (GG p469)
Benzodiazepines (FAII p433) (FAI2 p475) (GG p458)
Anesthetics - general principles (FAII p433) (FAI2 p475) (GG p527)
Inhaled anesthetics (FAII p433) (FAI2 p476) (GG p539)
Intravenous anesthetics (FAII p434) (FAI2 p476) (GGp532)
Local anesthetics (FAII p434) (FAI2 p477) (GG p565)
Neuromuscular blocking drugs (FAII p435) (FAI2 p477) (GG p258)
Dantrolene (FAII p435) (FAI2 p477)

Copyright © 2012, Doctors In Training.com. LLC All Rights Reserved.


6. Depolarizing vs. Non-Depolarizing Neuromuscular Blockers

DEPOLARIZING BLOCK
NOIBBUG PHASE I PHASED
NONDEPOLARIZING BLOCK

Tr a i n - o f - f o u r i Constant but diminished ,

Te t a n u s ', Constant but diminished

Posttetanic . „
Absent Present "o
potentiation l"SJ D

"a

lllll '

7. Succinylcholine vs. Tubocurarine


SUCCINYLCHOLINE
TUBOCURARINE
PHASE I PHASE II

Succinylcholine Additive
AHminictratinn Augmented Antagonistic
Administration
Tubocurarine
Antagonistic Augmented Additive
Administration

Effect of neostigmine ] Augmented Antagonistic Antagonistic

Posttetanic facilitation

Rate of recovery ! 4-8 min > 20 min3 30-60 min

Response to a Unsustained Unsustained


tetanic stimulus ' Sustained3

Initial excitatory effect ]


Fasciculations
on skeletal muscle '

PHASE II

Succinylcholine Administration (Img/kg)

Copyright 3 2012. Doctors In Trainmg.com. LLC. All Rights Reserved.


Which anesthetic fits the following description? (FAI I p433-435) (FAQ p476-477)
• IV, associated with hallucinations and bad dreams
• Inhaled, SE nephrotoxic
• IV, most common drug used for endoscopy
• Inhaled, SE convulsions/seizures
• Inhaled, SE hepatoxic
• IV, used for rapid anesthesia induction & short procedures
• Inhaled, used for rapid anesthesia
• IV, decreases cerebral blood flow (important in brain surgery)
• IV, does not induce histamine release like morphine
• High triglyceride content increases risk of pancreatitis with long-term use

What is the mechanism of action of dantrolene? (FAI I p435) (FAQ p477)

). What is the mechanism of action of local anesthetics? Which nerve fibers are blocked
first with local anesthesia? (FAI I p434) (FAQ p477)

. What drugs can be used to reverse neuromuscular blockade? (FAI I p435) (FAQ p477)

Copyright © 2012, Doctors In Training.com. LLC. All Rights Reserved.


uestion Warm-Up
HElIirSsfflBffljfflHff!!^
*]ff^<lll*"lllL*J

' Polycythemia
■ Neurofibromatosis II

Hyperprolactinemia ■▶ galactorrhea, amenorrhea, anovulation


Psammoma bodies
Perivascular pseudorosettes
Loss of peripheral vision
• Worst prognosis of any primary brain tumor

A 25-year-old female presents with sudden uniocular vision loss and slightly slurred
speech. She has a history of weakness and paresthesia that have resolved. What is th
most likely diagnosis?

What is the classic triad of symptoms in multiple sclerosis? (FAI I p425) (FAQ p465)

Seizures
H: Chapter 369
Phys: Chapter 59
Seizures (FAII p426) (FAI2 p466) (Phys p725) (H p325l)
Sturge-Weber syndrome (FAII p427) (FAI2 p467)
Tuberous sclerosis (FAII p427) (FAI2 p467) (R pi342)

4. Trigeminal Neuralgia
• AKA Tic Douloureux
• "Lightning-like" pain, "Electric shocks," along a division of the trigeminal (usually maxillary) triggered by
light touch (wind, bed sheets.)
• Rx: carbamazepine or other anticonvulsant (phenytoin, gabapentin, topiramate)

5. Tuberous Sclerosis (FAI I p86, 424) (FAQ p90, 467)


• Autosomal dominant; 1/5,000- 1/10,000 live births
• Complete penetrance BUT variable expression
• Most common mutations are in the TSCI or TSC2 genes
- TSCI gene ■» hamartin protein
- TSC2 gene ■> tuberin protein
• Classic triad: seizures, mental retardation, angiofibromas
• Additional findings: hypomelanotic macules (ash-leaf spots), retinal hamartomas, cortical tubers
(glioneuronal hamartomas)
• Tumor associations (FAI I p222): renal angiomyolipoma, cardiac rhabdomyoma, astrocytoma
(subependymal giant cell astrocytoma

Copyright © 2012. Doctors In Traimng.com. LLC. All Rights Reserved.


In regards to seizures,' IiS8BRI!8i1Sitt: srsrsfe
(FAI I p426) (FAQ p466)

What are the most common causes of seizures in children? (FAI I p426) (FAQ p466)

8. A 10-year-old child "spaces-out" in class (stops talking midsentence and then continues as

diagnosihad
Iff nothing s? happened). During the spells, there is slight quivering of lips. What is the

Anti-Seizure Medications
H: Chapter 369
GG: Chapter 21

Epilepsy drugs (FAII p43l) (FAI2 p473) (H p326l-3267) (GG p590-606)


Epilepsy drug toxicities (FAII p432) (FAI2 p474) (GG p590-606)
Phenytoin (FAII p432) (FAI2 p474) (GGp59l)

9. Drug of choice for partial (simple and complex) and tonic-clonic seizures:

10.4 other drugs useful in partial and tonic-clonic seizures:

Drug of choice for absence seizures:

12. Used to treat status epilepticus and eclampsia:

<£} 13. Common side-effects of epilepsy drugs:

00 14. Additional SE of phenytoin:

— 15. Which anti-epileptics are teratogens?

16. What drugs cause Stevens-Johnson syndrome?

17. What drugs are known to cause agranulocytosis?

Copyright © 2012. Doctors In Training.com. LLC. All Rights Reserved.


18. Hepatotoxic anti-epileptics:

19. What drugs induce the P450 system?

20. Which anti-epileptics block Na+ channels?

21. Which anti-epileptics work by potentiating the effects inhibitory effects of GABA?

What drugs are known for causing Stevens-Johnson syndrome? (FAI I p245) (FAQ p2

13. How is barbiturate overdose managed? How is benzo overdose managed?


(FAI I p432-433) (FAQ p474-475)

.4. What side effects are common to most all of the anti-epileptics? (FAI I p432) (FAQ p474)

. What are the toxic side effects of phenytoin? (FAI I p432) (FAQ p474)

Copyright (-3 2012. Doctors In Training.com. LLC. All Rights Reserved.


the symptoms of hyperthyroidism in addition to the following findings? (FAI I p298) (FAQ
P325)(Physp9l6)
• Extremely tender thyroid gland
• Palpation of single thyroid nodule
• Palpation of multiple thyroid nodules
• Recent study using IV contrast dye (iodine)
• Eye changes: proptosis, edema, injection

What are the differences between oligodendroglia and Schwann cells?


(FAII p396) (FAQ p434-435) (RpQ58, 1282)

Describe the sensory innervation of the tongue. (FAI I pi 29) (FAQ pi 37) (Phys p647)

Immunology Basics
R: Chapter 6
H: Chapter 314

Primary lymphoid organs: bone marrow and thymus (FAII p20l) (FAI2 p223) (Phys Fig. 34-1, p435)
Secondary lymphoid organs: lymph nodes, spleen (FAII p200-20l) (FAI2 p222-223)
Mucosa associated lymphoid tissue (MALT) (H p2675)
Lymph node histology (FAII p200) (FAI2 p222) (R pi89)
Lymph drainage (FAII p200) (FAI2 p222) (COA p44, Figl.27)
Innate vs. adaptive immunity (FAII p202) (FAI2 p224) (R pl84)
Tlymphocytes (FAII p344) (FAI2 p375) (R pi86)

<1HM

4. To which lymph nodes do the sigmoid colon and the rectum drain?
(FAII p200) (FAQp222)

5. Where can B cells and T cells be found in the lymph nodes? (FAI I p200) (FAQ p222)

Which MHC are found in T helper cells? Which MHC are found on cytotoxic T cells?
(FAI I p344) (FAQ p226)

Copyright (:•) 2012. Doctors In Training.com, LLC. All Rights Reserved.


Antigen Presentation
R: Chapter 6
MHC I and II (FAII p202) (FA 12 p224) (R pl90)
HLA associated with diseases (FAII p202) (FAI2 p223) (R pl93, table 6-1)
Dendritic cells (FAII p344) (FAI2 p374) (R pl87)
Langerhans cell histiocytosis (FAII p360) (FAI2 p394) (R p63l)

7. Dendritic Cells (FAI I p344) (FAQ p374) (R pi87)


• Covered with long membranous extensions that resemble the dendrites of nerve cells
• All display both class I and II MHC, B7 family of co-stimulatory molecules, CD80 and CD86
• All also have CD40 which can influence T cells byway of interaction with a complementary ligand
• Purpose: antigen capture at one location and presentation at another location. This is accomplished by
migration to lymph nodes for presentation at T cells
• When acting as peripheral sentinels (immature or antigen-capturing state), antigens are captured by 3
different mechanisms:
- Phagocytosis
- Receptor mediated endocytosis
- Pinocytosis
• Immature sentinels make a transition into a new type of cell (antigen-presenting state) that can travel
into circulation and into a cell that has maximal capacity to present to Th cells (increased expression of
class II MHC and co-stimulatory molecules)
• Different types:
- Langerhans (from immature epithelial tissue): in epidermal layers of skin (= Cutaneous-associated
lymphoid tissue, CALT)
- Interstitial (from immature nonepithelial, interstitial tissue): in interstitial spaces of virtually all organs
(except brain)
- Monocyte-derived (from monocytes (which also give rise to macrophages)): migrated from
bloodstream into tissues ■* migrate from tissues to blood and lymph and lymph nodes
- Plasmacytoid derived (from plasmacytoid): APCs in the innate immune response

8. Follicular Dendritic Cells


• Do not arise from bone marrow like the dendritic cells
• Do not express class II MHC ■» do not present antigen to Th cells
• Exclusively reside in lymph follicles
• Important in the maturation and diversification of B cells

WI!ffXw5!TihTIIR™'SI8ilf3rWitW
Langerhans cell histiocytosis? (FAI I p360) (FAQ p394)

What molecules are expressed on the surface of antigen presenting dendritic cells?
(FAII p344) (FAQ p374)

What are 3 cell types that are known for presenting antigens to T cells? (FAI I p204)
(FAQ p226)

Copyright © 2012, Doctors In Training.com, LLC. All Rights Reserved.


arm-Up
r
What hematologic changes would you expect to see in a patient without a function;
spleen? (FAI I p20l) (FAQ p223) (R p633)

Where can you find nicotinic acetylcholine receptors in the body?


(FAI I p235) (FAQ p262) (Phys p733) (GG p255-258)

What structures are derived from the branchial pouches? (FAI I pQ9) (FAQ pl37)

Differentiation of Tcells (FAII p203) (FA p225) (R pl95, Fig. 6-11)


Helper Tcells (FAII p204) (FAI2 p226) (R pi86)
Thymus (FAII p20l) (FAI2 p223) (R p635)
T cell activation (FAII p204) (FAI2 p226) (R pl95)
Cytotoxic Tcells (FAII p205) (FAI2 p227) (R p207)
Regulatory T cells (FAI2 p227)
NK cells (FAII p202) (FAI2 p224) (R pl88)

4. Which cytokines are proauce>


ftl^ioi^vlfJSi.©^ ^ W M ^ S i . c ^ i ' o l W l . t i - r. ^ l V i ' i S l r o r a L l t - c & | l © , \ © i l l ; - ; ' . '

(FAII p204) (FAQ p226)

. Which cytokines inhibit Thi cells? Which inhibit Th2 cells? (FAI I p204) (FAQ p226)

6. How do cytotoxic T cells kill virally-infected and neoplastic cells? (FAI I p205) (FAQ p

Copyright © 2012. Doctors In Training.com. LLC. All Rights Reserved.


Monocytes and Macrophages
R: Chapter 6
Phys: Chapter 33

Monocytes (FAII p343) (FAI2 p373) (Phys p426) (H p2654)


Macrophages (FAII p343) (FAI2 p373) (R pi88)
Interferon-gamma (FA p208) (FAI2 p230) (R pi95)
Acute phase cytokines (FA p208) (FAI2 p230) (R p6l, 193, 200)
Spleen (FAII p20l) (FAI2 p223) (R p632)

7. Monocytes (FAI I p343) (FAQ p373) (Phys p426) (H p2654)


• Leave bone marrow for blood ■» circulate 8 hours to mature ■* migrate into tissues to become
macrophages
• Blood, alveoli, intestines - macrophages
• Connective tissue - histiocytes
• Liver - Kupffer cells
• Kidney - mesangial cells
• Brain - microglia
• Bone - osteoclasts

Which cytokine more than any other should be known as the macrophage activating
cytokine? (FAI I p343) (FAQ p378) (R pi95)

What name is given to monocytes in other tissues of the body? (Phys p426-427)
• Blood, alveoli, intestines
• Connective tissue
• Liver
• Kidney
• Brain
• Bone

What are the acute phase cytokines that are produced by macrophages?
(FAI I p209) (FAQ p231) (R p203) (Phys p426-431)

. What is the function of macrophages in the spleen?


(FAI I p20l) (FAQ p223) (R p633) (Phys p427)

What name is given to RBCs partially digested by splenic macrophages?


(FAI I p348) (FAQ p338) (R p645) '

Copyright © 2012. Doctors In Training.com. LLC. All Rights Reserved.


Question Warm-
iTsl HiimS tvmSi i [•] ikvsI WSM Slv^uil Mil iPwTlT^^r-'l! IW il*] QMHa ■ ^J [*yi L* C*m*j I [afl I

Describe the usefulness of the dexamethasone suppression test.


(FAI I p296) (FAQ p323) (Phys p935)

What enzyme of the adrenal steroid synthesis pathway is inhibited by ketoconazole?


(FAI I p29l) (FAQ P3I8) (Phys p922-933)

B Cells and Antibodies


R: Chapter 6
H: Chapter 112
Phys: Chapter 34
B lymphocytes (FAII p344) (FAI2 p374) (R pl87)
Review of cell surface proteins (FAII p209) (FAI2 p23l) (H p2690)
Plasma cells (FAII p344) (FAI2 p374) (H p936)
Antibody structure and function (FAII p205) (FAI2 p227)
Immunoglobulin isotypes (FAII p206) (FAI2 p228) (H pi010)
Immunization
- Active vs. passive immunity (FAII p209) (FAI2 p232) (Phys p433, 442)
- Viral vaccines (FAII pl64) (FAI2 pl80)
- Antigen type and memory (FAII p206) (FAI2 p228) (Phys p434, 438)
- Vaccines (FAII pl39) (FAI2 pISO)
Autoantibodies (FAII p2l2) (FAI2 p235) (R p208)
Amyloidosis (FAII p224) (FAI2 249) (R p249) (H p945)

4. V(D)J Recombination - Highlights (FAI I p205) (FAQ p227)


• Rearrangements of the DNA segments named variable (V), diversity (D), and joining (J) compose the

coding regions for each specific antigen receptor on B and T cells


• The rearrangement process begins with breaks in the dsDNA at Re.
(RSSs) that flank the V, D, and J coding regions
• V(D)J recombination is initiated by the recombination activating gene complex (RAG I and 2). RAG I
and RAG 2 protein recognizes the RSSs
• Mutations in either of the RAG genes in mice causes an inability to initiate V(D)J rearrangements and an
arrest of B and T cell development

Copyright© 2012. Doctors In Training.com. LLC. All Rights Reserved.


(FAI I p206) (FAQ p228) (R pi87) (Phys p438)
Associated with allergies because it is bound by mast cells and basophils and causes them to
degranulate and release their histamine
Comprises 70-75% of the total immunoglobulin pool
Present in large quantities on the membrane of many B cells
Crosses the placenta and, additionally, confers immunity to neonates in the first few months of life
Can occur as a dimer
Largely confined to the intravascular pool and is the predominant early antibody frequently seen in the
immune response to infectious organisms with complex antigens
Distributed evenly between the intravascular and extravascular pools
The predominant immunoglobulin in mucoserous secretions such as saliva, colostrum, milk,
tracheobronchial secretions, and genitourinary secretions
Can be a pentamer

Which cellular organelles are particularly important to plasma cell function?


(FAI I p344) (FAQ p374) (H p936)

What initiates recombination of VDJ sequences?


(FAI I p205) (FAQ p227) (R pi86) (Phys p437)

Cytokines and Immunosuppressants


GG: Chapters 35, 37, 62
R: Chapter 6
Important cytokines (FAII p208) (FAI2 p23l) (R pl93,200)
Interferon mechanism (FAII p208) (FAI2 p23l) (R p2l6)
Recombinant cytokines and clinical uses (FAII p2l6) (FAI2 p240) (GG pl022)
Transplant rejection (FAII p2IS) (FAI2 p239) (R p226)
Immunosuppressants (FAII p2l5-2l6) (FAI2 p239-240) (GGpl006)
Therapeutic antibodies (FAII p2l7) (FAI2 p24l) (GGpl023)
Infliximab (FAII p338) (FAI2 p369. 432) (GG pl827)

8. Sirolimus MOA: binds FKBP-Q intracellular protein ■» inhibition of mTOR (mammalian


target of rapamycin) ■▶ inhibition of T-cell proliferation

9. Thalidomide
• Uses - immunosuppression (SLE, organ transplant), anti-angiogenic
• Mechanism of Action - affects TNFfJ
• Toxicity - phocomelia (prior use as a sedative during pregnancy)

Copyright © 2012. Doctors In Training.com. LLC. All Rights Reserved.


Quiz
0. What cytokine(s) matches the following statement? (FAI I p208) (FAQ p230)

Produced byThi cells


Produced by Th2 cells
Involved in growth and activation of eosinophils
Secreted by helper T cells and activates macrophages

Inhibits macrophage activation


Pyogenes secreted by monocytes and macrophages
Inhibits production of Thi cells
Inhibits production of Th2 cells
MediateBinflammation
I Promotes cell growth and differentiation

Enhances synthesis of IgE and IgG


Enhances synthesis of IgA
Released by virally infected cells
Supports growth and differentiation of bone marrow stem cells
Supports T cell proliferation, differentiation, and activation

What drugs are composed of antibodies against TNF? (FAI I p393) (FAQ p369) (GG pl82/

Which immunosuppressant matches the following statement?


(FAI I p208-209, 391-392) (FAQ p230-23l, 392-393)
• Precursor of 6-mercaptopurine
• May prevent nephrotoxicity with mannitol diuresis
• Antibody that binds to CD3 on T cells
• Antibody that binds IL-2 receptor on activated T cells
• Inhibits inosine monophosphate (IMP) dehydrogenase

Inhibits calcineurin resulting in the loss of IL-2 production and blockage of T cell differentiation
and activation
Binds FK-binding protein (FKBP) leading to loss of IL-2 production
Binds FKBP 12 leading to inhibition of mTOR and T cell proliferation
Used for lupus nephritis
Metabolized by xanthine oxidase, therefore increasing allopurinol toxicity

Copyright © 20I2. Doctors In Training.com. LLC. All Rights Reserved.


What enzyme is responsible fortRNA charging? What enzyme catalyzes peptide bon.
formation? (FAI I p74) (FAQ p77) (Phys p34)

Which cytokines are secreted by the two different types of helper T cells?
(FAI I p208) (FAQ p226) (R p206, Fig. 6-19)

What substances are utilized by natural killer cells to induce apoptosis in other cells?
(FAI I p202) (FAQ p224, 231) (R pi88)

Complement, Hypersensitivity, and Granulocytes


R: Chapter 6
rd Phys: Chapter 34
>^. H: Chapter 314
'> Complement (FAII p207) (FAI2 p229) (R p63) (Phys p438) (H p2665)
'±J
00Hypersensitivity
- Autoantibodies(FAII
(FAIIp2IO
p2l2)and
(FAI2
FAI2
p235)
p2ll) (R
(R p208,2l2)
pl97) (H p2722)
■ Granulocytes
I - Eosinophils (FAI I p343) (FAI2 p373) (R p72) (Phys p430)
- Mast cells (FAII p343) (FAI2 p373) (R p72)
J - Basophils (FA 11 p343) (FA 12 p3 73) (Phys p423) (Phys p431)

I 4. Paroxysmal Nocturnal Hemoglobinuria (PNH) (FAI I p352) (FAQ p384) (R p652)


• Hemosiderinuria
• Chronic intravascular hemolysis
• Thrombosis
• Dx. Ham's test

5. Erythroblastosis Fetalis (FAI I p353) (FAQ p385)


• Maternal antibodies to fetal RBC antigen
• The most common antibody is Rh-D
• In Rh(-) moms, dose anti-Rh-D immunoglobulin at 28wks, at any traumatic event (MVA), and within
3days of delivery
• Clinical features in the neonate:
- Anemia due to hemolysis of RBC by maternal Ab
- Jaundice ■* possible kernicterus
- Hydrops fetalis (generalized fetal edema)
- IU death

Copyright (£) 2012. Doctors In Traming.com, LLC. All Rights Reserved.


Quiz
6. Which type of hypersensitivity is responsible for the following clinical problem;

• Poststreptococcal glomerulonephritis
k(FAI
- Asthma
I p2l I) (FAQ p233) (R pl97)
Rheumatic fever
Tuberculosis skin test

Allergies, anaphylaxis, and hay fever


Polyarteritis nodosa
Serum sickness
ABO blood type incompatibility

Poison ivy
Eczema
Contact dermatitis
Goodpasture syndrome'

. . patient suffers from recurrent Neisseria infections. What complement proteins are
deficient?
(FAI I p207) (FAQ p229, 236) (R p235)

8. A 45-year-old female complains of malar rash and arthritis. The presence of which
antibodies are specific for SLE? (FAI I p2Q) (FAQ p234-235) (R p2l7, 220-221)

w'hich complement is responsible for neutrophil chemotaxis?


(FAI I p208) (FAQ p230) (Phys p439)

Immunodeficiencies
R: Chapter 6
Immunodeficiencies (FAII p2l3-2l4) (FAI2 p237-237)

Copyright © 2012. Doctors In Training.com. LLC. All Rights Reserved.


Immunodeficiencies

Bruton Agammaglobulinemia X-linked Immunodeficiencies


• X-linked (Boys) • Wiskott-Aldrich
• B cell deficiency ■* defective tyrosine kinase • Bruton Agammaglobulinemia
gene ■» low levels of all immunoglobulins • Chronic Granulomatous Disease (+/-)
• Recurrent Bacterial infections after 6m • Hyper-IgM syndrome (3 types) tlgM, ilg
u - X-linked ■» no CD ligand
cq; - AR"»noCD40
Thymic aplasia (DiGeorge)
"u • 3rd and 4th pouches fail to develop - -NEMO deficiency
^=
<L) 4
TJ - No thymus ■▶ no T cells
O Wiskott
c - No parathyroids ■*• low Ca2+ ■▶ tetany
Aldrich
D • Congenital defects in heart / great vessels
E • Recurrent viral, fungal, protozoal infections Immunodeficiency
E • 90% have a chrom 22ql I deletion (detect Thrombocytopenia and purpura
"D with FISH) Eczema
C
Recurrent pyogenic infections
• No IgM v. capsular polysaccharides of bacteria
Severe Combined Immunodeficiency (SCID)
> • Defect in early stem cell differentiation • Low IgM, high IgA
• Can be caused by at least 7 different gene • X-linked
on
c defects:
- Adenosine deaminase deficiency
Ataxia-Telangiectasia
0) • Last defense is cytotoxic NK cells • IgA deficiency
CL • Presentation triad: • Cerebellar ataxia, and poor smooth pursuit of
>>.
X 1) Severe recurrent infections moving target w/ eyes
-h - Chronic mucocutaneous Candidiasis • Telangiectasias of face > 5yo
c - Fatal or recurrent RSV, VZV, HSV, • t cancer risk lymphoma & acute leukemias
<
u
E measles, flu, parainfluenza • Radiation sensitivity (try to avoid x-rays)
- PCP pneumonia • +/- tAFP in children > 8m
Q. • Average age of death: 25 y/o
2) Chronic diarrhea
E
o 3) Failure to thrive
• No thymic shadow on newborn CXR
U Selective Immunoglobulin Deficiencies /3B&L

• IgA deficiency is most common


Chronic Mucocutaneous Candidiasis - Most appear healthy
>- • T cell dysfunction v. C. albicans - Sinus and lung infections
• Rx: ketoconazole - 1/600 European descent
o
_ l
- Associated with atopy, asthma

o
l"S>i
- Possible anaphylaxis to blood transfusions

z and blood products

D IL-Q receptor deficiency


• Mycobacterial infections

[10] Copyright (§ 2012. Doctors In Traming.com. LLC. All Rights Reserved.


Immunodeficiencies (continued)

Phagocyte Deficiencies
• Chronic granulomatous disease
• Chediak-Higashi syndrome
• Job syndrome
• Leukocyte adhesion deficiency syndrome

Chronic Granulomatous Disease (CGD) =^


• Lack of NADPH oxidase activity ^impotent phagocytes #—
• Susceptible to organisms with catalase (S. aureus, £ coli, Klebsiella spp., Asperg?//us spp., Candida spp.) -^
• Dx: (-) nitroblue tetrazolium (NBT) dye r\
- No yellow to blue-black oxidation I—
• Prophylactic TMP-SMX O
• INPV also helpful O

Chediak-Higashi Syndrome 4*-


• Defective LY S T gene (lysosomal transport) I
• Defective phagocyte lysosome ■» giant cytoplasmic granules in PMNs are diagnostic (~)
• Presentation triad: O

1) Partial albinism _^
2) Recurrent respiratory tract and skin infections >*"
3) Neurologic disorders 3
n>
Job Syndrome 3-
• Hyperimmunoglobulin E syndrome -r
• D e f i d e n t I N F v ■» P M N s f a i l t o r e s p o n d t o c h e m o t a c t i c s t i m u l i ( C 5 a , LT B 4 ) * <
• High levels of IgE and Eosinophils fD
• Presentation triad: ^

1) Eczema 5
2) Recurrent cold Staph, aureus abscesses (think of biblical Job with boils) !£.
3) Course facial features: broad nose, prominent forehead ("frontal bossing"), deep set eyes, and <"
"dough/' skin
• Also common to have retained primary teeth resulting in 2 rows of teeth
D
Leukocyte Adhesion Deficiency Syndrome
• Abnormal integrins ■» inability of phagocytes to exit circulation
• Delayed separation of umbilicus
C
D
O
Q_
fl>
=±>
Q
fl>"

Copyright ©2012. Doctors In Training.com. LLC. All Rights Reserved. [II]


What would you expect to see in a patient with Wiskott-Aldrich syndrome;
(FAII P2I4) (FAQP238)(Rp235)

What is the cause of chronic granulomatous disease? What infections are these
individuals susceptible to? (FAI I p2l4) (FAQ p238) (R p234)

»young child presents with tetany from hypocalcemia and candidiasis resulting from
immunosuppression. What cell type is deficient in this patient?
(FAI I P2I4) (FAQ p238) (R p234)

A young child has recurrent lung infections and granulomatous lesions. What is the defect
in neutrophils? (FAII P2I4) (FAQ p238) (Rp23l-233)

,, mother brings in her 2-year-old child who has had multiple viral and fungal infections
and is found to be hypocalcemic. Which of the 3 types of germ cells (ecto-, endo-, and
mesoderm) gives rise to the missing structure in this child?
(FAII p89, 213) (FAQ pl37, 237)

A child has an immune disorder in which there is a repeated Staph abscesses. It is found
that the neutrophils fail to respond to chemotactic stimuli. What is the most likely
diagnosis? (FAI I P2I4) (FAQ p237-238)

Copyright c 2012. Doctors In Training.com. LLC All Rights Reserved.


7. In intermediate starvation (48 hours after the last meal), how does the pattern of fuel
production and consumption change?
Produced -

• Used -
- Brain uses predominantly but also some
- Muscles and other tissue use predominantly but also some

8. What metabolic scenario favors the synthesis of ketone bodies?

9. True or false? Ketone bodies can be used by all body tissues including the brain.

10. What is the pattern of fuel utilization and production in prolonged starvation
(5 days after last meal)?
• Produced -

• Used-
- Brain uses predominantly
- Muscles and other tissue use predominantly
but also some

Comparing an overnight fast to a 3 day fast, what percentage of energy comes from
glucose and from ketone bodies?
• Overnight - % from glucose (2/3 from glycogen breakdown, 1/3 from gluconeogenesis)
% from ketone bodies

3day- % from ketone bodies (Vi are 8-hydroxybutyrate, J4 acetoacetate)


_% from glucose (most from gluconeogenesis)

. What is the primary energy source in a patient that has not eaten in two days?
(FAII pi 13) (FAI2pi 19)

. What hormone stimulates the storage of lipids in the fed state? (FAI I pi 13) (FA12 pi 19)

. What is the rate limiting enzyme in ketone body synthesis? (FAI I pi 12) (FAI2 pi 18)

5. Which ketone body is metabolized by muscle and brain tissue? (FAI I pi 12) (FA 12 pi 18)

Copyright (O 2012. Doctors In Training.com, LLC. All Rights Reserved.


Ethanol and Caloric Deficiencies
H: Chapter 392
R: Chapter 9
Ethanol metabolism (FAII p94) (FAI2 plOO) (R p4l2)
Ethanol hypoglycemia (FAII p94) (FAI2 pi00)
Malnutrition (kwashiorkor and marasmus) (FAII p94) (FAI2 plOO) (R p428)

lb 'i*liM*JnTA
A stressed physician comes home from work, consumes 7 or 8 shots of tequila in
rapid succession before dinner, and becomes hypoglycemic. Why did she become
hypoglycemic? (FAI I p94) (FAQ pi00)

What enzymes are used to metabolize alcohol? (FAI I p94) (FA12 pi00)

18. What are some of the hallmark features of kwashiorkor? (FAI I p94) (FAI2 plOO)

How does the brain utilize ketone bodies? (FAI I pi 12) (FAI2p 119)

Copyright© 2012. Doctors In Training.com, LLC. All Rights Reserved.


What are the 3 most com id cancer? (FAI I p298) (FAI2 p326)

What are the side effects of oriistat?

What are the arterial branches off of the celiac trunk? (FAI I p3l2) (FAI2 p340)

Lipid Transport
Phys: Chapter 68
H: Chapter 356

Lipid transport, key enzymes (FAII pi 14) (FA12 pi20) (H p3l45)


Major apolipoproteins (FAII pi 14) (FAI2 pl20) (Phys p82l) (H p3l45)
Lipoprotein functions (FAII pi 15) (FAI2 pl2l) (H p3l45)
Familial dyslipidemias (FAII pi IS) (FAI2 pl2l) (R pl47) (H p2l48)
Abetalipoproteinemia (FAII pi 15) (FAI2 pl2l) (H p3l53)
Hyperlipidemia signs (FAII p269) (FAI2 p294)

Copyright © 2012, Doctors In Trainmg.com. LLC. All Rights Reserved.


4. Lipid transport

Dietary Fat Exogenous Endog

Intestine

Into lymph,
through thoracic duct,
then into blood

Dietary Fat Exogenous

cholesterol,
bile acids

Intestine cholesterol endocytosls

B <v

>
chylomicrons
T triglycerides and
Into lymph. chylomicron olesterol phospholipids
through thoracic duct, remnants hepatic triglyceride
then into blood V CEPT f lipase (HGTL)

lipoprotein lipase j
lipoprotein lipase

Copyright © 2012. Doctors In Training.com. LLC. All Rights Reserved.


What deficiency causes IslHI BI!m!ra«OTSls5!aH!15

Which apolipoprotein matches the following statement? (FAI I pi 14) (FAI2 pi20)
• Activates LCAT
• Mediates chylomicron secretion
• Mediates VLDL secretion
• Binds to LDL receptor
• Cofactor for lipoprotein lipase
• Mediates uptake of remnant particles

When looking at a slide of RBCs you notice RBCs that look spiny like a medieval mace.
What is likely to be deficient in this patient? (FAI I pi 15) (FAI2 pl2l)

Fatty Acids and Cholesterol


Phys: Chapter 68
Fatty acid synthesis (FAII pi 12) (FAI2 pi 18, 119) (Phys p824)
Fatty acid degradation (FAII pi 12) (FAI2 pi 17)
Omega-3 and omega-6 fatty acids (H p3l60)
Cholesterol synthesis (FAII pi 13) (FAI2 pi 19) (Phys p826)

What is the rate-limiting enzyme for the following metabolic pathway?


• Hexose monophosphate pathway
• Fatty acid synthesis
• 3-oxidation of fatty acids
• Ketone body synthesis
• Cholesterol synthesis

9. Which group of medications inhibits the rate limiting enzyme of cholesterol synthesis?
(FAII pll3)(FAI2plOI)

Where in the cell would you find the following enzymatic processes taking place?
(FAII p95) (FAQ pi00)
• Fatty acid degradation
• Fatty acid synthesis
• Glycolysis
• TCA cycle
l chain (oxidative phosphorylation)

Copyright g! 2012. Doctors In Training.com. LLC All Rights Reserved.


e s t i o n Wa r m -
A 2-year-old girl has an increase in abdominal girth, failure to thrive, and skin/hair
depigmentation. What is her diagnosis? (FAI I p94) (FAI2 plOO)

What enzyme is deficient in the following diseases? (FAI I pl03-l04) (FAI2 pi09)
• Fructose intolerance
• Essential fructosuria
• Classic galactosemia

What structures run through the cavernous sinus? (FAI I p4l8) (FA12 p458)

Amino Acid Basics


Phys: Chapter 69
Amino acids (FAII pl04) (FAI2 pi 10) (Phys p83l)
- Essential amino acids (Phys p834)
- Ketogenic vs. glucogenic
- Amino acid claims to fame
- Acidic vs. basic
Amino acid derivatives (FAII pl06) (FAI2 pllO)
Catecholamine synthesis (FAII pl07) (FAI2 pi 12)

What are the essential ids? (FAI I pl04)(FAI2pM0)

What amino acid is a precursor to the following molecule? (FAI I pl06) (FAQ pi 11)
• Histamine
• Porphyrin, heme
• NO
• GABA (a neurotransmitter)
• S-adenosyl-methionine (SAM)
• Creatine

6. Which amino acids have the following structure? What is the following compoi


t C COOH H,N—C
r COOH
1 3 I

/ \
/ C H

Copyright © 2012. Doctors In Training.com. LLC. All Rights Reserved.


Nitrogen Metabolism
Phys: Chapter 69
Urea cycle (FAII pl05) (FAI2 pi 10) (Phys p835)
Ornithine transcarbamoylase deficiency (FAII pi05) (FAI2 pill)
Hyperammonemia (FAII pl05) (FAI2 pill) (H p2527)
Lactulose (GG pi332)

Sess
Compare carbamoyl phosphate synthetase I to carbamoyl phosphate synthetase II
pl05)(FAI2plll)

What is the mechanism of action of lactulose? (FAI I pl05) (FAI2 pi 11)

What is the most common urea cycle disorder? (FAI I pi 05) (FAI2p III)

Copyright © 2012. Doctors In Traming.com, LLC. All Rights Reserved.


(FAI2pll2)

I. What are the byproducts of MAO and COMT enzymatic activity on dopamine,
norepinephrine, and epinephrine? (FAI I pi07) (FA12 pi 12)

, What substrate and cofactor is required for the generation of GABA?


(FAII pl06)(FAI2pll2)

Amino Acid Disorders


H: Chapter 364

PKUfFAII pl07)(FAI2pll2)
Alkaptonuria (FAII pl08) (FAI2 pi 12)
Albinism (FAII pl08) (FAI2 pi 12)
Homocystinuria (FAII pl08) (FAI2 pi 13)
S-adenosyl-methionine (FAII p92) (FAI2 p98)
Cystinuria (FAII pl08) (FAI2 pll3)
Maple syrup urine disease (FAII pl08) (FAI2 pll3)
Hartnup disease (FAII pl08) (FAI2 pll3)

Quick Quiz
JSrSfcREliiSliwiraSTeif
What is the diagnosis?

A patient with PKU should have diet low in phenylalanine. What other dietary
modifications should a patient with PKU make?

A middle-aged man has dark spots on his sclera and has noted that his urine turns bk
when left sitting for a period of time. What is the diagnosis?

What is the underlying cause of maple syrup urine disease? (FAI I pl08) (FAI2 pi 13)

Copyright <£) 2012, Doctors In Tr.iining.coin. LLC. All Rights Reserved.


E l f ,

What are the differences between c L'MllU'J» etase (CPS) I and CPS II?
(FAI2plOO-IOI)
CPSI CPSII
Location
Pathway
Nitrogen source

A 45-year-old male alcoholic gets blistering lesions in sun-exposed areas especially the
dorsum of the hands. He also has hypertrichosis of the face. What is the diagnosis?
(FAI2 p386)

What is the rate-limiting enzyme of heme synthesis? (FAI I p354) (FA 12 p386)

Calcium, Iron, and Zinc


R: Chapter 9
H: Chapter 357
Calcium (FAII p293) (FAI2 p320)
Iron
- Iron in the human body
- Iron deficiency (FAII p349) (FAI2 p380) (R p659)
- Iron poisoning (FAII p224) (FAI2 p249)
- Hemochromatosis (FAII p334) (FAI2 p364) (R p86l)
Zinc (FAII p94) (FAI2 p99) (Phys p856) (R p439, Table 9-10)

4. Iron Proteins (FAI I p353) (FA12 p385)


Ferritin Transferrin
• Iron-protein complex (Ferric acid and • Protein that binds ferric molecules and transports
apoferritin) them through plasma. Synthesized in the liver
• Cellular storage protein for iron • t Vi = 8 days
• Acute phase reactant • Increased in iron deficiency

Copyright © 2012, Doctors In Training.com, LLC. All Rights Reserved.


uick Quiz

is iron bound in blood?

What has been the most common clinical scenario of excess iron ingestion in the last 3
decades? (FAI I p224) (FAI2 p249)

7. What are some of the clinical effects of zinc deficiency? (FAI I p94) (FAI2 p99)

8. What are some of the signs of hypocalcemia? (FAI I p464) (FAQ p464)

Trace Minerals and Toxic Metals


R: Chapter 9

Overview of elements
Trace minerals
Copper
Toxic metals
- Lead poisoning (FAII p350) (R p406)
- Mercury poisoning (R p407)
- Arsenic (R p408)
- Cadmium (R p408)

Copyright v':: 2012. Doctors In Training.com, LLC. All Rights Reserved.


9. Overview of Elements

H He
MnMn rrfroQMt
3 4 s 7 10
Li Be B C N O Ne
90l» Mil!.
11 12 11 IS 16
Na Mg Al Si p s CI Ar
fcrypon
19 w It 22 22 2S 26 » 29 Jl J2 3] 28
K Ca Sc Ti V Cr Mn Fe Co Ni Cu Zn Ga Ge A s Se Br Kr
m c i ■55
stm
roMxui tacmotn
_&ZL -wwi. eaauM
mn
Mkn
- O U -
liuur ffW-
M M 40 44 45 47 41 49 St S4

Rb Sr Y Zr Nb Mo Tc Ru Rh Pd A g Cd In Sn Sb Te Xe
_9iHi.
tanttUi
my urw
ss 56 S7-70 71 73 7$ 75 77 79 79 •I 92 « 14
Cs Ba * Lu Hf Ta W Re OS lr Pt A u Hg TI Pb Bi Po A t Rn
J.HV
tiaaet ttwoncun bcMun
,'wn -JgU2_
TumrT iiHMkjn
urn
87 89.102 10] 104 105 100 107 1M 109 m 112 11 4
Fr Ra * * Lr Rf Db s g Bh Hs Mt Uun Uuu Uubj Uuq
_no_ W1

CCOA -tkt"v ?CC=4CM» MTOUH •uceen easaan tOMCT «m«CMn — r o n u i - i—5555T-I


57 St 69 C
O 61 62 6] 64 65 6fi 67 69 69 70
Lanthanide series
La Ce Pr Nd Pm Sm Eu Gd Tb Dy Ho Er Tm Yb
«•« won »'" 144 M
(reUttnw uanwv
l » »
IH9
occuim cMonn
1MM 1W» ,M«1 (SM
•Actinlde series 89 90 91 92 9) 94 9S 96 97 99 99 100 101 102
Aranc Th
»»« Pa
aiiM
u
mn
Np
inn
Pu A m Cm B k Cf Es Fm Md No
B«fl no run m'l ma OCT ma
13411 (W*

10. Trace Minerals


Mineral Source Function Deficiency Excess
Lithium
Beryllium
Boron
Chromium
Manganese
Cobalt /&i$y

Nickel
Copper
en

I. Mercury Poisoning
c • Accumulates in the kidney and brain
• Acrodynia - peeling of the fingertips
• Abdominal pain
• Common sources: shark, swordfish, old thermometers, batteries

X
u
o
CD

[32] Copyright © 20I2. Doctors In Training.com. LLC. All Rights Reserved.


hypersensitivity? (FAI I p2IO) (FAI2 p233)

What are some of the risk factors for esophageal cancer? (FAI I p323) (FAI2 p35l)

What enzyme is deficient in Lesch-Nyhan syndrome? What is the treatment?


(FAII p69) (FAI2 p70)

Vitamins D, K, and A
R: Chapter 9
Phys: Chapter 71
Vitamin D (FAII p93) (FAI2 p99) (R p433) (Phys p855)
Vitamin K(FAII p94) (FAI2 p99) (R p438, Table 9-9) (Phys p855)
Vitamin A (FAII p90) (FA 12 p95) (R p431) (Phys p853)
o
Vitamin D (FAI I p93) (FA12 p99)
Tj 4, How does the biological form of vitamin D exert its actions?
It interacts directly with target cell DNA to selectively stimulate or repress gene expression.

True or false? Vitamin D is obtained by the body from both sun exposure and diet.
True, up to 8096 of required vitamin D can be derived from sunlight (at least 15 minutes daily), and the
remaining amount must be ingested.
en
.£ 6. How does vitamin D help maintain adequate plasma levels of calcium?
• Increases calcium uptake in the intestine (via increased expression of calcium binding protein)
• Stimulates PTH-dependent reabsorption of calcium in the distal tubules
• Stimulates bone resorption when necessary (along with PTH)

_Q
D 7 . What are the steps in the metabolism of vitamin D?
• Gut absorption (D2) or skin synthesis (D3)
• Binding to plasma a I-globulin (D-binding protein) and transport to liver
• Conversion to 25-hydroxyvitamin D (AKA 25-hydroxycholecalciferol) by 25-hydroxylase in liver
• Conversion of 25-hydroxycholecalciferol to 1,25-dihydroxycholecalciferol by a I-hydroxylase in the kidney

What is the precursor of vitamin D3 in the skin?


7-dehydrocholesterol

What other names have been given to D2, D3, and 1,25-dihydroxycholecalciferol?
• D2 - ergocalciferol (ingested form derived from plants)
• D3 - cholecalciferol (formed in sun-exposed skin)
• l,25(OH)2D - calcitriol (the active form of vitamin D)

Copyright © 2012, Doctors In Training.com, LLC. All Rights Reserved.


10. When prescribing vitamin D replacement, which form is preferred?
Over the counter D3 (cholecalciferol)

11. What are the names for vitamin D deficiency in adults and in children?
Adults - Children -

12. Explain how vitamin D deficiency brings about the clinical symptoms seen in
osteomalacia and rickets?
• Lack of vitamin D ■▶ hypocalcemia ■» increased levels of PTH ■»
- Mobilization of caldum from bone
- Decreased renal calcium excretion
- Increased renal excretion of phosphate ■» hypophosphatemia ■▶ impairment of bone mineralization

13. What is the basic derangement in both rickets and osteomalacia?


Excess in unmineralized bone matrix

14. What are some of the common clinical manifestations of rickets?


• Bow-legged
• Lumbar lordosis
• Pectus carinatum (pigeon chest) - protrusion of the sternum and ribs
• Rachitic rosary- overgrowth of cartilage or osteoid tissue at the costochondral junction
CO
15. What problem is encountered with a vitamin D intake 10-100 times the US RDA? Q
Hypercalcemia O
I
16. In what disease does vitamin D toxicity result from excess macrophage generation of
25-hydroxy-vitamin D?
Sarcoidosis

i
Vitamin K (FAI I p94) (FAI2 p99)
17. What is the principle role of vitamin K? &i'
m o d i fi c a t i o n o f v a r i o u s w h e r e i t s e r v e s a s a '
coenzyme in the carboxylation of certain glutamic add residues present in these proteins Sy

18. For what protein synthesis is vitamin K dependent upon? q-


Proteins C & S, prothrombin, and clotting factors II, VII, IX, and X (2,7,9, and 10) flj"

1 9 . W h a t c h a r a c t e r i z e s v i t a m i n K d e fi c i e n c y i n a d u l t s a n d c h i l d r e n ? H - "
Hemorrhagic disease |jj

20. Why are newborns particularly prone to vitamin K deficiency? What prophylactic ^
measures can be taken to reduce the incidence of this problem? ^
• Newborns do not have microbes in the gut to produce vitamin K, and the mother's milk only provides ^
l/5th of their need. Q-
• It is recommended that newborns receive a single IM dose (Img) of vitamin K at birth. ^
ZJ
21. What pharmacologic agents can cause vitamin K deficiency with long-term use? q"
• Coumadin X.
• Anticonvulsants CL
• Antibiotics (■» loss of gut bacteria) £j

22. What characterizes vitamin K toxicity?


Hemolytic anemia and jaundice in an infant
Copyright © 2012. Doctors In Training.com. LLC. All Rights Reserved. [35]
Vitamin A (FAI I p90) (FAQ p95)
23. What are the different forms of vitamin A?
• Retinol, retinal (both used by the body)
• P-carotene (deaved in intestine to yield 2 molecules of retinal)
• Retinoic add (cannot be reduced, unusable by the body)

24. Retinol esters are stored on the liver until needed by the body. How are they
transported in the body when needed?
Via plasma retinol-binding protein (RBP)

25. In what broad metabolic functions is vitamin A necessary?


• Vision
• Growth
• Reproduction (cf spermatogenesis, 9 prevent fetal resorption)
• Maintenance of epithelial cells (especially mucous secreting cells)

26. For what disorder® is topical retinoic acid useful? ^


en Acne and psoriasis """)

X> 27. What retinoid derivative can betaken orally for the treatment of acne? ^
X Isotretinoin (Accutane) ^

C 28. What are the signs of vitamin A deficiency?


< • Night blindness
C • Xerophthalmia (pathologic dryness of the conjunctiva and cornea) ■▶ corneal ulceration and blindness
rO
en .(wrinkling, douding of cornea)
C • ( d r y, s i l v e r - g r a y p l a q u e s o n t h e b u l b a r c o n j u n c t i v a )
£ /^s

^ 29. What are the signs of hypervitaminosis A (vitamin A toxicity)?


^> • Headache, nausea/vomiting, stupor
^ • Skin - dry and pruritic ■
_Q • Liver-enlarged (and possibly cirrhotic) >

O • Bone and joint


'**> pain
**Q • Increase in intracranial
^ pressure
ro ^^
30. True or false? Lack of vitamin A in pregnant women has the potential for causing ^
i
congenital malformations.
False, excessive vitamin A in pregnant women has the potential for causing congenital malformations
y- including: ""*,
LU
X 31. In which patient populations is vitamin A supplementation a particularly bad idea?
U • Pregnancy ■▶ increased risk of teratogenic damage
O • Smokers ■▶ increased risk of lung cancer
00

[36] Copyright © 2012. Doctors In Training.com, LLC. All Rights Reserved.


What vitamin in excess can cause hypercalcemia?

What are the symptoms of vitamin A toxicity? (FAI I p90) (FAQ p95)

In which common chronic diseases is vitamin D supplementation particularly essential?

Vitamins C, E, and Other Antioxidants


R: Chapter 9
Phys: Chapter 71
Vitamin C (FAII p93) (FAI2 p98) (R p437) (Phys p855)
Vitamin E (FAII p93) (FAI2 p99) (R p438. Table 9-9) (Phys p8S5)
Other antioxidants

Vitamin C (FAI I p93) (FA12 p98)


36. What is the main metabolic reaction that vitamin C is involved in? 7^
Hydroxylation of and residues of collagen f

3 7 . W h a t a r e t h e m a j o r s i g n s o f v i t a m i n C d e fi c i e n c y ( s c u r v y ) ? r n
• Sore, spongy gums ~^
• Loose teeth
• Fragile blood vessels ■» hemorrhages |vj
• Swollen joints (bleeding into joint spaces) i
• Impaired wound healing
• Anemia j

CO
Vitamin E (FAI I p93) (FA12 p99) i
38. What is the primary function of vitamin E? j
Antioxidant - prevention of the nonenzymatic oxidation of cell components (especially on red blood [ i
cells) by molecular oxygen free radicals

39. What is another name for vitamin E?

40. What is associated with vitamin E deficiency?


• Spinocerebellar degeneration ■» ataxia
• Peripheral neuropathy and proximal muscle weakness .-

41. Mutations in what gene results in the autosomal recessive hereditary vitamin E deficiency? >>
Alpha-tocopherol transfer gene protein !
i

42. True or false? Vitamin E supplementation can help prevent the development of
Alzheimer disease?
False, studies have not yet shown that vitamin E supplementation can reduce the risk of Alzheimer ,
disease; however, studies have shown that vitamin E supplementation at doses of < 1,100 IU daily do ~r'
increase one's risk of "all-cause" mortality. (Ann Int Med 2005; 142:37-46).

Copyright^-; 2012. Doctors In Training.com. LLC. All Rights Reserved. L


[38] Copyright © 2012. Doctors In Traming.com. LLC. All Rights Reserved.
tion Warm-Up

In what disease would you see the following antibodies? (FAI I p2l2) (FAI2 p235)
• Anti-mitochondrial
• Anti-TSH receptor
• Anti-centromere
• Anti-basement membrane
• Anti-neutrophil

What is the most common salivary gland tumor? What is the second most common
salivary gland tumor? What is the most common location for a salivary gland tumor?
(FAII p32l)(FAI2p350)

Folic Acid, Vitamin BI2, and Vitamin B6


Phys: Chapter 71
Water soluble vitamins
Folic acid (FAII p92) (FAI2 p97) (Phys p854)
Vitamin 6/2 (cobalamine) (FAII p92) (FAI2 p97) (Phys p854)
Vitamin B6 (pyridoxine) (FAII p9l) (FAI2 p96) (Phys p854)

Folate (B9) and BI2 (FAII p92) (FAQ p97)


4. In what metabolic reaction(s) is folic acid (B9) involved?
Synthesis of purines (A, G) and thymine (T)

5. What is the biologically active form of folic acid?

6. What is the intracellular storage form of folic acid (folate)?


N-methyl folate

7. What are the characteristics of folic acid deficiency?

Growth failure
Megaloblastic anemia

8. What is megaloblastic anemia?


A type of macrocytic (large RBC) anemia characterized by an elevated number of megaloblasts in the
marrow

9. Deficiency in which two vitamins may cause megaloblastic anemia?


Folate and BI2
Copyright © 2012. Doctors In Traimng.com, LLC. All Rights Reserved.
10. How can you determine if a megaloblastic anemia is caused by folate or BI2 deficiency?
• BI2 deficiency- decreased serum BI2 "^
• Folate defidency - decreased serum folate (both may have decreased RBC-folate) "^
( n o t e t h a t m e g a l o b l a s t i c a n e m i a c a n b e c a u s e d b y a d e fi c i e n c y i n b o t h a t t h e s a m e t i m e ) ^

11. How much folate should a sexually-active woman of childbearing age take in order to ^
prevent neural tube defects? ^
0.4 mg of folic add a day (4 grams if at high-risk for neural tube defects)

12. What results from an excess of folate? Why? """i


BI2 deficiency results because this vitamin is used in making tetrahydrofolate. /*^

13. True or false? BI2 deficiency can cause a deficiency in folate.


/^Jl\
True

14. What percentage of people in the US is estimated to have low serum folate levels? ~
15-20%

15. What is found in the center of the corrin ring of cobalamin (vitamin BI2)? ^
Cobalt e^

16. In what metabolic reactions is vitamin BI2 involved? '


• Homocysteine & methyl-THF ^ methionine & THF (required for SAM to function (FAI I p92) (FAI2 p98) ^
• Methylmalonyl CoA (coenzyme A) ■» succinyl CoA ^

17. What is seen in BI2 deficiency (most often due to a failure to absorb this vitamin /%».
rather than its absence from the diet)?
<2 Pernicious anemia: >
• — - Megaloblastic anemia ^
t - CNS symptoms (myelin degeneration in the dorsal and lateral tracts of the spinal cord) ■▶ sensory .m,
.p problems (i.e. pins-and-needles) '
^> - Atrophy of stomach fundic glands (■▶ achlorhydria) and replacement of the gastric epithelium with ^
<U mucus-secreting goblet cells that resemble those lining the large intestine (intestjnalization) „-.
~c\ - Homocystinuria and methylmalonic acid in urine
D yfHBk

O 18. What is another name for B12 neuropathy? **)


Subacute combined degeneration

,rt 19. What is usually the cause of malabsorption of B12?


p> Autoimmune destruction of gastric cells responsible for the synthesis of intrinsic factor (which is
' necessary for the absorption of B12) ***)
PO /*%
20. Where is BI2 absorbed in the Gl tract?
2L Distal ileum
111 ^
-X 21. Name two malabsorption problems of the distal ileum that cause BI2 deficiency. ^
y Crohn's disease and celiac sprue ^

^ 22. What is the treatment for B12 d e fi c i e n c y ? ^


IM injection of cyanocobalamin ^

[40] Copyright ©2012, Doctors In Training.com. LLC. All Rights Reserved.


Vitamin B6
23. What is the biologically active form of B6 (pyridoxine)?
Pyridoxal phosphate

24. What is the metabolic function of pyridoxal phosphate?


• Coenzyme for numerous enzymes including those of metabolism (transaminations
and deaminations)

25. What drug can lead to a deficiency in B6 as well as B3?


(used for tuberculosis)

26. What are the clinical findings of B6 deficiency?


Same as riboflavin deficiency + hyperirritability, and peripheral neuropathy

Quiz
7. A patient presents with convulsions and irritability, lat vitamin deficiency is causing
these symptoms in this patient?

I. What type of anemia can be caused by folate or BI2 deficiency? (FAI I p92) (FAQ p9/

Where is BI2 absorbed into the circulation? (FAI I p92) (FAQ p97)

Vitamins B5, Niacin, Riboflavin, Thiamine, and Biotin


Phys: Chapter 71
Vitamin B5 (pantothenate) (FAII p9l) (FAI2 p95) (Phys p855)
Vitamin B3 (niacin) (FAII p9l) (FAI2 p96) (Phys p8S3)
Vitamin B2 (riboflavin) (FAII p9l) (FAI2 p95) (Phys p854)
Vitamin BI (thiamine) (FAII p9l) (FAI2 p95) (Phys p853)
Biotin (FAII p93) (FAI2 p98)

30. What vitamins have the following names:


Thiamine Bi
Retinol, retinal A
Pantothenic acid Bs
Pyridoxine B«
a-tocopherol E
Folic acid B*
Niacin B3
Riboflavin B.
Ascorbic acid C hint The Rich Never Pan Pyrite Filled Creeks.
Cobalamin B» B. Bi B3 Bs B* B* Be

Copyright is) 2012. Doctors In Training.com. LLC All Rights Reserved


Vitamin BI (thiamine)
31. What is the functionally active form of thiamine (BI)?

32. In what reactions does thiamine pyrophosphate have a role? ^


• Pyruvate ■▶ acetyl CoA Pyruvate dehydrogenase ^
• a-Ketoglutarate ■▶ succinyl CoA (TCA) a-ketoglutarate dehydrogenase
• R i b o s e 5 - P < - -▶ g l y c e r a l d e h y d e 3 - P ( H M P ) T r a n s k e t o l a s e ^

33. How is thiamine deficiency diagnosed?


By an increase in erythrocyte transketolase activity observed upon addition of thiamine

3 4 . W h a t t w o s y n d r o m e s a r e a s s o c i a t e d w i t h t h i a m i n e ( B I ) d e fi c i e n c y ? I n w h a t ^
populations are these usually seen? ~
• Beriberi (dry and wet) Where polished rice is the major component of the diet
• Wernicke-Korsakoff syndrome Chronic alcoholism (hint BerI BerI) "^

35. What characterizes dry beriberi?


• Nonspecific peripheral with myelin degeneration
• To e - d r o p , wrist-drop, and foot-drop "^
• Musde weakness, hyporeflexia, areflexia ^
S%fi\

36. What characterizes wet beriberi?


• P e r i p h e r a l v a s o d i l a t i o n ■▶ h i g h - o u t p u t ■▶ p e r i p h e r a l e d e m a
• Cardiomegaly

37 What are the clinical characteristics of Wernicke-Korsakoff syndrome?


* / fi n i s
• Ocular disturbances and nystagmus '
CO
• Gait ataxia ^
C • M e n t a l d y s f u n c t i o n ( c o n f u s i o n , a p a t h y, l i s t l e s s n e s s , a n d d i s o r i e n t a t i o n ) ^
C Korsakoff psychosis- retrograde recall, inability to acquire new information, and /"^\
rd
> 38. What exactly is confabulation? ^
-Si The invention of fictitious detail about supposed past events (often to disguise an inability to remember "^
-3 past events) /^

oo Vitamins B2, B3, B5, and B6 1


(j) 39. What are the two biologically active forms of riboflavin (B2)? What is the role of these
molecules? ^
Jx)
/Wit»j

i
no Both are cofactors for oxidation-reduction reactions.

_ 4 0 . W h a t a r e s o m e o f t h e s y m p t o m s a s s o c i a t e d w i t h r i b o fl a v i n d e fi c i e n c y ? ^
ijj • Dermatitis ^
~f~ • Cheilosis/angular stomatitis
(^J • Glossitis (smooth, purple tongue) *
o 41. What is cheilosis? What is angular stomatitis?
CO • Cheilosis - fissuring of the corners of the mouth
• Angular stomatitis - inflammation of the comers of the mouth
[42] Copyright © 2012. Doctors In Training.com, LLC. All Rights Reserved.
42. In what patient population is angular stomatitis particularly common?
Denture wearers

43. What nutrient deficiencies are associated with cheilosis, glossitis, and stomatitis?
Iron, riboflavin, niacin, folate, and BI2

44. What are the biologically active forms of niacin (B3)? What is the role of these
molecules?
• Nicotinamide adenine dinucleotide (NAD+)
• Nicotinamide adenine dinucleotide phosphate (NADP+)
Both are cofactors for oxidation-reduction reactions.

45. Which amino acid required for the generation of niacin?

46. What disease is caused by niacin deficiency? What are the symptoms of this disease?

hint niadn (B3); NADH yields 3 ATP; 3 Ds

47. Why might pellagra be seen in a population that eats primarily corn?
can be metabolized to form niacin, and corn is lacking in tryptophan.

48. How is niacin (at a dose of lOOx the RDA) affective in treating type lib
hyperlipoproteinemia?
Inhibits lipolysis in adipose tissue ■▶ less circulating free fatty acids ■▶ less fatty acids to liver ■*•
less VLDL made ■▶ and less LDL produced rjrj

49. What is the main side effect of high dose niacin treatment of dyslipidemia? How can it O
o
be prevented? I
The peripheral vasodilation (flushing) of high-dose niadn can be lessened by taking aspirin with niadn. m

50. What is the role of pantothenic acid (B5) in metabolism? uj


It is a component of coenzyme A (CoA) which functions in the transfer of acyl groups.

51. True or false? There is no deficiency disease for pantothenic acid


£
(B5). r-
True ~5
CO
O
Biotin (FAI I p93) (FAI2 p98) c"
52. What is the metabolic ^ role of biotin?
(D
Apoenzyme in carboxylation reactions (hint buy-a-tin of C02) ^
7+
53. What can cause a d e fi c i e n c y in biotin? ^
• The glycoprotein avidin which is found in egg whites prevents absorption of biotin (with a normal diet 3
Ui
20 egg whites per day are required to induce a deficiency).
• Antibiotic use (gut bacteria make biotin for us)

Copyright® 2012. Doctors In Traimng.com. LLC. All Rights Reserved. [43]


MfflSTSSBRr
1 Increased RBC fragility
' Dermatitis, cheilosis, glossitis
• Peripheral neuropathy, glossitis
• Hemorrhagic disease
Neural tube defects
Dermatitis, diarrhea, dementia
Megaloblastic anemia
Pernicious anemia
Bitot spots, keratomalacia, xerophthalmia
Osteomalacia
• Rickets

Which vitamin matches the following statement?


• Can be used to treat acne and psoriasis
• Used in oxidation/reduction reactions
• Used in carboxylation reactions
• Involved in the hydroxylation of prolyl residues
• Requires intrinsic factor for absorption
• Deficiency may result from kidney disease
•. Used by pyruvate dehydrogenase and a-ketoglutarate dehydrogenase
• Given prophylactically to newborns
• Can be used to elevate HDL and lower LDL
• Deficiency can be caused by isoniazid use
• Cobalt is found within this vitamin
• Critical for DNA synthesis

Copyright © 2012. Doctors In Training.com. LLC. All Rights Reserved.


u est ion Warm-Up
Which lysosomal storage disease matches the following statement? (FAI I pill) (FAQ pi I.
Accumulation of GM2 ganglioside
Associated with renal failure
Accumulation of dermatan sulfate
Deficiency in hexosaminidase

Which antivirus) matches the following statement? (FAI I p 195-196) (FAQ p2l6-217)
Inhibits CMV DNA polymerase
Used in treatment for chronic hepatitis C
Blocks viral penetration and uncoating
Treats both influenza A and B
Second-line for CMV retinitis

What is the function of MacConkey's agar? (FAI I pl38) (FAQ pl48)

Genetic Laboratory Techniques


R: Chapter 5
PCR (FAII p8l) (FAI2 p84) (R pl74)
Blotting procedures (FAII p8l) (FA/2 p85) (R pl78)
Microarrays (FAII p8l) (FAI2 p8S)
Enzyme-linked immunosorbent assay (ELISA) (FAII p8l) (FAI2 p85)
FISH (FAII p82) (FAI2 p85) (R pl79)
Cloning methods (FAII p82) (FAI2 p86)
Gene expression modifications (FAII p82) (FAI2 p86)
Karyotyping (FAII p82) (FAI2 p86) (R pi58)
Electrophoresis

Copyright'S 2012. Doctors In Trainmg.com. LLC. All Rights Reserved


Gel electrophoresis

Cathode

Molecular
weight

Anode

4. If well I contains DNA sample A, well 2 contains DNA sample B, and well 3 contains
DNA sample C; then what can you say about wells 4, 5, 6, 7, and 8?

5. DNA sequences

Cathode

DNA sequence patient A

Molecular
weight
DNA sequence patient B

Anode

Copyright © 2012. Doctors In Trainmg.com. LLC. All Rights Reserved.


ick Quiz

7. What type of test uses a known antigen to discern the presence of an antibody? (FAQ pS

What type of test is performed in order to diagnose chromosomal imbalances?


(FAII p82) (FAQp86)

Inheritance
R: Chapter 5
Genetic terms (FAII p83) (FAI2 p87)
Hardy-Weinberg population genetics (FAII p84) (FAI2 p88)
Imprinting (FAII p84) (FAI2 p88) (R pl7l)
- Prader-Willi syndrome
- Angelman syndrome
Pedigrees
Modes of inheritance (FAII p85) (FAI2 p89)

9. What is the frequency of the BB phenotype and the Bb phenotype if the frequency of
allele B is 70%?

10. Prader-Willi Syndrome (FAI I p84) (FAQ p88)


• Deletion of proximal portion of chrom I5ql l-ql3 from paternal origin
• Presents in infancy: hypotonia, poor feeding, characteristic facial features (almond shaped eyes,
downward turned mouth)
• Sx: hyperphagia, obesity, short stature (partial GH deficiency), MR, behavior disorders (tantrums,
skin-picking, OCD), hypogonadotrophic hypogonadism ■*• genital hypoplasia, osteoporosis, delayed
menarche
• Dx: confirmed with FISH (fluorescence in-situ hybridization)
• Rx: limit access to food, GH if short stature

Copyright C) 2012. Doctors In Training.com. LLC. All Rights Reserved


II. The numbers in this diagram indicate the age of disease presentation. What is the
name of this phenomenon?

a a o '62

6 U—i—CL ML6 42 45 a
28l "27 c5~^
12. What is the likelihood that child X will have the genetic mutation?

13. If the shaded boxes indicate a phenotypic expression of a genetic mutation, what is the
name given to this
to phenomenon?
y
m
"c
<
u
o o
(J
y
"18
CO
5 <T5
®
IE
u 14. Mitochondrial Inheritance Defects
o • Mitochondrial myopathies (ragged-red muscle fibers seen on biopsy)
• Leber hereditary optic neuropathy
CO
• Leigh syndrome (subacute sclerosing encephalopathy)

[48] Copyright © 2012. Doctors In Training.com, LLC. All Rights Reserved.


of Session Quiz
15. What is the probability that a female heterozygous for an X-linked disease will pass it o
to her son?

16. What is the probability that a female heterozygous for an X-linked disease that mates wttr
a normal male will have a carrier daughter?

What is the probability that a female carrier of an X-linked disease will have a child wit
that disease assuming she mates with a normal male?

If aa symbolizes a recessive disease, what is the likelihood that parents Aa and Aa will have
a phenotypically normal child?

Cystic fibrosis is an autosomal recessive disorder. Two parents that are heterozygous
cystic fibrosis have a normal, non-affected child. What is the probability that the child
homozygous normal?

Upon examination of a pedigree, you note that both males and females are affected w
a disease in every generation. What type of genetic disease is this?

LI. What is the frequency of the Aa genotype and the AA genotype if the frequency of allel
A is 0.95?

If 49% of a particular population is homozygous for a curly hair gene that is dominant j
straight hair gene, what percentage of the population has curly hair?

5. Frequency of CFTR mutation X in patients with cystic fibrosis is 0.1. Cystic fibrosis car

fibrosisby
»caused patients
eitherare homozygotes
mutation for mutation
X or mutation Y? CFTR gene. What percentage of c)
Y in the

24. Two patients have the same mutation on chromosome 15 but have different phenotyr.
expressions. One patient received the mutation from the father while the other recei<
the mutation from the mother What is this an example of?

Copyright © 2012. Doctors In Trainmg.com. LLC. All Rights Reserved.


3 Quest/on Warm-Up
1. What is the rate-limiting enzyme for the following metabolic pathway?
(FAII p95) (FAplOI) (Phys p8l7)
• Glycolysis
• Gluconeogenesis
• TCA cycle
• Glycogen synthesis
• Glycogenosis

2. Which of the oral agents used in the control of type II diabetes has the following
characteristics? (FAI I p304-333) (FAI2 p330) (GG pl255, 1268-1269)
• Lactic acidosis is a rare but worrisome side effect
• Most common side effect is hypoglycemia
• MOA: closes K* channel on ■* cells ■» depolarization ■» Ca2"*" influx ■▶ insulin release
• MOA: inhibits d-glucosidase at intestinal brush border
• MOA: agonist at PPARy receptors

3. What three mineral compounds can be used to treat esophageal reflux?


(FAII p338) (FAQ p368) (GG pl3l6)

Coagulation Basics
R: Chapter 4
Phys: Chapter 36
H: Chapter 116

Coagulation, complement, and kinin pathways (FAII p345) (FAI2 p376) (R p64)
Coagulation cascade components (FAII p346) (FAI2 p346) (R pi 18) (H p986)
Bradykinin (R p65)
Coagulation disorders (FAII p35S) (FAI2 p387) (H p973)
Hereditary thrombosis syndromes leading to hypercoagulability (FAII p356) (FAI2 p388) (R pl22)

Quick Quiz
4. Which coagulation factors are dependent on vitamin K for synthesis? (FAII p345) (FAQ p376) ~T
m
5. What coagulation factor is d e fi c i e n t in hemophilia A? f^j
(FAI I p355) (FAQ p387) (R pi43) (Phys p458)
I
6. What coagulation factor is d e fi c i e n t in hemophilia B? r\
(FAI I p346) (FAQ p376) (Phys p458) q

7. What are some of the effects of bradykinin on the body? r/q


(FAI I p345) (FAQ p376) (Phys 199-200) -,-,
m
p.
8. What is the clinical consequence of a deficiency in either protein C or protein S? O
(FAI I p356) (FAQ p388) (Phys 457-458) (H p987) ' en

Copyright© 2012. Doctors In Training.com. LLC. All Rights Reserved. [I]


Agents Affecting Clotting Factors
GG Chapter 30
H: Chapter 118

Heparin (FAII p362) (FAI2 p395) (GG p853) (H p992)


Lepirudin, bivalirudin (FAII p362) (FAI2 p395) (H p997)
Argatroban, dabigatran
Low molecular weight heparins
Warfarin (FAII p362) (FAI2 p395) (GG p860)
Heparin vs. warfarin (FAII p362) (FAI2 p396)
Rivaroxaban
Thrombolytics (FAII p363) (FAI2 p396)

!MW«:rll
(Phys p459-460)

What lab value is used to monitor the following medications: heparin, warfarin,
enoxaparin? (FAI I p356-36l)(FAQ p388-396) (Phys p460-46l) (H p462-263)

What is the treatment for heparin-induced thrombocytopenia?


(FAI I p356) (FAQ p388, 396) (R pQ3, 668-669)

What are five hereditary thrombosis syndromes?


(FAI I p356) (FAQ p388) (R pQ2) (H p462)

Copyright ;='■ 2012. Doctors In Training.com. LLC. All Rights Reserved.


on Warm"
What are the vitamin K dependent clotting factors? (FAI I p345) (FAQ p376-387) (H p46l.

-,, ,at are some examples of substances eliminated at a constant rate (zero-order
elimination)? (FAI I p233) (FAQ p233)(H p35)

'. What disorder might you suspect in a patient with a perianal fistula? What study would y<
order to further confirm your suspidon? (FAI I p326) (FAQ p355-356) (COA p396, Fig B3.24)

Erythrocyte Basics
R: Chapters 3, 14
Phys: Chapters 32, 35, 83
Embryologic origins of bone marrow (Phys p4l4) (R p590)
Fetal erythropoiesis (FAII pi24) (FAI2 pl3l) (Phys plOI9)
RBC basics (FAII p342) (FAI2 p372) (Phys p4l3)
Blood groups (FAII p345) (FAI2 p375) (Phys p445)
Erythroblastosis fetalis (Phys p447, 1024)

What allows RBCs to change shape as they pass through vessels? (FAI I p342) (FAQ p372)

Where does fetal erythropoiesis take place? In which adult bones does erythropoiesis ■
place? (FAI I pl24) (FAI2

What is the life-span of an RBC (which is the maximum time to expect foreign RBCs to
in a patient after a blood transfusion)? (FAI I p342) (FAQ p372)

What name is given to immature erythrocytes? (FAI I p342) (FAQ p378)

Copyright @ 2012. Doctors In Traimng.com. LLC. All Rights Reserved.


RBC Pathology
R: Chapter 14
H: Chapters 57, 104

Pathologic RBC forms (FAII p348-349) (FAR p378)


Polycythemia (FAII p36l) (FAI2 p394) (R p609)
Heme synthesis, porphyrias, and lead poisoning (FAII p354) (FAI2 p386) (R p660)

8. What are the different forms of hemoglobin (Hb)? (H p852)

Hemoglobin Where Found Structure


HbA 97% of normal hemoglobin 02 02
HbA2 2% of normal hemoglobin 02 02
HbAlc Poorlyrcontrolled diabetes 02 B2-glucose
HbF Fetal hemoglobin 02 Y2 (gamma chains replace beta
chains)
Hb Gower Embryonic hemoglobin t> £3
HbS Sickle cell hemoglobin 02 Bs2 (glu ■> val in ■» chain)
HbC Hemoglobin C disease 02 3C2 (glu ■▶ lys in ■* chain)
Hb Bart's Severe Cf-thalassemia Y4 (no alpha chains)
HbH Severe O-thalassemia B4 (no alpha chains)

What are some of the different causes of polycythemia? (H p456-457)

. What pathologic form of RBC would you see in the following diseases?
• Lead poisoning
• G6PD deficiency
• DIC
• Abetalipoproteinemia
• Asplenia

What is the rate-limiting enzyme in heme synthesis? (FAI I p354) (FAQ p386)

What is the structure of HbH? What disease results in HbH production? What is the
structure of Hb Bart's? What disease results in Hb Bart's production?
(FAI I p349) (FAQ p38l) (R p652)

. Does HbF have more or less affinity for 2,3-bisphosphoglycerate than adult hemoglobin?
(H p852)

Copyright © 2012. Doctors In Training.com. LLC. All Rights Reserved.


(FAI2p386,395)(Rp406)

Which antibiotic matches the following description? (FAI I p68,70,75) (FAQ p78,204)
• Inhibits 50S peptidyltransferase (FAI I p75) (FAI2 p78)
• Binds 50S, blocking translocation (FAII p75) (FA 12 p78)
• Bind 30S, preventing attachment of tRNA (FAI I p74) (FAQ p77)
• Inhibits prokaryotic RNA polymerase (FAI I pl9l)(FAI2 p2l2)
• Inhibits prokaryotic topoisomerase (FAI I p70) (FAI2 p2l I)
• Inhibits prokaryotic dihydrofolate reductase (FAI I p68) (FA12 p2IO)

What is the most likely scenario that a person would receive toxic exposure to arsenic?
(Rp408)

Microcytic Anemia
H: Chapter 103
R: Chapter 14

Anemias (FAI2 p380)


Microcytic anemias (FAII p349) (FAI2p 380-381) (R p640)
- Iron deficiency
- Alpha thalassemia
- Beta thalassemia
- Lead poisoning
- Sideroblastic anemia
Distinguishing iron deficiency anemia from anemia of chronic disease
Lab values in anemia (FAII p353) (FAI2 p385) (R p640)

What test can be used to diagnosis beta-thalassemia minor?


(FAII p349)(FAI2p38l)(Hp65l)

>. What lab findings allow you to distinguish iron deficiency anemia from a microcytic,
hypochromic anemia resulting from thalassemia?
(FAII p349) (FAQ p 380-38l)(H p650-65l)

tat should you rule-out in a male over 50 with new onset iron deficiency anemia?
(FAI I p349) (FAQ p359) (R p308)

Copyright © 2012. Doctors In Training com. LLC. All Rights Reserved.


Macrocytic and Nonhemolytic, Normocytic Anemias
R: Chapter 14
H: Chapters 57, 103

Macrocytic anemia (FAII p350) (FAI2 p382)


- Folate deficiency
- 8/2 deficiency
Nonhemolytic, normocytic anemia (FAII p35l) (FA12 p383)
- Anemia of chronic disease (R p662)
- Aplastic anemia (R p662)

n i L* »• J ^ 14->7 [•JlKfj'li

A patient is diagnosed with a macroJHiSliitSSiwnfifciiErTiS i r f fl H B fi


giving folate alone? (FAII p92) (FAQ p97) (Rp657-659) (H p870-87l)

What is the cause of anemia given the following statement?


(FAII p350-353) (FAQp380-388) (H p454)
• Microcytic anemia + swallowing difficulty + glossitis
• Microcytic anemia + > 3.5% HbA2
• Megaloblastic anemia not correctable by BI2 or folate
• Megaloblastic anemia along with peripheral neuropathy
• Microcytic anemia + basophilic stippling
• Microcytic anemia reversible with B6
• HIV positive patient with macrocytic anemia
• Normocytic anemia + red urine in the morning
• Normocytic anemia and elevated creatinine

What are the causes of aplastic anemia? (FAI I p35l) (FAQ p383) (R p663)

Copyright © 2012. Doctors In Trajning.com. LLC. All Rights Reserved.


I. What are the 3 different mechanisms cells employ to break down proteins?
(FAI I p75) (FAQ p78) (Phys 34-38,41)

Which medication used in the treatment of HIV is known for causing bone marrow
suppression? (FAI I p350) (FAQ p382)

What is the treatment for lead poisoning? (FAI I p350) (FAQ p38l) (GG pl873-l874)

Intrinsic Hemolytic Anemia


H: Chapters 57, 104, 106
R: Chapter 14

Normocytic, normochromic anemias: types of hemolysis (FAII p350) (FAI2 p382) (H p454, Fig. 57-17)
Intrinsic hemolytic normocytic anemia (FAII p352) (FAI2 p382) (H p879)

A child anemic since birth has now been cured with splenectomy. What is the disease?
(FAII p352) (FAQp384)

What findings are associated with hereditary spherocytosis?


(FAI I p352) (FAQ p384) (Phys p420) (R p642)

lat is the difference between the hemoglobin S defect and the hemoglobin C defect?
(FAI I p352) (FAQ p384) (Phys p420) (H p855-857)

What can be seen in erythrocytes in patients that do not have a functional spleen?
(FAI I p349) (FAQ p379) (Phys p4l9)

Copyright © 2012. Doctors In Training.com, LLC. All Rights Reserved.


/ ^ v \

Extrinsic Hemolytic Anemias ^


H: Chapter 106 "*>
R: Chapter 14 "^
Extrinsic hemolytic normocytic anemia (FAII p353) (FAI2 p385) (R p653-6S4) ^

8. Coomb's (+) (FAI I p353) (FAI2 p385) ^


RBC agglutination with the addition of antihuman antibody because RBCs are coated with
immunoglobulin or complement proteins ^
Direct Coomb's ( D AT ) m^
• Prepared antibodies are added to a patient's washed RBC to detect the presence of ^
immunoglobulins already present on the RBC (Using an antibody to detect an antibody.)
• Positive in: hemolytic disease of the newborn, drug-induced autoimmune hemolytic anemia, '
hemolytic transfusion reactions "*)
Indirect Coomb's ^
• Patient's serum is incubated with normal RBC to detect for the presence of antibodies a
• Positive when: antibodies present to foreign blood (used to test blood prior to transfusion,
screening for maternal antibodies to a fetus'blood)

9. Cold Agglutinins (FAI I p353) (FAQ p385) ^


• Antibodies against RBCs that interact more strongly at low temps (4°C) than at body temp "^
• Nearly always ^
• Occur regularly in infections with
/^\
• Problems/disease occurs when there is drculation to a cold extremity ■▶ IgM binds RBC antigen ■»
c o m p l i m e n t f i x a t i o n ^ M A C l y s i s ( a n d o p s o n i z a t i o n ■» p h a g o c y t o s i s ) i

10. Warm Agglutinins (FAI I p353) (FAQ p385) «*,


• Antibodies that react against RBC protein antigens at body temperature ^
• Nearly always ^
• Seen in I)

2)
3 3)
C£ 4) ^
C

i^?\

O
E
I
I
^ -
LU
X
LU
X

[8] Copyrighc <Q 2012. Doctors In Tratning.com. LLC. All Rights Reserved.
d of Session Quiz
What is the difference between a warm agglutinin and a cold agglutinin?
(FAI I p353) (FAQ p385) (R p653)

What are schistocytes? (FAI I p353) (FAQ p378) (R p645-646)

'3. What are two protozoal diseases that can cause hemolytic anemia?
(FAII p353) (FAQ p382, 384)

In what hematologic disorder would you find the following abnormal tests/findings?
• Ham's test
• DEB test
• Heinz bodies
• Basophilic stippling
• Osmotic fragility test

An 11-year-old child presents with a chronic non-healing ulcer on his foot and imaging sh<
a small calcified spleen. What drug can improve his symptoms? (FAI I p386) (FAQ p422

Copyright -TO 2012. Doctors In Training.com. LLC. All Rights Reserved.


What enzyme catalyzes peptide bond formation during protein synthesis?
(FAII p75) (FAQp78)

2 What conditions are associated with target cells? (FAI I p348) (FAQ p379) (R p649)

What name is given to anemia resulting from mechanical destruction of erythrocytes due ■
aortic stenosis or prosthetic heart values? (FAI I p353) (FAQ p382) (R p654)

Platelet Basics
R: Chapter 4
Megakaryocytes (R p625)
Platelets (FAII p342) (FAI2 p372) (R pll7)
Platelet plug formation (FAII p346) (FAI2 p377) (R pi 17)
Thrombogenesis (FAII p347) (FAI2 p377) (R pl2l)

4. Platelet Stimulation (FAI I p346) (FAQ p377)


• Adhesion (endothelial damage, vWF, Gplb)
• Activation
- Secretion of ADP, PDGF, serotonin, fibrinogen, lysosomal enzymes, thromboxane A2, calcium,
thrombin
- thrombin: fibrinogen ■▶ fibrin
- thromboxane A2 ■» vasoconstriction and platelet aggregation
• Aggregation of platelets via Gpllb/llla

5. Von Willebrand Factor (R pi 17, 670)


• Basics: several subunits linked by disulfide bonds, synthesized by endothelial cells and megakaryocytes
• Major functions:
- Complexes with and stabilizes factor VIII (deficiency-* t PTT)
- Platelet adhesion to vessel wall and other platelets (deficiency -* t bleeding time)

£ Platelet Disorders
I H: Chapter 115
LO Platelet disorders (FAII p355) (FAI2 p387)
LU

Copyright © 2012. Doctors In Training com. LLC. All Rights Reserved.


UIZ
What is the cause of ITP? BlSEBItiSvEma
What is the defect in Bemard-Soulier disease? (FAI I p356) (FAQ p387)

What is the life span of a platelet (which is also the maximum life of platelets after a
transfusion)? (FAI I p342) (FAQ p372)

What molecule is expressed on the surface of a platelet after it becomes activated?


(FAI I p346) (FAQ p377)

Other Bleeding Disorders


H: Chapters 115, 116
Mixed platelet and coagulation disorders (FAII p356) (FAI2 p388)

Antiplatelet Medications
GG: Chapter 30
H: Chapter 118

Aspirin (FAII p363) (FAI2 p397) (R p59)


Clopidogrel, ticlopidine (FAII p363) (FAI2 p397) (GGp869)
Abciximab (FAII p364) (FAI2 p397) (GGp870)

NSAIDs inhibit the production of which substance important in platelet aggregation?



After a normal spontaneous vaginal delivery, the new mom bleeds profusely from her
vagina and later from her gums. What abnormal lab values would you suspect?

What is the mechanism of action of the following drugs?


(FAI I p362-364) (FAQ p395-397)
• Streptokinase
• Aspirin
• Clopidogrel
• Abciximab
• Tirofiban
• Ticlopidine
• Enoxaparin
• Eptifibatide

What lab findings are indicative of disseminated intravascular c" ' j:


<DIQ? (FAI I p356) (FAQ p388) (R p673)

Copyright"? 2012. Doctors In Tr.iining.com. LLC. All Rights Reserved.


F »•/ i i^i A [ • J i ■V/« f if 11B * J > 1
What neoplasms are associated with AIDS? (FAI I pl7v

What is the most common inherited bleeding disorder? (FAI I p356) (FAQ p388) (R p670)

What cancer is associated with Hashimoto thyroiditis? (FAI I p298) (FAQ p325) (R p6l3)

Lymphoma
R: Chapter 13
H: Chapter 109
Leukemia vs lymphoma (FAII p357) (FAI2 p389) (R p598)
Leukemoid reaction (FAII p357) (FAI2 p389) (R p595)
Lymphoma
- Hodgkin vs. non-Hodgkin lymphoma (FAII p357) (FA12 p389)
- Reed-Stemberg cells (FAII p357) (FAI2 p389)
- Hodgkin lymphoma (FAII p3S7) (FAI2 p389)
- Non-Hodgkin lymphoma (FAII p358) (FAI2 p390)

4. What hematologic disease matches the following statement?


Most common lymphoma in US Diffuse large B-cell lymphoma

Reed-Stemberg cells Hodgkin lymphoma


Particularly associated with EBV Burkitt and Hodgkin
Lymphomas
A s s o c i a t e d w i t h l o n g t e r m c e l i a c d i s e a s e I n t e s t i n a l T- c e l l l y m p h o m a
Lymphoma equivalent of CLL Small lymphocytic lymphoma
"Starry-sky pattern" due to phagocytosis of apoptotic tumor cells Burkitt lymphoma
Associated with Sjogren syndrome, Hashimoto thyroiditis, Marginal cell MALToma
and H. pylori

Compare the age distribution of those affected by Hodgkin lymphoma to those affected
non-Hodgkin lymphoma. (FAI I p357) (FAQ p389)

What is the most common type of non-Hodgkin lymphoma in adults? In children?


(FAII p358) (FAQp389)

What clinical presentation might lead you to suspect a patient may have lymphoma?
(FAII p357) (FAQp389, 390)

Copyright (£) 2012, Doctors In Training.com. LLC All Rights Reserved.


Leukemias
R: Chapter 13
H: Chapters 109, 110
Leukemias (FAII p3S9-360) (FAI2 p392) (R p600)
Auer bodies (FAII p360) (FAI2 p393) (R p623)
Chromosomal translocations (FAII p360) (FAI2 p93)

8. Acute Leukemia (R p600)


• Rapid onset and rapidly progressive
• Over 5096 myeloblasts (AML) or lymphoblasts (ALL) in the bone marrow
• Numerous blast (immature) cells (> 2096 blasts)
• Often associated with pancytopenia (anemia, bleeding tendency, infection)

ALL AML
• Philadelphia chromosome may be seen (poor prognosis) • Philadelphia chromosome
• Most common in children and young adults rarely seen
• Males > females, whites > blacks • Characteristic Auer rods
• B cell types more common that T cell • 8 different morphological
• 3 morphologic variants and 5 phenotypic variants classifications
• Most all are CD 13/33 (+)
• Bone pain is common
• Usually nonspecific
• Most have the enzyme terminal deoxynucleotide transferase
esterase (+) myeloid cells
(TdT) • Median age of onset is 50
• Very good prognosis in children (up to 9096 remission)
• PAS(-)
• PAS(+)
• Associated with numerous
• Difficult to diagnose on blood smear (others can be diagnosed
risk factors
with smear)

9. 8 types of AML
• MO undifferentiated
• Ml minimal differentiation/maturation
• M2 with differentiation/maturation m
• M3 acute promyelocytic
rn
• M4 myelomonocytic /myelomonoblastic
ON
• M5 pure monocytic /monoblastic
I


M6 erythroleukemia
M7 megakaryocyte n
D
O
C
D
10. AML risk factors
• Radiation, benzene, or alkylating agents (such as in Hodgkin lymphoma treatment)
• Myeloproliferative disease, myelodysplastic syndrome, or aplastic anemia
• Down syndrome, Fanconi syndrome, or Bloom syndrome 52
O
o
Cl

Copyright © 2012. Doctors In Training.com. LLC. All Rights Reserved. [13]


11. ALL type by morphology ALL types by phenotype
• LI-small blasts • common (= LI)
• L2- large blasts with prominent nucleoli • null (= LI)
• L3- large blasts with cytoplasmic vacuoles
•T(=L2)
•B(=L3)

12. Chronic Leukemia (R p603)


• Insidious onset and gradual progression (months to years)
• Mature cells (rather than blasts) (< 596 blasts) /5%

• Can be either myeloid (CML) or lymphoid (CLL)


• Associated with hepatosplenomegaly and lymphadenopathy
• Prominent infiltration of bone marrow by leukemic cells and peripheral WBC counts may be high

CLL CML
• Most common adult leukemia seen in • May progress to AML (80%) or ALL (2096)
western countries • Numerous basophils and PMNs are LAP (-)
• Males > females, whites > blacks • Adults ages 25-60
• Adults over age 50 • Hyperplasia of all 3 cell lines (granulocytic,
• 9596 have B cell markers (rather than T cell)
erythroid, and megakaryocytic) but
• 1096 progress to ALL granulocyte precursors predominate /""tv\

• Characteristic smudge cells and • Philadelphia chromosome (t 9;22) is always


• Autoimmune hemolytic anemia present
• Tends to be indolent • Fatigue, abdominal pain, splenomegaly,
bleeding tendency

Chronic Myeloproliferative Disorders


H: Chapters 109, 110
R: Chapter 13
Chronic myeloproliferative disorders (FAII p36l) (FAI2 p394)
o Polycythemia vera (R p609)
_o CML (FAII p359) (FAI2 p393) (H p9l4) (R p627)
go

CO
L .
<D Multiple Myeloma
U H: Chapter III
crd R: Chapter 13
U Multiple myeloma (FAII p3S8) (FAI2 p39l) (R p609) (H p937)
MGUS(FAIIp3S9)(Rp6ll)
vO

LU

[14] Copyright © 2012. Doctors In Training.com. LLC All Rights Reserved.


ess/on Quiz
13. A patient with anemia, hypercalcemia, and bone pain receives a bone marrow biopsy

what
twhich may beplasma
reveals found on urinalysis?
cells (large, round, off-center nucleus). What is the diagnosis, and

What form of leukemia matches the following statement?


• Most common leukemia in children
• Most common leukemia in adults in US
• AML associated with Down syndrome
• AML that are CDI3 and CD33 (+)
• Characteristic Auer rods

Myelodysplastic syndromes have a tendency to progress to


Myeloproliferative disorders may progress to
Greater than 2096 blasts in marrow
Leukemia with more mature cells and less than 596 blasts
AML that is CD 41 and CD 61 (+)

PAS (+) acute leukemia


Commonly presents with bone pain
Leukemia equivalent of Burkett lymphoma
Numerous basophils, splenomegaly, and negative for leukocyte alkaline phosphatase (LAP)
Most common neonatal leukemia

Always positive for the Philadelphia chromosome (t 9;22)


Only AML that is CD 13 and CD 33 (-)
Acute leukemia positive for peroxidase
Solid sheets of lymphoblasts in marrow
PAS (-) acute leukemia
Always associated with the BCR-ABL genes

Copyright © 2012. Doctors In Training.com. LLC All Rights Reserved.


•JJMWMiIW*
Which protozoa are responsible for the following diseases?
(FAI I pl6l-l62) (FAQ pl75-l77) (R p335)
• Chagas disease
• Protozoal vaginitis
• Malaria
• Birth defects
• Bloody diarrhea
• Foul-smelling diarrhea, flatulence, bloating

After bone marrow transplantation, a patient suffers from dermatitis, enteritis, and hepa
What disease process is occurring? (FAI I p2l5) (FAQ p239) (R p228)

Which regions of the brain are induded in the limbic system?


(FAI I p399) (FAQ p437) (Phys p7l4)

Genetics of Cancer
R: Chapter 7
Cancer epidemiology (FAII p230) (FA12 p255) (R p270)
Review of cell cycle (FAIIp76) (FAI2 p79)
- Roles of cyclins, CDKs, and CDK inhibitors (R p285, Table 7-29)
- Checkpoints (R p286)
Basic molecular basis of cancer (R p276)
Tumor suppressor genes (FAII p228) (FA/2 p253) (R p286)
Oncogenes (FAII p227) (FAI2 p253) (R p279)

4. Molecular Basis of Cancer (R p276)


• Malignant transformation (carcinogenesis) is fundamentally due to nonlethal genetic damage.
• Malignant tumors are derived from clonal expansion of a single precursor cell.
• The four main targets of genetic damage are:
- Proto-oncogenes
- Tumor suppressor genes
- Genes that regulate apoptosis
- DNA repair genes
• Malignant transformation is a multistep process, resulting from multiple mutations.
• The individual cells in a single tumor have varying degrees of malignant potential.

5. Retinoblastoma (R p288)
• 'A of cases are bilateral (both eyes), and all bilateral cases are inherited point mutations.
• The other Va of cases are unilateral (one eye); most of these are sporadic mutations.
• Overall, sporadic mutations account for about 6096' of the mutations in the Rb gene, and these are
always unilateral. In other words, 4096 of retinoblastomas are inherited (which results in either bilateral
or unilateral retinoblastoma).
• In order for the mutation to occur, there must be mutations to both alleles (AKA "two-hits").
- In the heritable form, one of those hits comes from the parent gene, and the other hit arises sporadically.
- In the somatic/sporadic form, both hits arise sporadically.

Copyright 9 2012. Doctors In Training.com. LLC. All Rights Reserved.


6. p53(Rp290)
• Acts through p2l to cause cell-cycle arrest
• Involved at the Gl/S checkpoint and G2/M checkpoint
• Causes apoptosis by inducing the transcription of pro-apoptotic genes (such as BAX)
• Mutations in this gene allow the cell to progress through the checkpoint despite the presence of DNA
damage/mutations

7. Proto-oncogenes
• Normal cellular genes that regulate cell proliferation and differentiation that can become oncogenes

8. Oncogenes
• Genes that promote autonomous cell growth in cancer cells by promoting cell growth in the absence of
normal mitotic signals
• Oncoproteins produced from these genes are devoid of important regulatory elements

9. RAS oncogene (R p282)


• Mutations in RAS is the most common oncogene abnormality in human tumors
• 15-20% of all human tumors contain mutated versions of RAS proteins
• K-RAS mutation ■» colon, lung, and pancreatic tumors (Kolon, panKreatic)
• H-RAS mutation ■» bladder and kidney tumors (Hematuria)
• N-RAS mutation ■» melanomas, hematologic malignancies

What type of cancer is associated with the following tumor suppressor genes? (FAI I p228)
(FAQ p253)
• Rb
• DPC
• p53
• APC
• WTI
• BRCAI and BRCA2

What cancers are associated with a mutation of the K-RAS oncogene? (R p282)

Among men and among women, compare the most common cancers and the most
common cancers causing mortality. (FAI I p230) (FAQ p256)

Copyright <t) 2012. Doctors In Trainmgcom. LLC All Rights Reserved.


Cancer Risk Factors
R: Chapter 7

Geographic and environmental (R p272)


Chemical carcinogens (FAII p229) (FAI2 p2S5) (Rp274, Table 7-3)(Rp309-3ll, table 7-10)
Radiation carcinogenesis (R p3l I)
Age (R p273)
Genetic predisposition (R p273) (R p27S, Table 7-4)
DNA repair defects (R p302)
Diseases associated with neoplasms (R p276) (FAII p227) (FAI2 p252)
Oncogenic microbes (FAII p229) (FAI2 p254) (R p3l2-3l6)

13. Chemical carcinogens (R p272-274)


• Arsenic lung cancer, skin cancers, hemangiosarcoma
• Benzene: leukemia, Hodgkin lymphoma
• Beryllium: lung cancer (possibly)
• Cadmium: prostate and lung cancer (possibly)
• Chromium compounds: lung cancer
• Nickel sulfide: lung cancer, upper airway cancers
• Radon: lung cancer

Radiation carcinogenesis (R p311-312.)


14. What are the most common cancers associated with ionizing radiation?

15. Which cancers are associated with UV radiation exposure? Which type of UV
radiation is most problematic? (R p3l2)

16. Inherited Predisposition to Cancer (R p273-276)


Gene Predisposition
Rb Retinoblastoma, osteosarcoma
APC FAP/colon cancer
P53 Li-Fraumeni syndrome
o
BRCAI. BRCA2 Breast cancer, ovarian cancer z
RET MEN Ha and lib, papillary thyroid cancer n
o
r~
17. What diseases are associated with DNA-repair defects? (R p302)
o
a
-<
18. Oncogenic Bacteria and Helminths
• Strep, bows ■▶ i
• K pylori'■▶ CO
£U
• Schistosoma haematobium ■»
• Oonorchis sinensis"* n"
</>

Copyright © 2012. Doctors In Training.com. LLC. All Rights Reserved. [3]


What neoplasms are associated with the following conditions? (FAI I p227) (FAQ p252)
• Hashimoto thyroiditis
• Down syndrome
• Plummer-Vinson syndrome
• Tuberous sclerosis
• Ataxia-telangiectasia
• Paget disease of bone

Which neoplasm is associated with the following? (FAI I p229) (FAQ p255)
• Nitrosamines
• Asbestos
• Naphthalene
• Arsenic
• EBV
• HPV
• Schistosoma haematobium

Copyright © 2012. Doctors In Traimng.com, LLC. All Rights Reserved.


ueszion warm-
Which nucleus of the hypothalamus fits the following description? (FAI I p399) (FAQ p437)
• Considered the "master clock" for most of our circadian rhythms
• Regulates the parasympathetic NS
• Regulates the sympathetic NS
• Produces antidiuretic hormone (ADH) to regulate water balance
• Mediates oxytocin production
• Regulates the release of gonadotropic hormones (i.e., LH and FSH)

What organism is transmitted by the following vector?


• Freshwater snail
• Ixodes tick
• Reduviid bug
• Anopheles mosquito
• Sandfly
• Tsetse fly

What drugs should not be given to sulfa allergic patients? (FAI I p246) (FAQ p246)

Neoplastic Progression
R: Chapter 7
Overview of neoplastic progression (FAII p225) (FAI2 p2S0) (R p278)
-plasia definitions (FAII p226) (FAI2 p25l) (R p262)
Tumor grade vs. stage (FAII p226) (FAI2 p2SI) (R p322)
Tumor nomenclature (FAII p226) (FAI2 p25l) (R p260) (R p263, Table 7-1)
Tumor differences (FAII p226) (FAI2 p25l) (R p262)

'iilfW:

What type of tumor matches the following description? (FAI I p226) (FAQ p25l)
• Benign tumor of epithelium
• Malignant tumor of blood vessels
• Benign tumor of bone
• Malignant tumor of smooth muscle

What unique enzyme is normally absent in somatic cells, but is active in stem cells and
cancer cells? (R p297)

What are the proangiogenic cytokines? (R p298)

Copyright © 2012. Doctors In Training.com. LLC. All Rights Reserved.


Downstream Effects of Neoplasia
R: Chapter 7
Mets to brain (FAII p230) (FAI2 p2S6)
Mets to liver (FAII p230) (FAI2 p256)
Mets to bone (FAII p230) (FAI2 p256)
Cachexia (FAI2 p252) (R p320)
Paraneoplastic effects of tumors (FAII p229) (FAI2 p255) (R p32l)
Serum tumor markers (FAII p228) (FAI2 p254) (R p326, table 7-12)
Psammoma bodies (FAII p230) (FAI2 p255)

(FAII p228) (FAQ p254)


• Hepatocellular carcinoma (Hep B and C patients)
• Ovarian cancer
• Pancreatic cancer
• Melanoma
• Colon cancer
• Astrocytoma

What is the most common cause of hypercalcemia? What cancers may cause
hypercalcemia? (FAI I p229) (FAQ p255)

What neoplasm is most commonly responsible for the following paraneoplastic


syndrome? (FAI I p229) (FAQ p255)
• ACTH ■* Cushing syndrome
• Erythropoietin ■* polycythemia
• ADH ■» SIADH

Which cancers metastasize to bone? (FAI I p230) (FAQ p256)

Which cancers metastasize to the brain? (FAI I p230) (FAQ p256)

Which cancers metastasize to the liver? (FAI I p230) (FAQ p256)

Copyright @ 2012, Doctors In Trainmg.com. LLC. All Rights Reserved.


Which cranial nerve is responsible for the following actions?
(FAI I p416) (FAQ p456) (COA p1054)
• Eyelid opening
• Taste from anterior 2/3 of tongue
• Head turning
• Tongue movement
• Muscles of mastication
• Balance
• Monitoring carotid body and sinus chemo- and baroreceptors

What CD surface marker fits the following statement? (FAI I p209) (FAQ p23l) (R pi901
• Displayed only by helper T cells
• Displayed only by cytotoxic T cells (and suppressor T cells)
• Found on all T cells (except NK cells)
• Used to ID B cells
• Found on all NK cells and binds the constant region of IgG
• Inhibits complement C9 binding
• Endotoxin receptor found on macrophages

3. What surface molecule on platelets binds fibrinogen? What surface molecule binds von
Willebrand factor? (FAI I p346) (FAQ p376-377) (R p 670)

Cancer Prevention
H: Chapter 82
Risk factor reduction (primary prevention) (H p655)
Cancer chemoprevention (H p656)
Screening and early detection (secondary prevention) (H p659)

4. What are the four most important lifestyle factors that impact cancer risk? (H p655)

5. What are the screening recommendations for the following cancers? (H p66l)
Cancer US Preventive Services Task Force (USPSTF) Recommended Screening
Breast cancer
Cervical cancer
Prostate cancer
Colon cancer

Copyright© 2012. Doctors In Training.com. LLC All Rights Reserved.


.•jTTf?:c«m
What is currently the m< KidSJLfJfflfLflfJlMMIiSigl^^
cancers?

Lung cancer
! Cervical cancer
Renal cell carcinoma
Breast cancer
Colon cancer

What is the only definitive way to prevent ovarian cancer?

Which male patients should be screened for prostate cancer with a serum PSA? (H p66l)

Host Defense Against Cancer


R: Chapter 7
Tumor antigens (R p3l6)
Antitumor effector mechanisms (R p3l8)
Immune surveillance and escape (R p3l9)

9. What types of antigens can the immune system use to distinguish tumor cells from
healthy cells? (Rp316-318)
• Products of mutated genes (ex: RAS oncogene, BCR-ABL proteins)
• Over-expressed normal cellular proteins (ex: tyrosinase products in melanomas)
• Tumor antigens produced by oncogenic viruses (ex: HPV, EBV)
• Oncofetal antigens (ex: OfP, CEA)
• Altered surface glycoproteins and glycolipids (ex: CA 125, CA 19-9)
• Cell type-specific differentiation antigens (ex: CD20)

10. What immune cells are involved in killing cancer cells? (R p318)
• CD8+ cytotoxic T lymphocytes
• Natural killer (NK) cells
• Macrophages

11. What mechanisms to cancer cells use to evade the immune system? (R p319-320)
• Selective outgrowth of antigen-negative variants
• Reduced expression of MHC molecules
• Lack of co-stimulation
• Immunosuppression (TGF-|3)
• Antigen masking (mucopolysaccharides, glycocalyx)
• Apoptosis of the cytotoxic T cells (FasL)

Copyright © 2012. Doctors In Training.com. LLC. All Rights Reserved.


12 What is the primary immune system cell type involved in host tumor immune surveillaa
(Rp3l6)

What enzyme present in melanoma generates peptides that are targets for host T cells?
(RP3I7)

How can tumor cells evade attack from cytotoxic T cells? (R p319)

How might tumor cells induce apoptosis of T cells that would antagonize them? (R p320)

Copyright © 2012. Doctors In Training.com. LLC. All Rights Reserved.


What are the symptoms of inhibiting parasympameTic a
(FAI I p239) (FAQ p265) (GG p225,242)

Which cancers are assodated with the following tumor markers?


(FAI I p228) (FAQ p254) (R p327, Table 7-12)
• PSA . CEA
• AFP . p-hCG
• CA 125 . SIOO
• Elevated alk phos

What medication is used to treat the following parasitic infection? (FAI I p 160-163) (FAQ
PI75-I79) (GG PI8I8)
• Gardia, Entamoeba, Trichomonas
• Plasmodium vivax or ovale
Most flukes and tapeworms

Cancer Drugs I
GG: Chapter 60
Cancer drugs - cell cycle (FAII p364) (FAI2 p398)
Antineoplastics (FAII p364) (FAI2 p398)
Cancer drugs (FAII p365-366) (FAI2 p399)
- Antimetabolites (FAII p365) (FAI2 p399)
- Antitumor antibiotics (FAII p 366) (FAI2 p400)

How does the mechanism of action of methotrexate differ from 5-fluorouracil?


(FAII p365) (FAQp399)

■low does the body metabolize 6-mercaptopurine? (FAI I p365) (FAQ p399)

Which anticancer drug is also often used in rheumatoid diseases as well as ectopic
pregnancies? (FAI I p365) (FAQ p399)

For which cancers is dactinomycin particularly useful? (FAI I p366) (FAQ p400)

Copyright © 2012. Doctors In Training.com. LLC. All Rights Reserved.


Cancer Drugs 2 (FAII p365-366) (FAI2 p399-40l)
GG: Chapter 60

Alkylating agents (FAII p366) (FAI2 p400)


Microtubule inhibitors (FAII p367) (FAI2 p40l)
Other cancer drugs (FAII p367-368) (FAI2 p40l-402)
Common chemotoxicities (FAI2 p402)

fjW-iMii
Which anticancer drug fits the following description? (FAI I p365-368) (FAQ p399-40l)
• Forms a complex between topoisomerase II and DNA
Alkylates DNA, toxicity ■* pulmonary fibrosis
Fragments DNA, toxicity ■* pulmonary fibrosis
Blocks purine synthesis, metabolized by xanthine oxidase
• Cross-links DNA, nephrotoxic, ototoxic

• Nitrogen mustard, alkylates DNA (electrophile that binds DNA)


- Folic acid analog that inhibits dihydrofolate reductase
Prevents tubulin disassembly
Intercalates DNA, produces oxygen free radicals, cardiotoxic
DNA alkylating agents used in brain cancer

Prevents tubulin assembly


Inhibits thymidylate synthase ■▶ decreased nucleotide synthesis
SERM—blocks estrogen binding to ER(+) cells
Mechanism similar to antivirals acyclovir and foscarnet
• Mechanism similar to fluoroquinolones

Mechanism similar to trimethoprim


Monodonal antibody against HER-2 (erb-B2)
Free radical-induced DNA strand breakage
Inhibitor of PRPP synthetase
Reversible with leucovorin

Treatment for choriocarcinoma


Treatment for AML
- Treatment for CML
Prevents breast cancer
Treatment for testicular cancer

Applied topically for AKs and basal cell cancers


Treatment for childhood tumors (Ewing sarcoma, Wilms tumor, rhabdomyosarcoma)
- Inhibits ribonucleotide reductase
SE of hemorrhagic cystitis
Antibody against Philadelphia chromosome

What are the potential side effects of prednisone use? (FAI I p367) (FAQ p40l)

Copyright '3 2012. Doctors In Training.com. LLC. All Rights Reserved.


•raTfffliwwwn
What G protein dasses do the following receptors stimulate? (FAI I p236) (FAQ p263)

Which anticancer drug fits the following description? (FAI I p365-367) (FAQ p399-400)
• Prevents breast cancer
• Treatment for testicular cancer
• Treatment for childhood tumors (Ewing sarcoma, Wilms tumor, rhabdomyosarcoma)
• Inhibits ribonucleotide reductase
• SE of hemorrhagic cystitis
• Antibody against Philadelphia chromosome

What is the rate-limiting step in purine synthesis? In pyrimidine synthesis?


(FAII P68)(FAI2p69)

Studies and Diagnostic Te s t s (


H: Chapter 3 (

Types of studies (FAII pSO) (FAI2 p52) T ■ j


Clinical trial (FAII p50) (FAI2 p52) ~~j
Drug development (GG p7, II)
Meta-analysis (FAII pSO) (FAI2 p58)
Evaluation of diagnostic tests (FAII p5l) (FAI2 p53) i

m
4. What is the sensitivity, specificity, positive and negative predictive value using antibodies <.
to X to detect disease X? p
Autoimmune Disease X j
Present Absent l

800 200
a a.

TJ I

Copyright^ 2012. Doctors In Training.com. LLC. All Rights Reserved.


lSBS^!m!9KSSS!tlS9MSfSs!s
predictive value? (FAI I p5l) (FAQ p53)

6. A physician is looking for risk factors for pancreatitis. He interviews 100 hospitalized

of study is this?

A group of people that smoke and that do not smoke is followed over twenty years.
Every two years, it is determined who develops cancer and who does not. What type of
study is this?

A certain screening test has a 1% false-negative rate. What is the sensitivity of the test7
I patients with pancreatitis and 100 hospitalized patients without pancreatitis. What type

Application of Test Data


H: Chapter 3
Prevalence vs. Incidence (FAII p52) (FAI2 p54)
Odds ratio vs. relative risk (FAII p52) (FAI2 p54)

9. Attributable Risk (AR) (FAI I p52)


• AR — incidence of disease in the exposed group - incidence of disease in the unexposed group
• Example: In a population of sexually-active people, 30% have HPV infection. In a population of
people who are not sexually active, only 5% have HPV infection. The attributable risk of sexual
activitytoHPVis25%.

10. Number Needed to Treat (NNT)


• NNT = l/absolute risk reduction
• Number of patients you would need to treat in order to save/effect one life
• Important number to help determine if a drug should be used or is cost effective
• Example: If out of 10,000 patients that took t-PA during a STEMI, 100 were saved by the t-PA, then the
NNT is 100. In other words, you would need to treat 100 patients in order to save/effect I life.

11. What is the absolute risk reduction and number needed to treat in the following
example: In a study where 100 patients received medication Z to prevent the
development of diabetes and 200 patients did not receive the medication, 10 patients
in the experimental group developed diabetes and 40 patients in the control group
developed diabetes?

Copyright © 2012. Doctors In Training.com, LLC. All Rights Reserved.


Session Quiz

to treat? (FAI I p52) (FAQ p54)

The small town of Mickey City (pop. 8,000) is immediately adjacent to factories whet
asbestos products are produced. During the past year, the prevalence of mesothelio.
has been 16 cases. In the town of Donaldville (pop. 6,000) 15 miles upwind of (and
theoretically safely distant from) Mickey City, there was a prevalence of 3 cases during
same year. What is the relative risk of mesothelioma for the population of Donaldville

A new glucose test arrives and you decide to see how well it works. There is a stand
substance provided that has 90 mg/dL of glucose. Your repeated measurements of th
substance reveal the following values: 54, 56,55, 54, 53, 56, 55, and 54. What can you say
about the precision and accuracy of your new glucose test?

The prevalence of varicella in population A is 2 times the prevalence of varicella in


population B. The incidence is the same in populations A and B. What can be assum
about the disease duration in population A versus population B?

Copyright (£) 2012. Doctors In Tmining.com. LLC. All Rights Reserved.


Which hormones share a common alpha subunit?

What is the difference between a case-control study, a cohort study, and a clinical trial?
Which studies use odds ratios, and which use relative risks? (FAI I p50) (FAQ p52)

Which neoplasm is most commonly responsible for the hormone paraneoplastic syndrome
(FAII p229) (FAQp255)
• ACTH ■» Cushing syndrome
• PTH-related peptide ■» hypercalcemia
• Erythropoietin ■» polycythemia
• ADH ■» SIADH

Bias and Error


Bias (FAII pS3) (FA12 p56)
Statistical distribution (FAII p53) (FAI2 p57)
Statistical hypotheses (FAII p54) (FAI2 p57)
Error types (FAII p54) (FAI2 p57)
Power (FAII p54) (FAI2 p58)

Examples of Bias:
• Selection bias - The referral centers for a trial of a new anticancer drug have more patients with end
stage disease than early stage, so more patients with end stage disease are referred for the trial than
early stage disease.
erkson's bias - Studies performed on patients that have been hospitalized... type of selection bias
ecall bias - Parents of autism patients having a more detailed recall of events and illnesses in their
child's first two years of life compared to parents of healthy controls.
• Sampling bias - A study performed in China may not be generalizable to the US population.
• Late-look bias - Sending a survey out to people diagnosed with a fatal illness 5 years after diagnosis will
preferentially sample those with a low grade disease (or few comorbidities)
• Procedure bias - The positive benefit of a new drug during a study simply may have been due to the
fact that study participants were required to attend clinic monthly, where they received extra disease
education and counseling compared with the controls.
Confounding bias - Are asbestos miners more likely to have cancer because they mine asbestos or
because they are more likely to smoke?
• Lead-time bias - While test PSA-xyz may detect prostate cancer before it is detected by a traditional
PSA, early detection using PSA-xyz does not increase cancer survival compared to traditional PSA.
• Pygmalion effect - An orthopedic surgeon investigator who finds statistically significant benefit of
arthroscopic surgery when compared to non-invasive therapeutic strategies. A chiropractor-led study
that finds significant benefit of the effects of cervical manipulation when compared to traditional
medicine strategies.
• Hawthorn effect - When studying the effects that infection control education has on physicians, the
investigator notes that both the experimental and the control groups improve their hand hygiene.

Copyright© 2012. Doctors In Training.com, LLC. All Rights Reserved.


show no benefit. What type of error is found in study X? (FAI I p54) (FAQ p57)

. Study Y shows that aspirin administration during an Ml offers no improvement in patie


morbidity or mortality. What type of error is present? (FAI I p54) (FAQ p57)

'. What is the difference between the mean, median, and mode? (FAI I p53) (FAQ p57)

(FAII p53) (FAQ p57)

In a randomized, placebo-controlled study of a new blood pressure medication, member


of the study group received their supply of pills at a monthly clinic visit at which the blooc
pressure was checked, and at which the clinic nurses were available to answer questions
and does
I What to instruct thetopatients
it mean say thatona the importance
statistical of controlling
distribution blood pressure
has a positive skew? through diet
exercise, and weight loss. Members of the control group received their supply of placebc
pills in the mail each month. After 6 months, the study group's mean systolic blood
pressure had decreased 11 points from baseline, compared to a 2-point decrease in the
control group. What type of bias may have contributed to the difference in outcomes
between the two groups?

Confidence Interval
Standard deviation vs. standard error of the mean (FAII p54) (FAI2 p57)
Confidence interval (FAII p55) (FAI2 pS8)
t-test vs. ANOVA vs. X2 (FAII pSS) (FAI2 p58)
Correlation coefficient (FAII p55) (FAI2 p58)

10. Confidence Interval


• Dr M: "My study shows that drug X reduces the risk of prostate cancer by 10%."
• Student P: "Does drug X actually reduce the risk of prostate cancer by 10% or was it a flawed study?"
• Dr M: "There is honestly no way that I can know if my study of 1,000 patients reflects the reality of the
entire population. Also, there may be some error somewhere in my study."
• Student P: "So, do you have any confidence at all that what you found in your study reflects reality?"
• Dr M: "I am 95% confident that drug X truly does reduce the risk of prostate cancer by somewhere in
the interval of 8-12%."
• Student P: "Where did you come-up with 8-12%?"
• Dr M: "Using the standard deviation of the data from the study and the sample size of the study, I
determined what the standard error of the mean is. I then took the standard error of the mean from
my study and multiplied it by 1.96. That number turned out to be 2, so I can be 95% confident that
drug X truly does reduce the risk of prostate cancer by 10% +/- 2."
• Student P: "Where did you come up with 1.96?"
• Dr M: "When you want a confidence interval of 95%, you always use 1.96."
• Student P: "What if you wanted a confidence interval of 90% or 99%?"
• Dr M: "For 90% I would use 1.645 x SEM, and for 99% I would use 2.57 SEM."

Copyright';) 2012. Doctors In Training.com. LLC. All Rights Reserved.


What is the equation for determining the confidence interval?

12. In a study of USMLE scores at a particular medical school, the mean score is 230 and
the standard deviation is 20. Knowing that the sample size is 100, calculate the 95%
confidence interval.

13. What is the 99% confidence interval in this same study?

14. In a study of USMLE scores at a particular medical school, the mean score is 230 and
the standard deviation is 20. Knowing that the sample size is 16, calculate the 95%
confidence interval.

In a normal Gaussian curve, what percentage of the sample population falls I standard
deviation, 2 standard deviations, and 3 standard deviations? (FAI I p54) (FAQ p57)

tat is the equation for determining the confidence interval?

In a study of diabetic patients on drug Meca-leca-liver, the average patient HgbAIC after
3 months is 8.0 and the standard deviation is 05. Knowing that the sample size is 10,000,
calculate the 99% confidence interval.

Copyright © 2012. Doctors In Training.com, LLC. All Rights Reserved.


What are the clinical manifestations of Addison disease? What is the cause of Addison
disease? (FAI I p296) (FAQ p324)

A heart failure patient is newly diagnosed with cancer and is being evaluated for
chemotherapy. Which chemotherapeutic agent should be avoided in this patient?

Assuming a normal Gaussian distribution forthe results of a particular test, an average vali
of 35, and SD of 4, what percentage of people will be in the interval between 31 and 43?
(FAIIp53)(FAI2p57)

Disease Prevention: Pediatrics and Obstetrics

Disease prevention (FAII p55) (FAI2 p58)


Prevention in childhood
Immunizations
Prevention in obstetrics

4. Car Seats
• < I yr and < 20 lbs ■* Infant seat in back seat facing backwards
• I -4 yr and > 20 lbs ■* sits in back seat but still in car seat, now have option of facing forward
• Once forward-facing car seat is outgrown (4yrs and 40 lbs) ■» booster seat in the back
• Keep in booster seat until the belt fits correctly (usually 4' 9" and 8-12 yr) ■» then belted with a lap/
shoulder belt in the back seat until 13 years of age

What is the current k'*M illi'iW«• I '■ [•TiT»TiI«=V4A ik!=

meningitis vaccin.

When is it important for a patient with a tetanus prone wound to receive a tetanus
vaccine?

7. Which adult patient populations should receive a pneumonia vaccine?

At what ag @?'<ajmfe|s=!fl? ^©iCTa}^ '"HsSSf'-^I^

Copyright © 2012. Doctors In Trnining.com. LLC. All Rights Reserved.


Prevention of Adult Diseases
H: Chapter 4
Prevention in women
Prevention in CAD
Prevention in DM
Prevention in patients over age 50
Reportable diseases
Leading causes of death in the US by age

9. Important Preventive Measures


Diabetes Serum glucose levels, HbAIC, urine microalbumin,
serum lipids, BP, foot exams, dilated eye exam, influenza
and pneumococcal vaccines
High-risk sexual behavior HIV and syphilis screening, Hepatitis B vaccine, GC/Ch
screen, Pap smear, HPV screen and vaccine, counsel on
STDs and condom use
Smoking Address quitting at each encounter, avoid vitamin A
supplementation, avoid OCPs in women over age 35,
abdominal ultrasound in males 65-75 to r/o AAA,
influenza and pneumococcal vaccines

What are the current indications on the herpes zoster vaccine?

At what age should patients begin receiving screening colonoscopies?

When should a woman begin to receive regular mammograms?

At minimum, what four medications should a patient with a prior Ml due to


atherosclerosis be taking?

What is the target HgbAIC for every diabetic patient?

Copyright © 2012. Doctors In Training.com, LLC. All Rights Reserved.


What are the common side effects of B-blockers? Which patient populations should use
caution when taking B-blockers? (FAI I p242) (FAQ p269)

A small study of USMLE test takers for a particular school revealed scores of 225,225,2,
229,230,240, and 250 with the average score being 232. What is the mean, median, ar
mode for these values? Would this create a positively-skewed curve or a negatively-skew
curve? (FAI I p53) (FAI2p57)

!, A patient of yours develops hypercalcemia from the most common primary tumor arisin;
within bones in adults. What lab findings would you suspect in this patient?
(FAIIp358)(FAI2p39l)

Physiological and Psychological Changes


H: Chapters 70-72

Changes in the elderly (FAII p6l) (FAI2 p63)


Suffering
Grief (FAI I p6l) (FAI2 p63)
Kiibler-Ross grief stages (FAII p6l)

4. What are the Kiibler-Ross grief stages?


• Denial
• Anger
• Bargaining
• Grieving (depression)
• Acceptance
Stages may occur in any order, and more than one stage may be present simultaneously.

5. At what point does grief/bereavement become pathological?


Grief becomes pathological when any of the following are found:
• Depression criteria met for at least 2 weeks after the first 2 months following the loss
• Generalized feelings of hopelessness, helplessness, worthlessness, and guilt
• Suicidal ideation
• Distressing feelings do not diminish in intensity by 6 months
• Inability to move-on, trust others, and re-engage in life by 6 months

uick Quiz

What is the general rule in prescribing new medications to elderly patients?

What are the Kubler-Ross grief stages? (FAI I p6l)

Copyright '.:•> 2012. Doctors In Training.com. LLC. All Rights Reserved.


Diseases and Resources for the Elderly
H: Chapters 4, 70-72
Diseases and medications common in the elderly
Prevention in patients over age 65
Medical equipment commonly needed by the elderly
Resources to aid in caring for the elderly

What are the 3 leading causes of death in patients over age 65?

10. Which patients should be screened for an abdominal aortic aneurysm using an abdominal
ultrasound?

I. At what age should every geriatric patient have had a DEXA bone density scan?

A 60-year-old businessman complains of a lack of successful sexual contacts with women


and a lack of ability to reach full erection. One year ago, he had a heart attack. What
might be the cause of his problem?

Copyright © 2012, Doctors In Trainmg.com. LLC. All Rights Reserved.


I. What patient populations are particularly susceptible to mucormycosis? (FAI I pl58) (FAQ pi >

Chromosomal analysis of a leukemia patient reveals the presence of the Philadelphia


chromosome t(9;22). What is the treatment7

. Which tumor marker would you use to follow the following cancer? (FAI I p228) (FAQ p
• Ovarian cancer
• Hepatocellular cancer (Hep B and C patients)
• Pancreatic cancer
• Melanoma
• Colon cancer
• Astrocytoma

Healthcare System - Part I


Medicine is a science. Healthcare is a business. Healing is an art.

What is the world without insurance?


• Physicians and hospitals can charge whatever they want to patients.
• Patients say they will pay anything to get their health back, and often lack other options for obtaining
health care, due to deficiency of local options or urgency of time.
• Patients sometimes incur excessive unforeseen medical expenses.
• Patients often are unable to compensate healthcare providers for the services rendered.
• Healthcare providers are less willing to take risks on establishing new healthcare institutions.

Healthcare Terms You Need To Know - Table #1


Premium The amount the insured person has to pay the insurance
company (usually paid monthly).
Co-pay The amount the insured person pays at the time of service
(e.g., $30 for a clinic visit or $15 for a particular drug)
Deductible The amount an insured person must pay "out-of-pocket"
before the health insurance begins to pay.

6. Health Insurance - Private Pay Insurance - Third-party payer


Party Financial Duty Risk Reward
Patient - Monthly premiums Paying more to (1) Financial benefit if the cost
insurance than what is of medical care exceeds what
- Co-pays received in medical care is paid to insurance
- Deductible
(2) Peace of mind allows the
individual to focus on other
things
Insurance Health care expenses of Paying more for a patient's Financial benefit if the patient
patient beyond what the medical care than what is pays more than the cost of
patient is required to pay received financially from medical care
the patient
i

Copyright ©2012. Doctors In Training.com. LLC. All Rights Reserved.


%
!/ $>

7. What problems came about when health insurance was introduced?


• In short, everyone tries to get the biggest slice of the finandal pie now in the hands of the hearth
insurance companies.
• Health insurance companies leverage their volume of subscribers to drive down the reimbursement to
medical care providers. Providers still provide care because they agreed to do so.
• Hearth care providers and hospitals counter by increasing the cost of medical care. Insurers pay
because they agreed to do so.
• Patients demand more and more care in order to "get what they have already paid for."
• Insurance companies ended up receiving too little funds from patients to pay the rising costs of health
care.

8. Healthcare Terms You Need To Know - Table #2


Pre-existing condition A condition that a patient is known to have that is not covered
by health insurance.
Lifetime maximum The maximum amount that an insurance company agrees to
pay, as specified in the plan that is purchased.
Network The group of healthcare providers that has agreed to a
reduced payment in order to have access to a larger number
of patients.
Major medical High deductible plan that covers the patient in the event of
excessive medical expenses

9. Health Maintenance Organization (HMO)


• PCP is the gatekeeper to more specialized care.
• In order for medical expenses to be covered, the provider has to be "in-network".

10. Preferred Provider Organization (PPO)


• There is no gatekeeper to the specialist.
• Patient can see whomever they want; however; the cost is higher for "out-of-network" medical care.

11. What problems came about when HMOs and PPOs were introduced?
• The HMO and PPO drive down the payments to physicians and hospitals.
• As physicians receive less compensation, the quality of visit is decreased in order to generate quantity of
visits.
• The patient pays more and more for less and less medical care.
• In short, the third party payers drove down the cost of care. The physidans counter by decreasing the
quality of care. The patient loses.
• Physicians that are unsatisfied with the new arrangement exit the system and establish new systems.

[12] Copyright ©2012. Doctors In Training.com, LLC. All Rights Reserved.


12. Newly-established systems established by physicians
Minor emergency clinics • Heavily utilized by patients without any third-party coverage
• Usually offers no primary or secondary prevention
• Physician refuses to take any third-party payment, essentially
Cash-only clinic
returning back to the old patient-physician system
• Patient may or may not carry additional "major-medical only"
coverage
Sliding scale clinic A patient pays a nominal fee based on income. This is
usually run by a charity with a volunteer physician in order to
minimize costs.
Concierge medicine Physician charges an annual fee (e.g., $l200/yr) for a
specifically designated set of services (e.g., unlimited visits, or
4 visits a year)
Boutique medicine Physician offers unique services not covered by insurance but
desired by affluent patients (usually cosmetic in nature) in
order to increase clinic revenue (e.g., Botox injections)
Group physician networks In an effort to exclude third-party systems and regain control
- Clinic systems of their own payments, physician groups join clinics together,
- Physician hospitals build their own hospitals, and run their own surgery centers.
- Day-surgery centers

Quick Quiz m
13. What is the difference between a premium, co-pay, and deductible? O
n
14. Which usually offers patients a greater variety of physicians to choose from: HMO or PPO?
m
15. Why might a 63-year-old patient with newly diagnosed multiple myeloma have a difficult
time obtaining insurance? What hope might this patient have of medical care in the future?
o
Q
m
16. Healthcare Terms You Need to Know - Table #3
Formulary The medications for which insurance companies will pay.
Generics are preferred.
3
Utilization Management Evaluation of the appropriateness, necessity, and efficiency of
health care services.
Resource-Based A patient pays a nominal fee based on income. This is
Relative Value Scale usually run by a charity with a volunteer physician in order to
minimize costs.
n
17. Health Plan - Managed Care Organizations (MCO) - Health Care Organizations
• Organization that attempts to maximize quality of care and minimize the cost of care. m
• Uses techniques such as encouraging patients and physicians to choose less costly care, controlling
inpatient admissions and lengths of stay, and emphasizing preventive medicine.
• Accomplished through a designated utilization management person or department.

Copyright ©2012. Doctors In Training.com. LLC. All Rights Reserved. [13]


18. What about those without the financial resources for healthcare?
• Medicare
• Medicaid
• Children's Health Insurance Plan (CHIP)
• City-, county-, or state-funded health networks
• Federally-funded teaching hospitals
• Federally-funded VA hospitals
• Universal health care

19. How are physicians compensated?

Fee-for-service • Surgical procedure


Payment is provided for a
specified service • Clinic visit (e.g., 99211-99215)
• Inpatient visit (e.g., 99231 -99233)

Capitation Fixed payment for a period • ER shift/Minor emergency shift


of time or number of • Concierge practice
patients • HMO

Salary Specified amount, • Universities


regardless of work • Hospital administration
performed • Base salary 0&1

Pay for performance Payment is increased if Increased pay by the health


a physician meets pre- care organization for meeting
established targets certain criteria such as preventive
medicine targets (vaccines,
colonoscopy, HgbAlc)
Nonpayment for No payment is made for A funding entity may agree NOT
performance a complication that could to pay a physician for complications
have been avoided such as leaving a catheter in place,
nosocomial infections, and surgical
site infections

20. Current Problems


• "Young Invincibles" These are people (usually under 30) that tend to not have many health problems and
tend not to purchase insurance. In order for a third-party payer to afford to care for an ailing patient at a
low premium cost to that patient, healthy patients must be present in the system to bear the burden of
the additional expense.
• Excessive preexisting conditions. There are some patients that are unable to get any health insurance at a
reasonable cost because of their preexisting conditions. The provider does not want to take on the high
risk of providing care for the patient.
• Freedom of choice. Patients that need others to carry the burden of their healthcare expenses are at
odds with those that want to be free to not have to carry that burden. The US government would like
everyone to carry the burden via the "Individual Mandate" in the PPACA. Others argue that this violates
their freedom.
• Excessive complexity. All this complexity leaves patients at a loss to understand where they need to go
for healthcare. Additionally, this complexity requires hiring individuals to manage the complexity which
drives up the cost of healthcare.
• Everyone still wants the biggest possible piece of the pie. Patients want the highest-costing care. ^^
Physidans want the highest reimbursements. Insurance providers want the largest profits. Drug

y companies want the largest possible profits.


• Physician shortage. Many physicians now see better opportunities elsewhere and are leaving full-time
Q medicine. Older physicians are tired of the complexity and will leave practice if reimbursements decrease.
LU The population continues to increase and the percentage of geriatrics in the population is also increasing.
/SIT,
• Insufficient government funds. It will be very difficult for the government to pay for Medicaid and for
Medicare for the baby boom generation.

[14] Copyright © 2012. Doctors In Training.com. LLC. All Rights Reserved.


21.Emerging Solutions
• Government intervention to regulate the system.
• Patients taking responsibility for their own health via the internet, OTC medication options, and
complementary and alternative medicine.
• Patients seeking US-trained physicians practicing in developing countries where the cost of care is low.
• New generic medications that help lower the cost of health care.
• Cooperative healthcare teams that help reduce the need for costly hospitalization.
• More mid-level practitioners to meet the rising demand
• A four-level healthcare system:

Level Burden carried by: Capabilities Examples


Level 1 Public • Minimal screening and • Charity clinics
(Sufficiently-Funded) prevention • ERs
• Major medical • VA hospitals
provided, but not the • County hospitals
highest quality
Level 2 Balance of • Lowest cost health • Minor-emergency
(Well-Funded) physicians, insurance plans clinics
patients, and • Minimal screening and • Most hospitals in
business
prevention the US
• Good quality medical • Most clinics that
care accept insurance
Level 3 Balance of • Moderate cost health • Personal PCP
(Greatly-Funded) physicians, insurance plans • Most hospitals in
patients, and • Good screening and the US
business • Any clinic that
prevention
• Good quality accepts insurance
medical care
Level 4 Benefactors, • Highest cost health • Exclusive private
(Abundantly-Funded) employers, and insurance plans hospitals in large cities
wealthy patients • Private pay • Executive physicals
• Highest quality • Concierge medicine
medical care • Boutique medicine

What is the difference between being paid by capitation and fee-for-service?

What must a hospital submit to a third-party payer in order to receive payment for services?

lat department in a hospital oversees the maximization of the quality of care while
minimizing the cost of care?

Copyright © 2012. Doctors In Training.com. LLC. All Rights Reserved.


I. What type of thyroid cancer matches the following: tementi

• Second most common type of thyroid cancer (10%)


• Activation of receptor tyrosine kinases
• Hashimoto thyroiditis is a risk factor
• Cancer arising from parafollicular C cells
I* Most common type of thyroid cancer (70-75%)

2. Metastasis to the brain, liver; and bone commonly come from which locations?
(FAII p230) (FAQp256)
Brain Liver Bone

What changes in sleep patterns and sexual anatomy are seen in the elderly?
(FAII p6l) (FAQ p63)

Ethical Principles
H: Chapter 9
Core ethical principles (FAII p57) (FAI2 p59)
Informed consent (FAII p57) (FAI2 p59)
Exceptions to informed consent (FAII p57) (FAI2 p59)
Consent for minors (FAII p57) (FAI2 p59)
Decision-making capacity (FAII p57) (FAI2 p60)
Advance directives (FAII p57) (FA12 p60)
Confidentiality (FAII p58) (FAI2 p60)
Exceptions to confidentiality (FAII p58) (FAI2 p60)
Malpractice (FAII p58)

A 68-year-old man is diagnosed with incurable glioblastoma multiforme. His family asks you,
the doctor; not to tell the patient How do you handle the situation?

A patient had made it clear to you on previous visits that if something should happen to
him that he would not want surgery. The patient now comes to you with a condition that
requires surgery, and because of his condition, he is incapable of providing for you his present
feelings on the matter: His friend relays to you that the man has told him numerous times
that he does not want surgery. His wife later shows up and tells you to do everything you
can do to save her husband's life, induding surgery. What should you do?

What should you do if a minor requests birth control during a clinic visit in the absence of her
parents? (FAI I p59)(FAI2p6l)

A 36-year-old female has a first degree relative who had breast cancer at age 40. You
recommend a mammogram, but she refuses to have one because she says it is too pain

Copyright © 2012, Doctors In Training.com. LLC. All Rights Reserved.


Ethical Scenarios
Ethical situations (FAII p59) (FAI2 p6l)

8. You have to deliver bad news to a patient of yours who you have just discovered to have
lung cancer The patient is a 52-year-old male in marketing that has smoked I pack a day

the bad news the patient replies, "How can this be? I'm a healthy guy. I eat right, watch
my weight, and exercise regularly." What is the most appropriate next response?

I for You find yourself


the past 25 years,attracted to regularly,
exercises your 24-year-old patient.aHow
and maintains do you
healthy diet.handle the situation
After you disclose

,. A 72-year-old man in the hospital with an Ml refuses to take his aspirin on the ground
that it makes him feel "funny". What do you do?

II. How should you handle a situation where you smell alcohol on a physician's breath
that physician is practicing medicine?

Copyright © 2012, Doctors In Training.com. LLC. All Rights Reserved.


Question

Rectum
External anus
Testes
Scrotum

What are the leading causes of death in ages 1-14 and in ages 25-64? (FAI I p56)

. Which anticancer drug fits the following description? (FAI I p2l 6) (FAQ p399)
• Fragments DNA, toxicity ■» pulmonary fibrosis
• Blocks purine synthesis, metabolized by xanthine oxidase
• Folic acid analog that inhibits dihydrofolate reductase
• Prevents tubulin disassembly
• DNA alkylating agents used in brain cancer
• SERMs—block estrogen binding to ER(+) c

Diaphragm and Respiration


Phys: Chapter 37
Diaphragm structures (FAII pS03) (FAI2 p560) (COA p308)
Embryology of the diaphragm
Diaphragmatic hernia (FAII p3l6) (FAI2 p345) (COA p3l7)
Controls of the diaphragm (Phys p505-SI3)
Muscles of respiration (FAII p504) (FAI2 p560)

lick Quiz
What structures traverse the diaphragm, and at what vertebral levels do they pass throuj

Which molecule has a greater direct effect on respiration at the brainstem, oxygen or
hydrogen ion? (Phys 506)

By what mechanism does oxygen effect respiration? (Phys p509)

What is the difference between a hiatal hernia and an indsional hernia after an abdominal
surgery? (FAI I p3l6) (FAQ p345)

Copyright @ 2012. Doctors In Training.com. LLC. All Rights Reserved.


Respiratory Tree and Alveoli
COA: Chapter I
Phys: Chapter 39
Respiratory tree (FAII p502) (FAI2 p558)
Bronchopulmonary segments (FAII p503) (COA pi 14)
Lung relations (FA 11 p503) (FAI2 p560)
Pneumocytes (FAII p502) (FAI2 p559)
Gas exchange barrier (FAII p502) (FAI2 p559) (Phys p49l)

•HE
What histological change takes place in the trachea of a smoker?

A patient in the ER is having anaphylaxis. You make an incision beneath thyroid cartilage to
establish airway. What structure was cut?

I What cell type proliferates during lung damage?

What amniotic fluid measurement is indicative of fetal lung maturity?

A young woman has infertility, recurrent URIs, and dextrocardia. Which of her proteins is
defective?

Copyright © 2012. Doctors In Training.com. LLC. All Rights Reserved.


estfon Wi

(FAII p204) (FAQp226)

2. Compare the leading causes of death in ages 15-24 to those in ages 25-64? (FAI I p56)

Which protozoal organism matches the following statement?


(FAII Pl60-l6l)(FAI2pl75-l76)
• Diarrhea in campers and hikers
• Itchy vaginitis
• Sandfly is the vector
• Ixodes tick is the vector
• Anopheles mosquito is the vector
• Sodium stibogluconate is the treatment
• Suramin or melarsoprol is the treatment
• Maltese cross seen in RBCs
• Treat with metronidazole
• Severe diarrhea in AIDS patients

Copyright © 2012, Doctors In Training.com. LLC. All Rights Reserved.


Lung Volumes and Hemoglobin
Phys: Chapters 37, 40

Important lung products (FAII p504) (FA/2 p56l)


Lung volumes (FAII pS04) (FAI2 p56l) (Phys p469)
Determination of physiologic dead space (FAII p505) (FAI2 pS62)
Lung and chest wall (FAII pS05) (FAI2 p562)
Hemoglobin (FAII p505) (FAI2 p562)
Hemoglobin modifications (FAII p506) (FAI2 p563)
Oxygen-hemoglobin dissociation curve (FAII pS06) (FAI2 p563) (Phys p498)

4. Label the following diagram of lung volumes.

5 6
V)

c £

>/\aa/
o 3 +

c 1 i' * -* m.

u
c
rcj
C~
V1 7

Time i
_o
GO
o
E
<u
I 5. What substances are known for causing methemoglobinemia?

I
(N

<

o
D
o_

[4] Copyright © 2012. Doctors In Training.com, LLC. All Rights Reserved.


lick Quiz
6. Label the following diagram of lung volumes

The following lung volumes are obtained from an elderly smoker FRC 5.0L, IRV I.5L,
2.0L, VC 3.5L What is his total lung capacity?

If a lung collapses, what happens to the intrathoracic volume? (FAI I p505) (FAQ p562

What is the treatment for methemoglobinemia? (FAI I p506) (FAQ p563)

Pulmonary Circulation and Pulmonary Hypertension


COA: Chapter I
H: Chapter 250

Pulmonary circulation (FAII p507) (FAI2 p564) (COA pi 16)


Pulmonary vascular resistance (PVR) (FAII p508) (FAI2 pS64)
Pulmonary hypertension (FAII p507) (FAI2 p564) (H p2076)
Treatment for pulmonary hypertension (H p2078)
- Bosentan (FAII p5l6) (FAI2 pS77) (GG pi 10) (H p2079)

10. Primary Pulmonary Hypertension (FAI I p507) (FAQ p564)


• AKA Idiopathic Pulmonary Arterial Hypertension
• Associated with abnormalities in BMPR2: Bone Morphogenetic Protein Receptor type II
• Associated with HIV and Kaposi sarcoma (HHV-8)
• More common in women, average age is 36

Copyright © 2012. Doctors In Training.com. LLC. All Rights Reserved.


ension? (FAI I p507) (FAQ p564)

What are some the secondary causes of pulmonary hypertension? (FAI I p507) (FAQ p564)

What are some of the treatment options available for pulmonary hypertension? |
(FAII p5l6)(FAI2p577)

is the mechanism of action of bosentan? (FAI I p5l6) (FAQ p577)

Copyright © 2012, Doctors In Training.com, LLC. All Rights Reserved.


uestion Warm-Up
I. What is the characteristic sequence of the promoter region? What does a mutation i
the sequence cause? (FAI I p72) (FAQ p75)

2. What are the pathological characteristics of the arteries in pulmonary hypertension?


(FAII p507) (FAQp564)

3. What is the rate-limiting enzyme for the following metabolic pathway?


• Urea cycle
• Hexose monophosphate pathway
• Fatty acid synthesis
• p-oxidation of fatty acids
• Ketone body synthesis
• Cholesterol synthesis
• Bile acid synthesis
• Heme synthesi

Oxygenation
Phys: Chapters 39,40

Oxygen content of blood (FAII p508) (FAI2 p565)


Alveolar gas equation (FAII p508) (FAI2 p565)
Oxygen deprivation (FAII p508) (FAI2 p565)

Copyright © 2012. Doctors In Training.com, LLC. All Rights Reserved.


4. What controls the amount of oxygen the body receives?
Factor Effector Source of effector
Inspired air 02 and C02 Mankind, nature, altitude
concentrations
Air quality Smoke, NOx, ozone Society, industry, cigarettes
Upper airway Obstructed or open Environment
Bronchial tree diameter Histamine, inflammation Immune system
Parasympathetics Vagus nerve, brainstem
Sympathetics Adrenals
Lung diffusion barrier Thickness Immune system
Surface tension Surfactant Type II pneumocytes
Respiratory rate and Diaphragm, phrenic nerve Carotid body, aortic body
depth
Pulmonary circulation Prostaglandins Stress
Blood flow
Number of RBCs Erythropoeitin Kidney, bone marrow
Nutrients (iron, BI2, folate) Gl tract
Concentration of Prostaglandins RBCs in bone marrow
hemoglobin in RBCs
Type of hemoglobin Transcription, translation Local O2 demand and usage
Ability of 02 to CO2 , acidosis, temperature
dissociate from Hgb 2,3-DPG
Local vasodilation Prostaglandin, adenosine,
lactic acid, serotonin,
bradykinin, histamine
Local occlusion Thromboxane Platelets

What is the normal value for the A-a gradient? What might an elevated A-a gradient
indicate? (FAI I p508) (FAQ p565)

A 42-year-old woman with fibroids is chronically tired. What is the most likely diagnosis, and
what changes have occurred in the oxygen content and saturation?

'. Patient is shown to have hypoxia and CXR reveals an enlarged heart What is the most likely
cause of hypoxia?

Copyright (>:> 2012. Doctors In Training com. LLC. All Rights Reserved.
Ventilation
Phys: Chapter 40
VIQ mismatch (FAII p509) (FA12 p566)
COl transport (FAII p509) (FAI2 p566)
Response to exercise (FAII p509) (FAI2 p567)

WBS&aSM
obstruction? During blood flow obstruction?

How is CO2 transported from the tissues to the lungs? (FAI I p509) (FAQ p566)

How do CO2 levels in circulation change during exercise? (FAI I p509) (FAQ p566)

Copyright © 2012. Doctors In Training.com. LLC. All Rights Reserved.


In what disorder is there an abnormal breakdown of elastin? (FAI I p80) (FAQ p84)

What is the name of the genetic syndrome that fits the following description?
• Alcoholics with BI deficiency and neurologic defects
• Abnormal collagen type I synthesis
• Heinz bodies
• Musty/mousy odor, albinism, MR, eczema
• Hyperextensible skin, loose joints, bleeding tendency
• Hypoglycemia, jaundice, cirrhosis
• Bloating, cramps, osmotic diarrhea
• Dark brown urine, organs, and connective tissue: benign disease
• Multiple fractures and blue sclerae

3. What questions are asked during the 4 clinical phases of drug development?
(FAII p50) (FAQ p52)
Phase I-
Phase II-
Phase III-
PhaselV-

Altitude, Aviation, and Space


Phys: Chapter 43
High altitude
- Response to high altitude (FAII p509) (FAI2 p567)
- Effects of low oxygen pressure on the body (Phys p527)
Aviation
- Effects of acceleratory forces on the body in aviation and space physiology (Phys p53l)
Space
- Artificial climate in the sealed spacecraft (Phys p533)
- Weightlessness in space (Phys p533)

Copyright © 2012. Doctors In Training.com, LLC. All Rights Reserved.


How does the body compensate for hypoxia at high altitude? (

now much does the H&H change in a person that has acclimatized to a hypoxic
environment for weeks? (Phys p529)

At what positive G force does visual "black-out" occur? Why does this occur? (Phys p531'

How is the body affected by a prolonged stay in space at zero gravity? (Phys p533)

Deep Sea and Hyperbarics


Phys: Chapter 44
Effect of high partial pressures of individual gases on the body (Phys p535)
Scuba diving (Phys p539)
Special physiologic problems in submarines (Phys p540)
Hyperbaric oxygen therapy (Phys p540)

n Tf ■• j * y = + > 11

What physiologically is taking place in decompression sickness? (Phys p538)

What are the symptoms of decompression sickness? (Phys p538)

m For what conditions is hyperbaric oxygen therapy particularly helpful? (Phys p540)

Copyright © 2012. Doctors In Training.com. LLC. All Rights Reserved.


Which immunosuppressant matches the following statement?
• Precursor of 6-mercaptopurine
• May prevent nephrotoxicity with mannitol diuresis
• Antibody that binds to CD3 on T cells
• Antibody that binds IL-2 receptor on activated T cells
• Inhibits inosine monophosphate dehydrogenase
• Inhibits calcineurin ■» loss of IL-2 production ■▶ blockage of T cell differentiation and activation
• Metabolized by xanthine oxidase, therefore allopurinol increases its toxicity

What is the treatment for homocystinuria?

What is the treatment of choice for the following protozoa? (FAI I pl6l) (FAQ pl75-l76)
• Trichomonas vaginalis
• Trypanosoma cnm
• Plasmodium vwax
• Leishmania donovani
• Entamoeba histolytica
M f W, :

PE, DVT, and COPD


^ H: Chapters 260, 262
R: Chapter 15
Embolus types (FAII p5IO) (FAI2 p567) (R pl25-l27)
lr, Deep venous thrombosis (FAII pSIO) (FAI2 p568)
PE
-^ Obstructive lung disease (COPD) (FAII p5IO) (FAI2 p 568) (H p2l5l)
Obstructive vs. restrictive lung disease (FAII p5l2) (FAI2 p568-569)

q- 4. Virchow's triad (FAI I p5IO) (FAQ p568)


Q_ • Stasis: post-op, long trips, cast, pregnancy
• • Hypercoagulability: sickle cell, polycythemia, CHF, estrogen excess, smoking
' • Endothelial damage: fracture, post-op, postpartum
LU
0_ 5. DVT (FAI I p5IO) (FAQ p568)
i • Sx: swollen foot/ankle (unilateral), ± pain, ± Homan's sign (pain with ankle dorsiflexion), ± palpable cord
'-*) • Prevent: sq heparin bid-tid, SCDs/compression stockings, long-term warfarin
• Rx: heparin until warfarin therapeutic

6. Pulmonary Embolism (PE)


. • Sx: pleuritic CP, SOB, cough, hemoptysis (rare), fever, tachypnea, tachycardia, AMS/confusion
• Studies: elevated d-dimer, ± DVT on LE US, usually nl CXR, large Aa gradient on ABG, ± EKG
changes (SIQ3T3 - wide S in lead I, large Q and inverted T in lead III), CT scan, V/Q scan, gold
I standard = pulm angiogram
Q_ • "Saddle embolus" = death
• Rx: if massive PE then consider thrombolysis, but usually just heparin/warfarin

12 1J Copyright 6) 2012. Doctors In Training.com. LLC All Rights Reserved.


ick Quiz
DBSRUn

. patient suffers a stroke after incurring multiple long bone fractures in a skiing accident
What caused the infarct?

A patient with a recent tibia fracture and no history of COPD or asthma is shown to haw
hypoxia. CXR is normal. What is the cause of the hypoxia, and what disease process does
mimic?

How does the emphysei king differ from the emphysema caused by al.
sindefiden

Asthma
H: Chapter 254
R: Chapter 15
GG: Chapter 36
Asthma medication (FAII p5l5) (FAI2 p576)
Expectorants (FAII p5l6) (FAI2 pS77) (GGplOS7)
HI blockers (FAII p5l4) (FAI2 pS75) (GG p9l8)

II. What is the differential diagnosis for eosinophilia?


mi

12. Antihistamines
Drug Use
Cyproheptadine Appetite stimulant
Promethazine Nausea, vomiting
Chlorpheniramine OTC allergy/cold
Hydroxyzine Sedation, itching
Meclizine Vertigo

Copyright©2012. Doctors In Training.com. LLC. All Rights Reserved.


responsible for anaphylaxis in this patient?

What asthma medication fits the following statement (FAI I p5l5) (FAQ p576)
• Inhaled treatment of choice for chronic asthma
• Inhaled treatment of choice for acute exacerbations
• Narrow therapeutic index, drug of last resort
• Blocks conversion of arachidonic acid to leukotriene
• Inhibits mast cell release of mediators, used for prophylaxis only
• Inhaled treatment that blocks muscarinic receptors
• Inhaled long-acting 02 agonist
• Blocks leukotriene receptors

A patient has an extended expiratory phase. What is the disease process?

Copyright © 2012. Doctors In Training.com. LLC. All Rights Reserved.


estion Wa
What are the two HIV envelope proteins and the drugs that interfere with them?
(FAII pi73) (FAQ pi92)

A patient has a genetic disease in which the treatment includes protein restriction to
prevent mental retardation, ketoacidosis, and death. What is the diagnosis?

3. What pathology fits the following high-yield statement? (FAI I p5l8-520) (FAQ p580-582)
• Opacities seen on x-ray on both sides of the carina
• Dermatitis, diarrhea, dementia, possibly death
• Greenish rings around the periphery of the iris
"• Elastic skin, joint hypermobility
1 Enlarged, hard, left supraclavicular lymph node

Restrictive Lung Diseases and ARDS


H: Chapters 261, 268
R: Chapter 15
Restrictive lung disease (FAII p5ll) (FAI2 p569)
Acute respiratory distress syndrome (ARDS) (FAII p5ll) (FA12 p570)
Neonatal respiratory distress syndrome (FAII pSIl) (FAI2 p569)

4. ARDS (FAI I p5ll) (FAI2 p570)


Shock, infection, toxic gas inhalation, aspiration, high [O2], pancreatitis, heroin OD
i
Inflammatory cells / mediators and oxygen free radicals
I
Damage to endothelial or alveolar epithelial (type I) cells
;
Diffuse Alveolar Damage (DAD) and Hyaline Membrane Disease (HMD)

. A preterm infant has difficulty breathing. An x-ray reveals diffuse ground glass appearani
with air bronchograms. What is the diagnosis, and what could have prevented this condition?

> patient develops AFIDS from an occupational inhalation of nitrogen dioxide. What
histologic change is seen in a patient recovering from ARDS?

7. What are common causes of ARDS?

Copyright© 2012. Doctors In Training.com. LLC. All Rights Reserved.


Pneumoconiosis and Sarcoidosis
H: Chapter 261
R: Chapter 15
Pneumoconiosis (FAII p5ll) (FAI2 p569)
Sarcoidosis (FAII p386) (FAI2 p42l)
Brief review: Wegener granulomatosis, bleomycin, busulfan, amiodarone
Sleep apnea (FAII p5l2) (FAI2 p570)
Lung-physical findings (FAII p5l2) (FAI2 p57l)

8. Coal Workers Pneumoconiosis (CWP)


• Anthracosis (mild) - black pigment in lung
• Simple CWP - small fibrotic lung nodules
• Complicated CWP - progressive massive fibrosis

9. Sarcoid
• Granulomas
• RA
• Uveitis (eye)
• Erythema nodosum (tibial)
• Lymphadenopathy (hilar, bilateral)
• Idiopathic
• NotTB
• Gamma globulinemia
• (ACE increase)

10. H&E of lung biopsy from a plumber shows elongated structures with clubbed ends in tissue.
What is the diagnosis and what is he at increased risk for?

I. What are the manifestations of sarcoidosis? (FAI I p386) (FAQ p42l)

What do patients with silicosis need to be worried about7 (FAI I p5l I) (FAQ p569)

CXR shows pleural effusions. What are the clinical findings?

A tall, thin male teenager has abrupt onset dyspnea and left-sided chest pain. Percussion on
the affected side reveals hyperresonance, and breath sounds are diminished. What is the
diagnosis?

Copyright © 2012. Doctors In Training.com. LLC. All Rights Reserved.


m-Up

(FAQ p346)

What is the triad of Kartagener syndrome? What is the underlying defect?

3. What pathology fits the following high-yield phrase?


• Gout + mental retardation + lip-biting
• Lack of Gpllb/llla ■» defect in platelets ■* prolonged bleeding
• Anti-histone antibodies
• Psammoma bodies (FAI I p230) (FA12 p255)
Lytic bone lesions on x-ray

Lung Cancer
H: Chapter 89
R: Chapter 15

Lung cancer (FAI I p5l3) (FAI2 p57l)


Pancoast tumor (FAII p5l3) (FAI2 p572)

A patient of yours develops bronchogenic lung cancer but has never smoked. He is a coal
miner Exposure to what substance has put him at risk for developing lung cancer?

What complications can arise from lung cancer?

What are the 3 most common locations of lung cancer mets?

Copyright© 2012. Doctors In Training.com. LLC All Rights Reserved


Lung Infections
R: Chapter IS
H: Chapters 257, 258
Pneumonia (FAII pSI4) (FAI2 pS73)
Common causes of pneumonia (FAII pi 76) (FAI2 pi 96)
Lung abscess (FAII p5l4) (FAI2 pS73)
Pleural effusions (FAII pSI4) (FAI2 p574)

7. Which infectious agent fits the following description?


• Common cause of pneumonia in immunocompromised patients
• Most common cause of atypical / walking pneumonia
• Common causative agent for pneumonia in alcoholics
• Can cause an interstitial pneumonia in bird handlers
• Often the cause of pneumonia in a patient with a history of exposure to bats and bat droppings
• Often the cause of pneumonia in a patient who has recently visited South California, New Mexico, or
West Texas

• Pneumonia assodated with "currant jelly" sputum


• Q fever
• Associated with pneumonia acquired from air conditioners
• Most common cause of pneumonia in children I-year-old or younger
• Most common cause of pneumonia in the neonate (B-28d)
to
C • Most common cause of pneumonia in children and young adults (including college students, military
.Q recruits, and prison inmates)
tj
0*1 • Common cause of pneumonia in patients with other health problems
— • Most common cause of viral pneumonia
"O • Causes a wool-sorter's disease (a life-threatening pneumonia)
rd • Endogenous flora in 20% of adults
V; • Common bacterial cause of COPD exacerbation
U • Common pneumonia in ventilator patients and those with cystic fibrosis
kj • Pontjac fever
u
OJ0
§ 8. Pleural Effusions - Causes
—' • Transudate • Exudate
I - CHF - Pneumonia, infections, TB
^ - Cirrhosis - Cancer

£Z - Nephrotic syndrome - Uremia


- PE - Connective tissue disease
<
z Ruid overload

o
z_ J
D
Q_

[18] Copyright © 2012. Doctors In Training.com. LLC. All Rights Reserved.


sion Quiz
9. Examination of a lung at autopsy reveals a peripheral lesion with caseous necrosis. What.
the diagnosis?

10. A 30-year-old comatose man on ventilatory support in the ICU develops an infection and
dies. Autopsy reveals a pus-filled cavity in his right lung. What is the likely etiology?

A 55-year-old man who is a smoker and heavy drinker presents with a new cough and flu
like symptoms. Gram stain shows no organisms; silver stain of sputum shows gram-negati
rods. What is the diagnosis?

CXR shows collapse of middle lobe of right lung and mass in right bronchus; patient has
history of recurrent pneumonias. What is the diagnosis?

What infectious agent is the cause of pneumonia based on the following lab test?
• Gram (+) cocci in clusters
• Gram (+) cocci in pairs
• Gram (-) rods in 80-year-old
• Gram (+) coed in neonate
• Gram (-) rods in neonate

Copyright © 2012. Doctors In Training.com. LLC. All Rights Reserved.


( FA I I p Q 4 ) ( FA Q p l 3 l ) ( l

What adult structures are derived from the 3rd, 4th, and 6th aortic arches?
(FAII pQ7) (FAQ pl35)

What cell wall inhibitor matches the following statement?


(FAII pi85-187) (FAQ p205-207) (GG pl497)
• Next step in treatment of otitis media if resistant to amoxicillin
• Prophylaxis against bacterial endocarditis
• Increases the nephrotoxicity of aminoglycosides
• Sufficient for the treatment of syphilis
Single dose treatment for gonorrhea

Overview and Ventricle Development


R: Chapter 12
H: Chapter 224
P: Chapters 23, 83
Heart embryology (FAII pl23) (FAI2 pl30) (R p538, Fig. I2-3)(H pl809)
Truncus arteriosus (FAII pl23) (FAI2 pl30)
Interventricular septum development (FAII pl23) (FAI2 pl30) (R p540. Fig. 12-4) (H pl923)

4. Outline the pathway by which the heart tube forms the atria of the four-chambered heart.
Tube grows, elongates, and folds into an S-shape ■» atrial chamber lays posteriorly in S, and ventricular
chamber lays anteriorly in S ■» atrial chamber grows and incorporates superior vena cava and
pulmonary vein ■▶ septum primum forms ■*• septum secundum forms incompletely (leaving a hole
called foramen ovale) and cell death in septum primum forms a hole called ostium secundum

5. What divides the right and left atria?


Septum primum and septum secundum

6. How is blood shunted from the right atrium to the left atrium in an embryo?
Through the foramen ovale (of the septum secundum) and ostium secundum (of the septum primum)

7. Outline the pathway by which the ventricles and their outflow tracts are separated.
• Ventricular chamber lays anteriorly in the S-shaped heart tube ■» muscular ventricular septum forms
which begins to divide the ventricles
• Truncoconical swellings (ridges) of the truncus arteriosus meet, fuse, and zip (both superiorly and
inferiorly) in a 180 degree turn to form the spiral septum (AKA aorticopulmonary septum)
• Inferior portion of spiral septum meets with muscular ventricular septum to divide the ventricles and
form the aorta and pulmonary arteries

8. Describe how the ventricles are remodeled in order to form the atrioventricular valves, q~
• Myocardium erodes ■* ventricles enlarge as a result ■> residual mesodermal tissue becomes fibrous 0^9
and forms chordae tendineae
• Formation of papillary muscles and AV valves

Copyright @ 2012. Doctors In Training.com. LLC. All Rights Reserved. L


9. Name 6 different truncoconical (spiral) septum defects.
• Fenestrae
• Ventricular septal defect (VSD)
• Tetralogy of Fallot
• Persistent truncus arteriosus
• Transposition of the great vessels (RV ■» aorta, LV"» PA)
• Dextrocardia

Which embryologic structure of the heart gives rise to the following aduft structure?
• Ascending aorta and pulmonary trunk
• Coronary sinus
• SVC
• Smooth parts of the left and right ventricle
• Smooth part of the right atrium
• Trabeculated left and right atrium
• Trabeculated parts of the left and right ventricle

What structure divides the truncus arteriosus into the aortic and pulmonary trunks? What is
the cellular origin of this structure?

Atrial Development and Fetal Circulation


H: Chapter 224
>> P: Chapters 23, 83
CxO F
O Interatrial septum development (FAII pl24) (FAI2 pl3l)
O Fetal circulation (FAII pi25) (FA 12 pi32)
>^
_q Q. What are the 3 possible causes of an atrial septal defect?

, 13. What structure grows to close the opening/canal between the atrial chamber and
ventricular chamber into two smaller openings?
y

<C 14. What genetic abnormality is commonly associated with endocardial cushion defects?

essio
Which fetal vessel has the highest oxygenation?

What causes the ductus arteriosus to dose? (FAI I p 125) (FAQ p 132)

Q5)(FAI2pl32)

Copyright© 2012. Doctors In Training.com. LLC. All Rights Reserved.


What germ layer gives rise to the following adult structures? (FAI I pi 19) (FAQ pi26)
• Retina
• Salivary glands
• Pancreas
• Muscles of the abdominal wall
• Thymus
• Spleen
• Aorticopulmonary septum
• Anterior pituitary
• Posterior pituitary
• Bones of the skull
• Cranial nerves

What are the clinical uses for 1st, 2nd, 3rd, and 4th generation cephalosporins?
(FA pi86) (FAQp 206) (GG pl497)

What are the classic symptoms of carcinoid syndrome?


(FAI I p302) (FAQ p33l) (R p787) (H p3062)

Right-to-Left Shunts
R: Chapter 12
H: Chapter 236

Congenital heart disease (FAII p267) (FAI2 p 292)


Eisenmenger syndrome (FAII p267) (FAI2 p 292) (R p540)
Tetralogy of Fallot (FAII p267) (FAI2 p 292) (R p542) (H pl926)
D-transposition of great vessels (FAII p268) (FAI2 p 293) (R p543) (H pl927)

What are the components of the tetralogy of Fallot? (FAI I p267) (FAQ p292)

Explain how the great vessels are attached in a transposition of the great vessels.
(FAII p268) (FAQp293)

Copyright ..) 2012. Doctors In TrAining.com. LLC All Rights Reserved.


Left-to-Right Shunts and Other Anomalies
R: Chapter 12
H: Chapter 238
VSD (R p54l)
ASD (R p54l)
Patent ductus arteriosus (FAII p268) (FAI2 p 293) (R p54l)
Coarctation of the aorta (FAII p268) (FA12 p 293) (R p544)
Congenital cardiac defect associations (FAII p269) (FAI2 p 294)

7. Ebstein's anomaly
• Associated with maternal lithium use
• Tricuspid leaflets are displaced into right ventricle, hypoplastic right ventricle, tricuspid regurg or stenosis
• 80% have a patent foramen ovale with a R to L shunt
• Dilated right atrium causing increased risk of SVT and WPW
• Physical exam: widely split S2, tricuspid regurgitation

A 45-year-old male presents with a BP of 160/90 on the right arm and 170/92 on the left
arm. There are no palpable pulses in the feet/ankle. What problem does this patient most
likely have?

Describe blood flow thru a PDA.

What heart defect is associated with the following disorder? (FA p269) (FAQ p 294)
• Chromosome 22q 11 deletions
• Down syndrome
• Congenital rubella
• Turner's syndrome
• Marian's syndrome

What problems are offspring of diabetic mothers at higher risk for?

Copyright © 2012. Doctors In Trnning.com. LLC. All Rights Reserved.


Question Warm-Up
I. What irreversible enzymes are involved in gluconeogenesis?
(FA plOI) (FAQ pi07) (Phys p8l6)

What cranial nerves innervate the tongue in the following ways?


(FAI I pi29) (FAQ pl37) (COA p94l)
• Taste in the anterior 2/3
• Taste in posterior 1/3 (main innervation)
• Motor
• Sensation in the anterior 2/3
• Sensation in the posterior 1/3

Which tumors arise centrally in the lung and are linked to smoking? Which tumors arise
peripherally in the lung and are less linked to smoking (if at all)? (R p72l)

Cardiac Output Variables


Phys: Chapters 20, 21
Cardiac output (FAII p254) (FA/2 p28l)
Cardiac output variables (FAII p255) (FAI2 p282)
Cardiac drugs: sites of action (FAII p283) (FAI 2 p308)
Preload and afterload (FAII p255) (FAI2 p282)

4. What is the pulse pressure in a patient with systolic BP of 150 and a MAP of 90?

5. What is the basic equation for cardiac output? What is the Fick prindple?

What medications affect a heart's contractility? (FAI I p255) (FAQ p282)

,at ionic changes affect a heart's contractility? (FAI I p255) (FAQ p282)

How can the myocardial oxygen demand be decreased in circumstances where the he;
ischemic? (FA p255) (FAQ p282)

Copyright © 2012. Doctors In Training.com. LLC. All Rights Reserved.


Charting Cardiac Output
Phys: Chapters 9, 20, 21

Starling curve (FAII p255) (FAI2 p28l) (Phys pi 10)


Ejection fraction (FAII p255) (FAI2 p282) (Phys 106)
Resistance, pressure, flow (FAII p256) (FA12 p282) (Phys 158)
Cardiac and vascular function curves (FAII p256) (FAI2 p282) (Phys. pl68, 15-1)

What factors affect stroke volume?

What is the heart ejection fraction?

Copyright fc) 2012. Doctors In Training com, LLC. All Rights Reserved.
arm-
I. What is the most common type of TE fistula? (FAI I pl3l) (FAQ pi39) (R p765)

What are the major regulatory enzymes of citric acid cycle?


(FAI I p96) (FAQ pi02) (Phys p8l3)

What infections are caused by Chlamydia species? What is the treatment for most
Chlamydia infections? (FAI I pl56) (FAQ p 169) (GG pl529)

Heart Failure Pathophysiology


Phys: Chapter 22
R: Chapter 14
H: Chapter 234
Normal pressures (FAII p266) (FAI2 p29l)
CHF(FAII p274) (FAI2 p300) (R pS33)
Renin-angiotensin-aldosterone system (FAII p462) (FAI2 p5IO) (Phys p260)

4. CHF Compensation
. l.V contractility! I—▶ 4, Cardiac output -fc? Sympathetic _w ^LV contractility
\w^ ^ activation ^

■f Renal Na' 1* Systemic


*1* Renin-angiotensin-_ venous pressure,
aldosterone and H2O
reabsorption ■^preload

Peripheral edema

Pulmonary venous g» 4. rv output


congestion

5. What are the signs of right-sided heart failure? (FAI I p274) (FAQ p300)

What are the signs of left-sided heart failure? (Phys p259)

7. How does poor cardiac output result in an increase in aldosterone?


(FAI I p274,462) (FAQ p300,5I0) (Phys 260)

Copyright © 20I2. Doctors In Training.com. LLC. All Rights Reserved.


Heart Failure Medications
GG: Chapters 27, 28
H: Chapter 234
CHF medications
- Chronic treatment (FAI2 p300)
- Acute treatment
- Nesiritide (FA 12 p309) (GG p696)
Cardiac glycosides (FAII p283) (FAI2 p309) (Phys p259) (GG p838)

8. Cardiac Glycosides (Digoxin)

Ion Exchange

• , ■ 1.
Free Ca' fi t

What medications are used to treat chronic heart failure?

medications are used to treat acute heart failure?

What is the mechanism of action of the cardiac glycosides (digoxin, digftoxin)?

Copyright J.) 2012. Doctors In Training.com, LLC. All Rights Reserved.


Question Warm-U
Which Rickettsial species has properties unique from the other Rickettsial organisms.
What are those unique properties? (FA p 154-155) (FAQ pi 67-168) (R p7l4)

What is the classic presentation of congenital pyloric stenosis?


(FAII pl3l) (FAQpl39)(Rp766)

Explain why a deficiency of the enzyme that is the rate-limiter for the HMP shunt can
result in hemolytic anemia. (FA pi02) (FAQ pi07) (Phys 816-817, Fig. 67-8)

Capillaries and Edema


Phys: Chapters 16, 25
R: Chapter 11

Capillary fluid exchange (FAII p266) (FAI2 p29l) (Phys pi84) (R pi 11-113)

4. Illustrate the Starling forces affecting capillaries. (Phys pi84)

How do the following circumstances impact the Starling forces of fluid movement throu
capillaries? (FAI I p266) (FAQ p291)
• Heart failure
• Liver failure
• Renal failure
• Infections and toxins
• Nephrotic syndrome
• Lymphatic blockage
• Bums
• Diuretic administration
• IV infusion of albumin or clotting factors
Venous insufficiency

Copyright; 3 2012. Doctors In Training.com. LLC. All Rights Reserved.


Capillaries and Shock
Phys: Chapter 24
R: Chapter 4
Shock (FAII p224) (FAI2 p249) (R pl29-l32)
Central lines (H: 2218)
Femoral region (FAII p3l4) (FApl2 p343) (COA p204, Fig 2.15)

6. How do SVR and CO change in the following types of shock? (FAI I p224) (FAQ p245)
SVR CO Rx
Hypovolemia
Heart failure

Sepsis/anaphylaxis
Neurogenic

7. Locations of central lines:


• The femoral is the easiest site with the least risk; however, it cannot stay in place more than 5-7 days
due to risk of infection.
• The subclavian (SC) is preferred by some because it is easy to find, can remain longer (3-4 weeks), and
is not as uncomfortable to the patient; however, it has the highest risk of pneumothorax. Not such a
great choice in those with COPD (barrel-chested) or lung tumors.
• The internal jugular (IJ) is a good choice because there are good landmarks, can remain a long time (3-4
weeks); however, there it can be a little more uncomfortable to the pt, and there is a risk of puncturing
the carotid or causing pneumothorax. Do not place a Left IJ without discussing it with staff, due to a
greater risk of perforating the left SC vein due the angle at which the left IJ and left SC meet.
■ catheter preferred sites: right IJ > right SC > left SC > left IJ

8. How does the vascular resistance and stroke volume differ in hypovolemic shock compared
to septic shock?
Vascular resistance
Hypovolemic shock
Septic shock

What are some of the causes of cardiogenic shoe OTCTiPi^l

. How is the skin of a patient different in cardiogenic shock compared to septic shock?

What other severe problems often coexist with septic shock in ICU patients?

What serum lab marker is helpful in assessing the appropriate oxygenation of tissues?

Copyright © 2012. Doctors In Training.com, LLC. All Rights Reserved.


What is the primary energy source in a patient that has not eaten in two days?
(FAII pi 13) (FAQ pi 18)

When placing a femoral venous catheter, while palpating the pulsatile femoral artery,
where is the femoral catheter placed in relation to the artery?
(FAI I p3l4) (FApQ p344) (COA p204, Fig 2.15)

1. Where in the cell would you find the following enzymatic processes taking place?

• Fatty acid degradation


• Fatty acid synthesis
• Glycolysis
• TCA cycle
I (FAII
• Electron
p95)transport
(FAQ pi00)
chain (oxidative phosphorylation)
• Gluconeogenesis
• Protein synthesis (RER)

K»•Urea
Steroid synthesis (SER)
cycle
Heme synthesis

Atria and Ventricles


Phys: Chapters 9, 20
Cardiac Cycle (FAII p257) (FAI2 p284) (Phys pl08, Fig. 9.8)
- Pressure volume loop (Phys pi06-109)
- Pressure!time: aorta, LV, LA (Phys pi 10-111)

When does isovolumetric contraction take place?

How does an increase in afterioad affect the stroke volume of the heart assuming
contractility remains the same? (FAI I p257) (FAQ p284)

What impact does an increase in contractility have on stroke volume assuming preloi
and afterioad remain constant? (FAI I p257) (FAQ p284)

Heart Sounds and Jugular Waves


Phys: Chapter 9
Cardiac cycle (FAII p257) (FAI2 p284)
- Heart sounds
- Splitting (FAII p258) (FAI2 p284)
- jugular pulse
-ECG

Copyright (<•' 2012. Doctors In Training.com. LLC. All Rights Reserved.


What protozoal/helminth matches the following statement?
(FAII pi60-163) (FAI2pl75-l76) (Rp336)
• Contracted by eating undercooked fish and causes an inflammation of the biliary tract
• Most common protozoal infection in US
• Cause of Chagas disease
• Most common helminthic infection in the US
• Snail host, "swimmers itch"
• Diarrhea in campers and hikers
• Transmitted in raw meat or infected cat feces

What effect does stress have on adipocytes? (Phys p929)

.at are the 4 obligate aerobic bacteria? (FAI I pl38) (FAQ pi49)

Systolic Murmurs
H: Chapter 227
Phys: Chapter 23
Auscultation of the heart (FAII p258) (FAI 2 p284) (Phys p26S, Fig. 23-2, 23-3)
Heart murmurs (FAII p259) (FAI2 p285) (Phys p267)

4. What heart sounds are considered benign when there is no evidence of disease?
• Split SI
• Split S2 on inspiration
• S3 heart sound in a patient <40yo
• Early, quiet systolic murmur

5. What are the most common causes of aortic valvular stenosis?

Copyright © 2012. Doctors In Training.com, LLC. All Rights Reserved.


Diastolic Murmurs
H: Chapter 227
Phys: Chapter 23

Crescendo-decrescendo systolic murmur


best heard in the 2nd-3rd right interspace
close to the sternum

Early diastolic decrescendo murmur heard


best along the left side of the sternum

Late diastolic decrescendo murmur heard


best along the left side of the sternum

Pansystolic (AKA holosystolic or uniform)


murmur best heard at the apex and often
radiates to the left axilla

Late systolic murmur usually preceded by


a mid-systolic click

Crescendo-decrescendo systolic murmur


best heard in the 2nd-3rd left interspaces
close to the sternum

Pansystolic (AKA holosystolic or uniform)


murmur best heard along the left lower
sternal border and generally radiates to
the right lower sternal border

Rumbling late diastolic murmur with an


opening snap

Pansystolic (AKA holosystolic or uniform)


murmur best heard at the 4th-6th left
intercostal spaces

Continuous machine-like murmur (in


systole and diastole)

High-pitched diastolic murmur associated


with a widened pulse pressure

10. Know the classic descriptions of heart murmurs. What is the murmur heard best in left
lateral decubitus position?

An 80-year-old man presents with a systolic crescendo-decrescendo murmur: What,


the most likely cause?

Copyright (5)2012. Doctors In Training.com. LLC. All Rights Reserved.


arm-
Why do the kidneys retain fluid in heart failure patients?
(FAI I p464) (FAQ P5I2) (Phys p259)

Which hereditary hyperbilirubinemia matches the following statement?


(FAI I p333) (FAQ p363) (R p84l-842)
- Mildly decreased UDPGT

Completely absent UDPGT


- Grossly black liver
Responds to phenobarbital
- Treatment includes plasmapheresis and phototherapy
• Asymptomatic unless under physical stress (alcohol, infection)

What is a disulfiram-like reaction? What drugs cause a disulfiram-like reaction?


(FAI I p94) (FAQ plOO) (GG p643-644)

Action Potentials
Phys: Chapters 9, 10
H: Chapters 228, 258
GG: Chapter 29
Cardiac myocyte physiology (FAII p260) (FAI2 p288) (Phys pl02, Fig. 9-5)
Ventricular action potential (FAII p260) (FAI2 p286) (Phys pi 15, 118)
Pacemaker action potential (FAII p26l) (FAI2 p287) (Phys pll5-U9)

Copyright © 2012. Doctors In Trainmg.com. LLC. All Rights Reserved.


Antiarrhythmics
GG: Chapter 29
Na+ channel blockers (FAII p284) (FAI2 p3ll) (GG p840)
^-blockers (FAII p285) (FAI2 p3ll) (GGp830)
K1" channel blockers (FAII p285) (FA/2 p3ll) (GG p843)
Ca2* channel blockers (FAII p285) (FAR p3l2) (GGp830)
Other antiarrhythmics (FAII p286) (FAI2 p3l2) (GG p842)

What is the mechanism of action of each class of anti

Which antiarrhythmic has the side effect of cinchonism? (FAI I p284) (FAQ p3l I)

What are the potential side effects of amiodarone use? (FAI I p285) (FAQ p3l I)

To which dass of antiarrhythmics does each of the following agents belong?


(FAII p284-286)(FAI2p309-3Q)
• Sotalol
• Propranolol
• Bretylium
• Quinidine
• Verapamil
• Procainamide
• Lidocaine
• Diltiazem

Copyright © 2012. Doctors In Training.com, LLC. All Rights Reserved.


Identify the hepatitis B status of the following patients based on their hepatitis B serol
markers. (FAII pl72) (FAQ pl9l) (Rp843)

HepBsAg HepBsAb HepBcAb Status


Negative Positive Positive

Negative Negative Positive


Positive Negative Positive IgM
Positive Negative Positive IgG
Negative Positive Negative

With what type of congenital heart defect would increasing afterioad be beneficial?
(FAI I p267-268) (FAQ p292-293)

What drugs are known for inhibiting cytochrome P450?


(FAI I p245) (FAQ p273) (GG pQ9)

Atria to AV conduction
Phys: Chapter 10
Phys: Chapter 11
H: Chapter 232

Electrocardiogram (FAII p262) (FAI2 p288)


Axis
ECG tracings (FAII p263-264) (FAI2 p289-290)
- Atrial fibrillation (H pi881)
- Atrial flutter (H pi885)
Problems with the AV conduction (H pl888)
- AV block
- Mobitz type I
- Mobitz type II
- Third degree
Wolff-Parkinson-White syndrome (FAII p263) (FAI2 p289)

Copyright © 2012. Doctors In Traming.com. LLC. All Rights Reserved.


4. ECG Axis

Common conditions that result in


axis deviation:
Left axis deviation
. Inferior wall myocardial infarction
• Left anterior fascicular block
-i5o« aVR aVL -30= . Left ventricular hypertrophy
(sometimes)
. LBBB
. High Diaphragm

Right axis deviation


. Right ventricular hypertrophy
• Acute right heart strain
(ex. massive pulmonary embolism)
. Left posterior fascicular block
. RBBB
. Dextrocardia

II +60°
+90° aVF
The net electrical signal (cardiac axis) will
fall within the shaded region in normal
'2£o» cardiac physiology.

Sinus Bradycardia

B5»HMB«
■M 1 1 M B M B B SPIBIMIWIM^I;
Normal P wave BPM < 60
Normal QRS complexes

Paroxysmal Supraventricular Tachycardia

Copyright is) 2012. Doctors In Traimng.com. LLC. All Rights Reserved.


n
5. What are the two different types of second degree AV block? How do they differ?

6. How does the cause of a narrow QRS complex differ from the cause of a wide QRS
complex?

7. What is the ECG axis given the QRS deflections in the following scenarios?
• Positive in lead I, positive in lead II
• Positive in lead I, negative in lead III
■ Negative in lead I, positive in lead III
- Positive in lead I, negative in aVR

.. Why is warfarin anticoagulation important in patients with chronic atrial fibrillation?


(FAII p263) (FAQp289)

Ventricular Arrhythmias
Phys: Chapter 23
H: Chapter 23
Ventricular arrhythmias (H pi890)
Torsades de pointes (FAII p262) (FAI2 p288) (H pl89l, Fig 233-9)
Ventricular fibrillation (Phys pl49-l5l)

9. Drugs that prolong the QT interval


• Anti-infection: macrolides, chloroquine
• Antipsychotics: haloperidol, risperidone
• Anti-HIV: HIV protease inhibitors (-navirs)
• Anti-arrhythmics: class IA (quinidine) and class III (K1' channel blockers such as sotalol)
Ventricular Premature Beats ,

Monomorphic VentrkularTachycardia

Ventricular Fibrillation

Copyright© 2012. Doctors In Training.com. LLC. All Rights Reserved.


What is the treatment for ventricular fibrillation? (FAI I p264) (FAQ p289)

What is the treatment for ventricular tachycardia when there is no pulse?

What is the hallmark of athird degree heart block? (FAI I p264) (FAQ p289)

What drugs are known to prolong the QT interval, increasing the likelihood of torsades in
those at risk?

Copyright © 2012. Doctors In Traming.com. LLC All Rights Reserved.


What is the difference between primary biliary cirrhosis and primary sclerosing cholangitis?
(FAI I p334) (FAQ p365) (R p867, 869)

. ,ow does increasing the diameter of a vessel by two times affect the resistance of the
vessel? (FAI I p256) (FAQ p282) (Phys pi64)

3. What substances are known teratogens? List as many as you can recall.
(FAI I pQO) (FAQ pQ7) (R p450, Table 10-2)

Physiology of BP Regulation
Phys: Chapters 18, 19,20
Maintenance of mean arterial pressure (FAII p265)
Baroreceptors and chemoreceptors (FAII p265) (FA/2 p290)
Physiology of vasoconstriction
- Smooth muscle contraction (FAII p378) (FAI2 p4l3)
Review of organs involved in BP regulation (Phys p204, 213, 220)

4. Physiology of Vasoconstriction

•Sildenafil
* bradykinin (Viagra) Smooth muscle cell
*ACh
*alpha-2-agonist cGMP phosphodiesterase
•histamine
•serotonin myosin
* shear stress
activated myosin
phosphatase
myosin-P04 actln > contraction
ased intracellular
Icium activates myosin light-
Guanylyl Cyclase chain kinase
L-arginine
myosin

calmodulin-Ca cAfV
dtrulline
and NO
L-arginine
Endothelial cell
* Nitrates * Ca-channel * Epinephrine LVj
blockers » Prostaglandin E2

Copyright © 2012. Doctors In Training.com. LLC. All Rights Reserved.


5. Organs involved in BP regulation

Organ or Tissue Mediator


Atria ANP
Aorta Aortic baroreceptor
Carotid sinus Carotid body baroreceptor
Medulla Control of sympathetic NS
Adrenal glands - zona glomerulosa Aldosterone
Adrenal glands - zona fasciculata Cortisol
Adrenal glands - medullary Epinephrine and NE
chromaffin cells
Liver Angiotensinogen
JGA of kidneys Renin (angiotensinogen ■» angiotensin I)
Lungs ACE (angiotensin I ■* angiotensin II)
Kidney glomerulus GFR impacted by multiple variables
Kidney tubules Impacted by angiotensin II, ADH, aldosterone
Local tissue effectors NO, Prostaglandins, serotonin, histamine, bradykinin,
adenosine, acidosis, lactate, O2, CO2, K1"

6. Outline the mechanism by which the kidneys regulate BP.

.. .at substances act on smooth muscle myosin light-chain kinase? How does this effect
blood pressure?

Describe the chain of events in which hypotension causes a reflex tachycardia.

Copyright © 2012. Doctors In Training.com. LLC. All Rights Reserved.


Hypertension
H: Chapter 247
Phys: Chapter 19

Circulation through organs (FAII p265) (FAI2 p29l)


Autoregulation (FAII p266) (FAI2 p29l)
Hypertension (FAII p269) (FAI2 p294)
Left-sided hypertensive heart disease (R p559)
Aortic dissection (FAII p270) (FA/2 p295)

Know the current standards for HTN for Step I:

http://www. nhlbi. nih.gov/guidelines/hypertension/phycard. pdf

Session
What BP values mark the diagnosis of hypertension? What values
(FAII p269) (FAQ p294)

What would you most suspect the cause of hypertension to be in a patient with the foil
clinical dues?
• Paroxysms of increased sympathetic tone: anxiety, palpitations, diaphoresis
• Age of onset between 20 and 50
• Elevated serum creatinine and abnormal urinalysis
• Abdominal bruit
• BP in arms > legs
• Family history of HTN
• Tachycardia, heat intolerance, diarrhea
• Hyperkalemia
• Episodic sweating and tachycardia
• Abrupt onset in a patient younger than 20 or older than 50, and depressed serum K*" levels
• Central obesity, moon-shaped face, hirsutism
• Normal urinalysis and normal serum K4" levels
• Young individual with acute onset tachycardia

Hypokalemia
- Proteinuria

11. What chest x-ray finding is a possible sign for aortic dissection? (FAI I p270) (FAQ p295)

What category of BP medications is preferred in the treatment of aortic dissection?

Copyright © 2012. Doctors In Training.com. LLC. All Rights Reserved.


Which group of medications inhibits the rate limiting enzyme of cholesterol synthesis?
(FAI I pi 13) (FAQ plOI, 119) (Phys p827-829)

What are the primary mechanisms of action of the different classes of antiarrhythmics?
• Class I
• Class II
• Class III
• Class IV

Name 8 different indirect cholinergic agonists and state the use for each.
(FAII p238) (FAQ p265)

Antihypertensive Agents
GG: Chapters 25, 26, 27
ACE inhibitors (FAII p475) (FAI2 p527)
CO Hydralazine (FAII p280) (FAI2 p306)
<D Minoxidil (GG p78l)
. ^ Calcium channel blockers (FAII p280) (FAI2 p306)
Nitroglycerin, isosorbide dinitrate (FAII p280) (FAI2 p306)

■£ 4 . M i n o x i d i l
^ • Mechanism of action: opens potassium channels and hyperpolarizes smooth muscle, resulting in
>>, relaxation of vascular smooth muscle
• Therapeutic use: severe hypertension (topical application for hair loss)
• Toxicity: hypertrichosis, hypotension, reflex tachycardia, fluid retention/edema

' 5. Calcium Channel Blockers (CCB)


• Dihydropyridine CCB: nifedipine, amlodipine, felodipine, nicardipine, nisoldipine
- Mechanism of action: act on vascular smooth muscle to cause vasodilation
- Therapeutic use: hypertension, angina, vasospasm (Prinzmetal's angina, Raynaud phenomenon)
esophageal spasm, migraine prophylaxis
I s - Toxicity: peripheral edema, flushing, dizziness, constipation, reflex tachycardia
) • Non-dihydropyridine CCB: verapamil, diltiazem
- Mechanism of action: block calcium-channels at pacemaker cells (FAII p286) (FAI2 p3l2)
. - Therapeutic use: hypertension, angina, arrhythmias
- Toxicity: cardiac depression, AV-block, flushing, dizziness, constipation

Copyright © 2012. Doctors In Training.com. LLC. All Rights Reserved.


[•JTTrJ

6. Which antihypertensive class or drug fits the following side effect?


First dose orthostatic hypotension
Ototoxic (especially with aminoglycosides) |
j Hypertrichosis
Cyanide toxicity !
Dry mouth, sedation, severe rebound HTN j
Bradycardia, impotence, asthma exacerbation
j R e fl e x tachycardia |
| Cough |
Avoid in patients with sulfa allergy

j Possible angioedema j
j Possible development of drug-induced lupus j
Hypercalcemia, hypokalemia

l While on an ACE inhibitor, a patient develops a cough. What is a good replaceme


drug, and why doesn't it have the same side effects? (Phys p520)

Which calcium channel blockers can be used to treat both hypertension and
tachyarrhythmia?

Copyright'.-) 2012. Doctors In Training.com. LLC. All Rights Reserved.


Choice of Antihypertensive
GG: Chapters 25,26, 27
Phys: Chapter 19
Antihypertensive therapy (FAII p280) (FAI2 p306) (GGp772-784)
Malignant hypertension treatment (FAII p280) (FAI2 p306) (GG p73)

9. For each comorbid condition, what antihypertensives would be considered first, and
which antihypertensives should be avoided (either used with caution, or absolutely
contraindicated)? (H p2054, Table 247-8)
Hypertension plus... Initial Therapy Options Avoid
CHF • Diuretics • P-blocker (in acute
> ACE inhibitor/ARB decompensated CHF or
• P-blocker cardiogenic shock)
• Aldo antagonist • CCB

DM • ACE inhibitor/ARB
P-blocker
• Thiazide
Post-MI/CAD • Thiazide
' P-blocker
» ACE inhibitor/ARB
• CCB (as needed for angina)
• Nitrates (as needed for
angina)
C/) Atrial fibrillation Diltiazem/verapamil (for rate

I
CO Bradycardia
control)
• Diltiazem/verapamil
C • P-blocker
CD
Renal insufficiency ACE inhibitor/ARB (for • ACE inhibitor/ARB (may
(U proteinuria) increase creatinine)
Ql • K*"-sparing diuretics
Renal artery stenosis ACE inhibitor/ARB
c BPH Ct-blocker
<
Hyperthyroidism Propranolol
Hyperparathyroidism Loop diuretic Thiazide
Osteoporosis Thiazide
or: Gout Thiazide
• Hydralazine ACE inhibitor/ARB
Pregnancy
• Methyldopa
D
U • Labetalol
CO • Dihydropyridine CCB
• CCB
Migraines
o Essential tremor
• P-blocker

Q Propranolol

<
U

[26] Copyright © 2012. Doctors In Training.com, LLC. All Rights Reserved.


Which antihypertensives are particularly benefidal to heart failure patients?
(FAII p280) (FAQp306)

What categories of antihypertensives are considered first line in hypertensive patients wh


diabetes mellitus? (FAI I p280) (FAI2 p306)

Which antihypertensives are safe to use in pregnancy?

A 25-year-old pregnant woman in her 3rd trimester has a normal BP when standing and
sitting. When supine, her BP drops to 90/50. What is the diagnosis?

Copyright g! 2012. Doctors In Training.com. LLC. All Rights Reserved


regurgitation. (FAI I p259) (FAI2 p285)

Which 2 bacteria are well known for being obligate intracellular bacteria? Why can't
these bacteria replicate extracellularly? (FAI I pl39) (FAQ pi50)

What marker of inflammation produced by the liver and within atherosclerotic plaques i
a strong predictor of Ml risk?

Atherosclerosis and Coronary Artery Disease (CAD)


R: Chapters II, 12
H: Chapters 243, 244, 245

Arteriosclerosis (FAII p269-270) (FAI2 p294) (R p496)


Coronary arteries (FAII p254) (FAI2 p280) (R p547. Fig. 12-12)
Ischemic heart disease (FAII p270) (FAI2 p295) (R p545)
Causes of chest pain (R p546)

Coronary Arteries
Anterior View Posterior View

Lippincott Williams & Wilkins Atlas of Anatomy ©2008 Wolters Kluwer Health. Inc. All rights reserved.

Copyright ©2012. Doctors In Training.com. LLC. All Rights Reserved.


4. Pathogenesis of Atherosclerosis (R p498-504)
• Endothelial injury leads to increased vascular permeability, leukocyte adhesion and thrombosis
• Accumulation of lipoproteins: occurs in the vessel wall and is mostly LDL
• Monocyte adhesion to the endothelium: subsequent migration of the monocytes into the intima and
then transformation of these cells into macrophages and foam cells.
• Platelet adhesion
• Factor release: activated platelets, macrophages and vascular wall cells; induces smooth muscle
recruitment
• Smooth muscle cell proliferation and ECM (extracellular matrix) production
• Lipid accumulation: occurs extracellularly and within macrophages and smooth muscle cells

5. Complications of Atherosclerosis Based on Location of Plaque (R p504-507)


• Smaller vessels can become occluded and then compromise distal tissue perfusion
• Ruptured plaque can embolize atherosclerotic debris and cause distal vessel obstruction or can lead to
acute vascular thrombosis (stroke)
• Destruction of the underlying vessel wall can lead to aneurysm formation, with secondary rupture and
thrombosis

6. Abdominal Aortic Aneurysm (R p507)


• Caused by atherosclerotic plaque compressing the underlying media
• Nutrient and waste diffusion is compromised
• Media degenerates and necroses, leading to arterial wall weakness
• More common in men over 50 and in smokers
• Presents as a pulsating mass in the abdomen s-\
• Consequences include: rupture leading to fatal hemorrhage, embolism from atheroma, obstruction of a *^
branch vessel and impingement on an adjacent structure (ureter) -rj
D
7. Deadly Causes of Acute Chest Pain Q
• Aortic dissection (or dissecting aortic aneurysm) <C
• Unstable angina ^
• Ml O
• Te n s i o n pneumothorax C_

70
What is the most likely cause of chest pain in the following scenarios?
• ST segment elevation only during brief episodes of chest pain to
i
• Patient is able to point to localize the chest pain using one finger ^
• Chest wall tenderness on palpation c*
• Rapid onset sharp chest pain that radiates to the scapula fD
• Rapid onset sharp pain in a 20-year-old and assodated with dyspnea O
• Occurs after heavy meals and improved by antacids $
• Sharp pain lasting hours-days and is somewhat relieved by sitting forward (D
• Pain made worse by deep breathing and/or motion O
• Chest pain in a dermatomal distribution r^'
• Most common cause of non-cardiac chest pain

Copyright © 2012. Doctors In Training.com. LLC. All Rights Reserved. [29]


What are the most common locations for atherosclerosis, and what disorders result from
plaques in these locations?

A patient with poorly-managed HTN has acute, sharp substernal pain that radiates to the
back and progresses over a few hours. Death occurs in a few hours. Diagnosis?

During a high school football game, a young athlete collapses and dies immediately. What
type of cardiac disease did he have?

Antianginal Therapy and Lipid-Lowering Agents


GG: Chapters 27, 31

Antianginal therapy (FAII p28l) (FAI2 p307) (GG p76l)


Lipid-lowering agents (FAII p282) (FAI2 p308) (GG p884)

essio
Which lipid-lowering agent matches the following description? (FAI I p282) (FAQ p307-308)
1 SE: facial flushing
• SE: elevated LFTs, myositis
• SE: Gl discomfort, bad taste
' Best effect on HDL
Best effect on triglycerides/VLDL
Best effect on LDL/cholesterol
• Binds C diff. toxin

13. A 50-year-old man starts on lipid-lowering medication. Upon his first dose, he develops a
rash, pruritus, and diarrhea. What drug is he taking?

\. How can the flushing reaction of niadn be prevented?

Copyright © 2012. Doctors In Training.com. LLC All Rights Reserved.


Histamine
Porphyrin, heme
NO
GABA (a neurotransmitter)
S-adenosyl-methionine (SAM)
Creatine

What is the basic equation for cardiac output? What is the Fick principle? (FAI I p254)
(FAI2 p28l)

What is the mechanism of action of lactulose? (FAI I pi05) (FA12 pi 11)

Ml Pathophysiology
R: Chapter 12
H: Chapters 243-245

Evolution of an Ml (FAII p27l) (FAI2 p297)


Serum diagnosis of an Ml (FAII p272) (FAI2 p298)
Type of infarcts (FAII p272) (FAI2 p298)

4. Evolution of an Ml
Timeframe Gross Features Microscopic Features Risks and Other Findings

0-4 hrs

4-24 hrs

1-3 days

3-10 days

10 days-8 wks

> 2 months

Copyright (;•) 2012. Doctors In Training.com. LLC. All Rights Reserved.


(FAQ p298)

Which coronary artery is most commonly occluded in an Ml? (FAI I p27l) (FAQ p297)

Describe the evolution of an Ml.


Timeframe | Gross Features | Microscopic Features | Risks and Other Findings

Copyright Gl 2012, Doctors In Training.com. LLC. All Rights Reserved.


Diagnosis and Treatment of Ml
H: Chapters 243-245
GG: Chapters 25, 26, 27
ECG diagnosis of Ml (FAII p272) (FAI2 p298)
Ml complications (FAII p272) (FAI2 p298)
Ml treatment

8. Evolution of Myocardial Infarction


i r -i 1 r~

Normal Acute Hours Days later Weeks later

4^.

ST elevates
P
ST elevated T wave inverts ST normal ST normal
R wave decreases Q wave deepens T wave inverted T wave normal
Q wave appears Q wave persists

9. EKG Changes with Ml (Always obtain a previous EKG for comparison!)


• ST segment elevation of at least I mm in 2 contiguous leads
• T wave inversion
• NewLBBB
• New Q waves (at least I block wide or 1/3 height of the total QRS complex)

10. Arteries - Walls - EKG Leads


Artery Wall Perfused EKG Leads
Left anterior descending Anterior wall V1-V4, V5
Left circumflex Lateral wall aVL, V5, V6 n
Right coronary Inferior wall* II, III, aVF >
Right coronary Posterior wall R precordial EKG: V4
g
* Always obtain a right-sided EKG (VI-V6 on right chest) in inferior wall Ml ■▶ if ST segment
elevation in V4, then posterior right ventricle also affected. This indicates a "right-sided Ml"" Fluids
and avoid NTG.
i
n
c
5
I

Copyright © 20I2. Doctors In Training.com. LLC. All Rights Reserved. [33]


. What are the two most common complications after an Ml?

What would cause the following findings after an Ml? (FAI I p272) (FAQ p298)
• Cardiac tamponade
• Severe mitral regurg
• NewVSD
• Stroke

What is Dressier syndrome? (FAI I p272) (FAQ p298)

-. Which ECG leads will show evidence of ischemia in an anterior wall Ml?

". Which ECG leads will show evidence of ischemia in an inferior wall Ml?

16. What medications would you thinkto prescribe in an ER patient diagnosed with Ml?

Copyright © 2012. Doctors In Training.com. LLC. All Rights Reserved.


What does a relative risk less than I indicate? (FAI I p54) (FAQ p57)

A screening test under investigation uses a lab marker SCSQ to identify early small ce
and squamous cell lung cancer. In a population of 100 smokers over age 60, 20 test
positive. Of the 20 that tested positive, 5 actually had either of the two lung cancers,
those that did not test positive, it was determined that 5 had either of the two cancer
What is the sensitivity, specificity, PPV and NPV of this study? (FAI I p53) (FAQ p57)

What are normal BPs in the right and left ventricles?

Cardiomyopathies and Endocarditis


R: Chapters 6, 12
H: Chapter 238
Cardiomyopathies (FAII p273) (FAI2 p299)
Myocarditis (R p578) (H pi953-1961)
Bacterial endocarditis (FAII p275) (FAI2 p30l) (R p566)
Libman-Sacks endocarditis (FAII p275) (FA p30l) (R p220, Fig6-34)

4. Myocarditis (R p578) (H pi953)


• Generalized inflammation of the myocardium (not resulting from ischemia)
• Most common cause in US;
• H i s t o : d i ff u s e i n t e r s t i t i a l i n f i l t r a t e o f w i t h m y o c y t e

What are the typical signs and symptoms of endocarditis?


• Fever, chills, weakness, anorexia
• New regurgitation heart murmur or heart failure
- Mitral valve is most common
- Tricuspid is most common in IV drug users ■▶ septic pulmonary infarcts
• Splinter hemorrhages in fingernails
• Osier's nodes (painful red nodules on finger and toe pads)
• Janeway lesions (erythematous macules on palms and soles)
• Roth spots (retinal hemorrhages with clear central areas)
• Signs of embolism: brain infarct ■* focal neuro defects, renal infarct ■* hematuria, splenic infarct
abdominal or shoulder pain
• Systemic immune reaction: glomerulonephritis, arthritis

Copyright© 2012. Doctors In Training.com. LLC. All Rights Reserved.


What are the differences between acute and subacute bacterial endocarditis?

An IV drug user presents with chest pain, dyspnea, tachycardia, and tachypnea. What is ma.,
likely the cause?

A patient in a MVA presents with chest pain, dyspnea, tachycardia, and tachypnea. What is
the most likely cause?

A post-op patient presents with chest pain, dyspnea, tachycardia, and tachypnea. What is the
most likely cause?

A young girl with congenital valve disease is given penidllin prophylactically. In the ER,
bacterial endocarditis is diagnosed. What is the next step in her management7

w> Other Cardiac Pathology


-p: R: Chapter 12
q H: Chapters 237-239
rd Rheumatic heart disease (FAII p276) (FAI2 p302) (R p565)
Q_ Pericarditis (Rp58l)(Hpl97l)
U Cardiac tamponade (FA 11 p2 76) (FA 12 p302)
.ftJ Syphilitic heart disease (FAII p276) (FAI2 p302)
XJ Cardiac tumors (FAII p277) (FAI2 p303) (R p583) (H pl98l)
rd
{J II. What are the diagnostic criteria (Jones criteria) for rheumatic fever?
L_ If evidence of a group A Strep infection, there is a high probability of acute rheumatic fever if a
~ patient has two major criteria and one minor criterion.

- Joints (migratory polyarthritis)


- Heart (pancarditis)
- Nodules (subcutaneous)
- Erythema marginatum (serpiginous skin rash)
- Sydenham chorea (chorea of the face, tongue, upper-limb)
• Minor criteria: arthralgia, fever, elevated ESR or CRP, prolonged PR interval on EKG

12. Kussmaul's sign vs. pulsus paradoxus:


Kussmaul's sign Pulsus paradoxus
Event JVD with inspiration Decreased SBP by more than 10
mmHgwith inspiration
Mechanism Decreased capacity of RV Decreased capacity of LV
Disease Constrictive pericarditis » Cardiac tamponade » pericarditis
tamponade

Copyright 3 2012. Doctors In Training.com, LLC. All Rights Reserved.


sion
Under what drcumstances might you see pulsus paradoxus?

tat are the Jones criteria for the diagnosis of acute rheumatic fever?

What are the common causes of restrictive cardiomyopathy?

16. What heart pathology fits the following statement7


I Diffuse myocardial inflammation with necrosis
and mononuclear cells
Focal myocardial inflammation with multinucleate
giant cells
Fever + IVDA + new heart murmur

Chest pain and course rubbing heart sounds in


patient with Cr of 5.0
Tree-barking of the aorta

Child with fever, joint pain, cutaneous nodules 4


weeks after a throat infection
ST elevations in all EKG leads

Disordered growth of myocytes

EKG shows electrical alternans

Copyright © 2012. Doctors In Training.com. LLC. All Rights Reserved.


What organisms are known for causing endocarditis? (FAI I p275) (FAQ p30l)

In a study of 100 people, your study results show an average of 500 with a standard
deviation of 50. Calculate the 95% confidence interval. (FAQ p54) (FAQ p57)

Vascular Disorders I
R: Chapter II
H: Chapter 326

Varicose veins (FAII p277) (FAI2 p303)


Raynaud phenomenon (FAII p277) (FAI2 p303)
Sturge-Weber disease (FAII p277) (FAI2 p305)
Vascular tumors (FAII p279) (FAI2 p305)
Small-vessel vasculitis (FAII p277-278) (FAI2 p304-305)
- Microscopicpolyangiitis (FAII p277) (FAI2 p304)
- Wegener's granulomatosis (FAII p277) (FA12 p304)
- Churg-Strauss syndrome (FAII p277) (FAI2 p304)
- Henoch-Schonlein purpura (FAII p278) (FAI2 p305)

£29
Which disorders are commonly discovered in patients with Raynaud phenomenon?
(FAIIp277)(FAI2p303)(Rp5l8)

What local skin problems may arise from venous insufficiency resulting in varicose veins?
(Rp5l8)

What are the characteristic features of Henoch-Schonlein purpura? (FAI I p278) (FAQ p303)

Copyright © 2012. Doctors In Training.com. LLC. All Rights Reserved.


Vascular Disorders 2
R: Chapter II
H: Chapter 326

Medium-vessel vasculitis (FAII p278) (FAI2 p304)


- Polyarteritis nodosa (FAII p278) (FAI2 p304)
- Kawasaki disease (FAII p278) (FA12 p304)
- Buerger disease (FAII p278) (FAI2 p304)
Large-vessel vasculitis (FAII p278-279) (FAI2 p304)
- Temporal arteritis (FAII p279) (FAI2 p304)
- Takayasu arteritis (FAII p278) (FAI2 p304)

n Quiz

diagnose temporal arteritis?

What are the characteristic features of Kawasaki disease?

Which type of vasculitis fits the following high-yield characteristics?


Weak pulses in upper extremities
Necrotizing granulomas of lung and necrotizing
glomerulonephritis
Necrotizing immune complex inflammation of visceral/renal
vessels
| Young male smokers
Young Asian women
Young asthmatics
i Infants and young children; involved coronary arteries
| Most common vasculitis
| Associated with hepatitis B infection
I Occlusion of ophthalmic artery can lead to blindness
Perforation of nasal septum
j Unilateral headache, jaw claudication

Which vascular tumor fits the following description? (FAI I p279) (FAQ p305)

Benign, raised, red lesion about the size of a mole in older


patients
Raised, red area present at birth, increases in size initially
then regresses over months to years
Lesion caused by lymphoangiogenic growth factors in an
infected HIV patient
Polypoid red lesion found in pregnancy or after trauma j
I Benign, painful, red-blue tumor under fingernails
Cavernous lymphangioma associated with Turner syndrome
Skin papule in AIDS patient caused by Bartonella

Copyright © 2012. Doctors In Training.com, LLC. All Rights Reserved.


What should I do five days prior to the test?
• I would split your Step I book reference such as First Aid into 4-5 equal sections. Since FA contains
some 600 pages of material, each section will have about 120 pages. Over 4-5 days review your text
entirely by going through about 120 pages per day. If you go through 10 pages an hour; it should take
you 9-12 hrs per day to do this. As you come across a detail that you did not recognize before or have
not seen before, write it down. At the end of each day and at the beginning of the next day, review
those details you have written. The small details in FA are often overlooked and may be on your test
If you dedde to continue with a question bank during these 5 days, I wouldn't do more than 25-50
questions a day. Concentrate on your knowledge base.
• At the end of Day 5,1 would do the following:
- Review all of the unfamiliar details written from the previous 5 days
- Review all the questions in the DIT study guide, the "definitely know" topics highlighted during the
course, and the 3-5 star topics highlighted during the course.
- Decide what information is going to be put on your marker board right before the test and memorize
that information.
- Consider taking a 150 NMBE test online (free or $45). Since these tests are slightly easier than the
real thing rt will boost your confidence a Ifttle. Since there are no answers given, you won't waste
time going through the answers. You can expect 3-4 of those questions to be on your actual test It
will allow you to regain familiarity with the actual testing format and make sure you've gone through
the tutorial so you can skip it on your test day and add that time to your break time.
- Skim through FA again at your leisure to review random topics, but pay particular attention to the
"definitely know" and 3-5 star topics
- Stop studying at least 2 hours before bedtime to make sure you get a good night's sleep.
• By going through this 5-day program, you will be able to say and know that you've reviewed everything
in the 5 days before the test

What should I put on my markerboard prior to the start of the test?


• Don't write on your markerboard for more than 5 minutes before you start your test
• Put whatever you want but you may want to consider the following
- Developmental milestones
- The 4 pharmacokinetic equations
- Error square
- Sens, spec, PPV, NPV, OR, RR equations and square
- Lung volume diagram

Test Day Tips


• Bring a cooler with ice, water; Gatorade, or juice. Pack a lunch. Bring some fruits and snacks. (You may
not be able to predict what you're going to want to eat so it's better to bring too much than too little.)
Eat ligfit, not heavy.
• Consider getting out in the sun and/or stretching during your breaks.

j-. • Bring a light sweater or sweatshirt in case it's cold.


Q_ • Don't forget your ID and USMLE pass.
|— • Take your breaks when you need them/however you want them (for example: 2 sections ■» break ■▶
s_ 2 sections ■* break ■▶ I section ■» break ■* 2 sections). Some breaks may need to be longer than
<£ others. Don't be afraid to take a small 5 minute bathroom break
Q • Expect 5-10 questions in each section that you have never seen before. If you expect this, then it won't
I freak you out when it happens (and it will happen).
(jO • Bring your own watch to keep track of your break time!
M-* • Consider answering 10 practice questions prior to going into the test center for "warm-up" (but don't
' look at the answers in case you are incorrect).

[40] Copyright © 2012, Doctors In Training.com. LLC. All Rights Reserved.


stio
Which exotoxin works via the following mechanism? (FAI I pl43) (FAQ pl54)
• Inactivates EF-2
• Actives Gs
• Disables Gi
• Bacterial adenylate cyclase (no G protein involved)
• Blocks GABA and glycine

Which immune deficiency matches the following descriptions? (FAI I p2l3) (FAQ p23"
• Anaphylaxis on exposure to blood products with IgA
• Course facial features, abscesses, eczema
• Thrombocytopenia, purpura, infections, eczema
• Delayed separation of the umbilicus
• Neuro defects, partial albinism, recurrent infections

Describe how the murmur of mitral regurgitation is different than the murmur of aor
regurgitation. (FAI I p255) (FAQ pFA282)

Bone Histology and Pathology


R: Chapter 26

Bone histology (R pl206, Fig. 26-1)


Bone formation (FAII p379) (FAI2 p4l4)
Osteoclasts (Rpl206, 1216)
Osteoblasts (R pl207)
Achondroplasia (FAII p379) (FAI2 p4l4)
Osteoporosis (FAII p379) (FAI2 p4l4)
Bisphosphonates (FAII p392) (FAI2 p490)
Osteopetrosis (FAII p379) (FA/2 p4l4)
Osteomalacialrickets (FAII p379) (FAI2 p4l4)
Osteitis fibrosa cystica

Copyright'.'^ 2012. Doctors In Training.com, LLC. All Rights Reserved.


4. Osteoporosis Treatment
• Sfejp smoking, stop alcohol consumption, stop steroids (if able), avoid PPI and H2-blockers if possible
• Exercise (aerobic, resistance, and weight bearing)
• EaJLrJsk assessment at home
• Vitamin D supplementation
• Calcium supplementation
• Bisphosphonates (inhibit osteoclastic resorption, 4. fracture risk by 40-50%)
• ETH (anabolic agent that stimulates osteoblasts) for moderate-severe OP treatment: Teriparatide
decreases hip fracture rate by 53%, duration of therapy not to exceed 2yrs, must use bisphosphonate
after stopping PTH to maintain BMD

Other Treatments Worth Knowing


• Testosterone replacement for men with low testosterone (relatively un-researched but theoretically a
good idea; OP with testosterone deficiency should still be treated with bisphosphonates.
• Conjugated estrogen (WHI trial ■▶ hip fx reduction of 33%) USPSTF recommends not using estrogen
for the sole purpose for fx reduction risk due to risk of other SE (Ml and CVA). May use with
bisphosphonate. Accelerated bone loss is seen after withdrawal of estrogen therapy.
• SERM: Raloxifene decreases vertebral fx by 40% in women with OP, no effect on risk of non-vertebral
fx risk, reduces risk of breast CA.
CO
i_ • Calcitonin (Miacalcin) - inhibits osteoclasts, dosed nasally or SC/IM, not as effective as bisphosphonates;
in fact, overall effectiveness is questionable
"2^ • Combination therapy (i.e., bisphosphonate + Raloxifene)

L-J 5. Osteitis fibrosa cystic (AKA von Recklinghausen disease of bone)


^ • A bony manifestation of an endocrine disorder
O • Can be caused by:
CD - Hyperparathyroidism
• High PTH ^ high serum calcium, low serum phosphate, high alkaline phosphatase
c
fti - Type IA pseudohypoparathyroidism (AKA Albright Hereditary Osteodystrophy)
V • PTH resistance at the renal tubules ■▶ low serum calcium and high phosphate
O * L o w c a l d u m ■» h i g h P T H
CQ • High PTH ■» excess osteoclastic activity ■» "Brown tumors" in bone which are cystic spaces lined by
I osteodasts filled with fibrous stroma and blood
— • DEXA scan reveals low bone mineral density, but the mechanism of bone loss is different from OP

zerr
LU
Q
Q
<

I
CfT

[2] Copyright © 2012. Doctors In Training.com. LLC. All Rights Reserved.


UIZ
6. Where does new bone formation take place in growing long bones?

What cells types are most similar osteoclasts?

Which bony disease fits the following description?


Reversible when vitamin D is replaced
Excess osteoclastic activity results in disorganized bony architecture
Bone is replaced by fibroblasts, collagen, and irregular bony trabecule
Soft bones due to defective mineralization of osteoid
Failure of bone resorption ■▶ thickened and dense bones
Genetic deficiency of carbonic anhydrase II

What are the risk factors for osteoporosis? (FAI I p379) (FAQ p379)

What measures can be taken to prevent osteoporosis?

11. What bone disorder results from excess PTH?

Paget Disease and Bone Tumors


R: Chapter 26

Paget disease (FAII p380) (FAI2 p4l4)


Polyostotic fibrous dysplasia (FAII p380) (FAI2 p4l5)
Lab values in bone disorders (FAII p380) (FAI2 p4!5)
Primary bone tumors (FAII p38l) (FAI2 p4l6)

Copyright © 2012. Doctors In Training.com, LLC. All Rights Reserved.


1111 ii i u ic iuiiuwii rg Lduic. inru i pjou; i^r/-\iz p^iD/

Serum Ca2+ Serum Phos Alk Phos PTH


Paget Disease
Osteomalacia/rickets
Osteitis fibrosa cystica
Osteoporosis
Osteopetrosis

. Which primary bone tumor fits the following description? (FAI I p38l-382) (FAQ p 415-416)
• Most common malignant primary bone tumor of children
• Most common malignant primary bone tumor in adults
• Most common benign bone tumor
• 11 ;22 translocation
• Soap-bubble appearance on x-ray
• Onion-skin appearance of bone
• May actually be a hamartoma
• Codman's triangle on x-ray
Associated with Gardner

Copyright © 2012, Doctors In Training.com. LLC All Rights Reserved.


arm-
I. What signs and symptoms would lead you to suspect rheumatic heart disease? (FAI I
p276) (FAQ p302)

When does implantation of the ovum take place? (FAI I p482) (FAQ p536)

What nerve is damaged when a patient presents with the following symptom (upper
extremity)? (FAII p374-375) (FAQ p409-4IO)
• Claw hand
• Wristdrop
• Scapular winging
• Unable to wipe bottom
• Loss of forearm pronation
• Weak lateral rotation of arm
• Loss of arm and forearm flexion
• Loss of forearm extension
Unable to raise arm abt

Basics and Lower Extremity Joints


COA: Chapters 3, 5
R: Chapter 26

Joint basics
Types of joints
Chondrocytes (R pi227)
Hip
- Anterior hip dislocation
- Osteoarthritis (FAII p382) (FAI2 p4l7)
Knee
- Unhappy triad I knee injury (FAII p370) (FAI2 p405)
- Patellofemoral syndrome
- Prepatellar bursitis (housemaid's knee)
Anklelfeetltoes
- Ankle sprain
- Plantar fasciitis
- Gout

Copyright © 2012, Doctors In Trainnig.com. LLC. All Rights Reserved.


A football player who was kicked in the leg suffers from a damaged medial meniscus.
What else is likely to have been damaged?

What ligaments are typically injured during an ankle sprain?

What is the leading cause of knee pain in patients younger than 45?

7. What is the other name for housemaid's knee?

Upper Extremity Joints


COA: Chapter 6

Shoulder
- Rotator cuff muscles (FAII p37l) (FAI2 p405)
- Shoulder dislocation
- Shoulder separation
- Subacromial bursitis (H: Chapter 337)
- Adhesive capsulitis
Elbov/
- Repetitive elbow trauma (FAII p376) (FAI2 p4ll)
- Epicondylitis
- Ulnar nerve injury
- Tear of biceps tendon
Hands and wrist
- Bones of the hand
- Carpal tunnel syndrome
- Dupuytren contracture

8. What is the difference between a dislocated shoulder and a separated shoulder?


Dislocated - head of humerus rotates out of glenoid cavity
Separated - clavicle separates from acromion and coracoid process of the scapula

9. What structures can be damaged in an anterior shoulder dislocation?


• Axillary nerve and posterior circumflex artery
• Supraspinatus tendon
• Anterior glenohumeral ligaments and glenoid labrum separation from the articular surface of the
anterior glenoid neck (AKA Bankart lesion)
• Posterolateral humeral head defect (due to abrasion against the anterior rim of the glenoid)
- (AKA Hill-Sack lesion)

Copyright © 2012. Doctors In Training com. LLC. All Rights Reserved.


n
10. Carpal Bones

Lippincott Williams & Wilkins Atlas of Anatomy ©2008 Wolters Kluwer Health. Inc. All rights reserved.

Which muscles of the rotator cuff are responsible fo EHS^Hfft ctions? (FAI I p
(FAQ p 405)
• Initial 15 degrees of arm abduction
Lateral rotation of arm
Medial rotation of arm

What elbow injury is common in tennis players? What elbow injury is common in golfers?
(FAII p376)

Which antibiotic class is known to increase the risk of tendon rupture in adults?

A patient comes to the clinic complaining of anterior shoulder pain that radiates down
into the forearm. On examination, you notice a swelling of the biceps muscle. What
the most likely diagnosis?

Copyright © 2012. Doctors In Training.com. LLC. All Rights Reserved.


What portion of the brachial plexus is injured in Erb-Duchenne Palsy? What are the
symptoms? (FAI I p375) (FAQ P4I0)

What is the clinical appearance of intemuclear ophthalmoplegia? With what disorder is it


commonly associated? (FAI I p424) (FAQ p464)

Which lipid-lowering agent matches the following description? (FAI I p282) (FAQ p308)
• SE: facial flushing
• SE: elevated LFTs, myositis
• SE: Gl discomfort bad taste
• Best effect on HDL
• Best effect on triglyceridesA/LDL
• Best effect on LDL/cholesterol
\nds C.diff. toxin

Osteoarthritis and Treatment


H: Chapter 332
R: Chapter 26
GG: Chapter 34
en
<D Osteoarthritis (FA11 p382) (FAI2 p417)
$ Aspirin (FAII p39l) (FAI2 p429)
OJ NSAIDS (FAII p39l) (FAI2 p429) (H p2835, Table 332-1)
. !£ COX-2 inhibitors (FAII p392) (FAI2 p430)
Q Acetaminophen (FA 11 p392) (FA 12 p430)

What is the mechanism of treating acetaminophen overdose? (FAI I p392) (FAQ p43<

What is the classic feature of osteoarthritis? (FAI I p382) (FAQ p43l)

By what mechanism do NSAIDS cause renal disease? (FAI I p39l) (FAQ p430)

Copyright © 2012, Doctors In Training.com, LLC. All Rights Reserved.


Other Joint Diseases
H: Chapters 331, 333, 334
R: Chapter 26
GG: Chapter 34
Rheumatoid arthritis (FAII p383) (FAI2 p4l8)
Sjogren syndrome (FAII p383) (FAI2 p4l8)
Gout (FAII p 384) (FAI2 p4l9)
Gout drugs (FAII p392) (FAI2 p4l9) (GG p994)
Pseudogout (FAII p384) (FAI2 p4l9) (H p2839)
Infectious arthritis (FAII p384) (FAI2 p420) (H p2842)

joints and Achilles tendon, and exquisite pain in the metatarsophalangeal joint of his ri
big toes. Biopsy reveals needle-like crystals. What is the diagnosis?

What medications are used in the treatment of an acute gout exacerbation?

What drugs can be used in the treatment of chronic gout? (FAI I p393) (FAQ p431)

tat are the classic symptoms of Sjogren syndrome? (FAI I p383) (FAQ p4l8)

Copyright© 2012. Doctors In Training.com. LLC. All Rights Reserved.


A 75-year-old man presents with acute knee pain and swelling. An X-ray reveals absenc
of erosion of the joint space and calcium deposits in the menisci. What is the diagnosis,
and what would you find on aspiration of the joint? (FAI I p384) (FAQ p4l9) (H p2839)

What are the common locations for tophi in gout patients? (FAI I p384) (FAQ p4l9)

What is the mechanism and time-frame of acute transplant rejection?


(FAI I p2l5) (FAQ p239) (R p228)

Systemic Rheumatic Diseases


H: Chapters 318-323
Seronegative spondyloarthropathies (FAII p385) (FA12 p42l)
TNF-alpha inhibitors (FAII p393) (FAI2 p432)
Systemic lupus erythematosus (FAII p385) (FAI2 p42l)
Polymyalgia rheumatica (FAII p386) (FAI2 p42l)
Scleroderma (FAII p387) (FAI2 p423)

4. Systemic Lupus Erythematosus = 4/11


• Skin
- Malar rash
- Discoid rash
- Photosensitivity
- Oral ulcers
• -itis
- Arthritis (non erosive, 2 joints)
- Serositis (pleuritis, pericarditis)
- (+)ANA
• Disorders
- Renal (proteinuria, cellular casts)
- Neuro (seizures, psychosis)
- Heme (hemolytic anemia, leukopenia, lymphopenia, thrombocytopenia)
- Immune (antiphospholipid Ab (+), anti-dsDNA, anti-Smith, false (+) VDRL)

Copyright © 2012. Doctors In Training.com, LLC. All Rights Reserved.


ick Qui
5. What drugs are known for causing drug-induced lupus? (FAI I p385) (FAQ p4_.

lat are the manifestations of CREST scleroderma? (FAI I p387) (FAQ p423)

A patient has difficulty swallowing, distal cyanosis in cold temperatures, and anti-
centromere antibodies. What else would you expect to see in this patient?

A patient presents with photosensitivity, arthritis, renal disease and recurrent oral ulcers is
taking primaquine and NSAIDS. What type of check-up should she be receiving twice aye.

30-year-old woman presents with a low grade fever, a rash across her nose that gt
worse when she is out in the sun, and widespread edema. What blood test would >
orderto confirm your clinical suspicion?

Muscle Disorders
R: Chapter 26
H: Chapter 335
Muscular dystrophies (FAII p87) (FAI2 p9l) (R pl268)
Polymyositisldermatomyositis (FAII p386) (FAI2 p422 ) (R pl270)
Neuromuscular junction diseases (FAII p386) (FAI2 p422) (R pl270)
Fibromyalgia (H p2849)

10. Fibromyalgia
• Excess muscular tenderness in II of 18 particular sites
• Chronic generalized pain, fatigue, sleep disturbances, HA, cognitive difficulty, mood disturbances
• 30% also have depression and/or anxiety
• Pharm treatment
- FDA-approved: pregabalin, milnacipran
- Traditional (non FDA-approved): amitriptyline, low dose analgesic, fluoxetine
• Non-pharmacological treatment: reassurance that it is a real disease and that it is benign, exercise and
stretching, sleep, relaxation techniques, stress reduction

Copyright © 2012. Doctors In Training.com. LLC. All Rights Reserved.


. What is the cause of Duchenne muscular dystrophy? (FAI I p87) (FAQ p9l)

What is the function of the protein that is absent in Duchenne muscular dystrophy?
(FAII p87)(FAI2p9l)

13. What are some of the characteristics of polymyositis that distinguish it from polymyalgia
rheumatic? (FAI I p386) (FAQ p422)

hat drug category is often used in the treatment of myasthenia gravis?


(FAII p386) (FAQp422)

Which cancer is particularly associated with Lambert-Eaton syndrome?


(FAII p386) (FAQ p422)

Copyright © 2012. Doctors In Training.com. LLC. All Rights Reserved.


What are the symptoms of normal pressure hydrocephalus? (FAI I p408) (FAQ p448)

,n which genetic disorder would you find acanthocytosis of RBCs and excess lipids in
enterocytes? (FAI I pi 15) (FAQ pQI)

What disorder is associated with the following mutation? (FAI I p86-87) (FAQ p 90-9P
• FGF receptor 3
• FMRI gene

Derm Basics and Derm Path Part I


H: Chapters S3, 54
Epidermis layers (FAII p370) (FAI2 p404)
Epithelial cell junctions (FAII p370) (FAI2 p404)
Nonepithelial cell junctions
Fibroblasts (R pi02)
Melanocytes (R pi 168)
Derm terminology (FAII p387) (FAI2 p423)
Derm path part I (FAII p388-389) (FAI2 p424-427)

4. Nonepithelial/Nonjunctional Adhesion Mechanisms


• Cadherins
• ICAMs
• Integrins
• Selectins

5. Hemidesmosome, cadherin, integrin, ICAM-I. Which joins only cells of the same type
and does not attach cells to the basement membrane?

6. Fibroblasts
• Collagen
• Glycosaminoglycans
• Reticular and elastic fibers
• Glycoproteins
• Stimulated by tissue damage

7. Melanocytes
• Melanin-producing cells
• Located in the stratum basale (bottom layer) of the epidermis
• Responsible for skin color

Copyright© 2012. Doctors In Traimng.com. LLC. All Rights Reserved.


8. Energy Exchange That Takes Place at the Skin
• UV light is used to generate vitamin D
• Melanocytes displace UV light as heat
• Bilirubin is converted
• Dendritic cells process antigens
• Mechanical energy is converted to electrical energy by nerve endings

9. Atopic dermatitis (eczema) Treatment:


• Switching to a moisturizing soap (Dove, Aveeno) and adding an OTC emollient may be all that is
needed for maintenance and mild cases
• Calcineurin inhibitors: tacrolimus (Protopic) or pimecrolimus (Elidel)
• Topical Steroids
• Antibiotics for open lesions (cover Staph, aureus and Strep, spp.)
• Antihistamines
• Leukotriene inhibitors (Montelukast) - theoretical efficacy supported by weak studies
• UV light therapy
• Systemic steroids (l-2mg/kg/d in children then taper) only in severe cases and only for short duration
• For very severe cases, consider methotrexate, cyclosporin, azathioprine

10. Acne
Pathophysiology Treatment
Hyperkeratosis
Sebum overproduction
Propionibacterium acnes proliferation
Inflammation

A pregnant patient comes to the clinic complaining of new dark pigmentation on her face.
What is the most likely diagnosis? (FAI I p388) (FAQ p424)

What are some of the hallmark features of necrotizing fasciitis? What organism causes this
infection? (FAI I p388) (FAQ p425)

What organism causes painless white patches on the tongue that cannot be scraped off?
(FAII p388) (FAQp425)

What is a lesion of excessive collagen deposition especially around the face, ears, neck, or
upper chest called? (FAI I p387) (FAQ p423)

What organism is responsible for acne vulgaris?

Copyright © 2012, Doctors In Trainmg.com, LLC. All Rights Reserved.


Derm Path Part 2
H: Chapter 52

Derm path part 2 (FAII p388-389) (FAI2 p424-427)


Psoriasis (H p395)
Blistering disorders
Skin cancers (FAII p390) (FAI2 p428)

16. Psoriasis Treatment Options


• Treatment Options (effectiveness commonly wanes, so rotate therapies about q-year)
• Topical steroids - NEVER give oral steroids to someone with psoriasis!
• Calcipotriene (vitamin D3 analog that inhibits epidermal cell proliferation)
- Most worldwide prescribed single-agent treatment for psoriasis
• Tazarotene (Tazorac) (topical retinoid ■* normalizes keratinocyte proliferation)
• Coal tar (suppresses DNA synthesis)
- Brown tar (liquor carbonis detergens, LCD) comes in moisturizers, soaks, or ointments
- Black tar (crude coal tar) comes as 2%, 5%, or 10% (FDA ruled 5% or less not carcinogenic)
- Use with UV therapy (Goeckerman therapy) for mod-severe psoriasis
- Must be left on for at least 4 hours. Stains clothes and sheets.

- It is messy (purple-brown staining of skin (reversible) and everything), irritating, and moderate efficacy
- Absorption occurs within I hour ■» may shower an hour after application
Salicylic acid (keratolytic used to remove excess scale)
i l IV -t-hpt-apy- for patients with more than 10% (refer to a Dermatologist)
e (acitretin)
- Use if > I0%BSA
- CBC, LFTs, Lipids qlmx 3 then q3m
- Absolute contraindication in pregnancy!
: iniprtinnc; into dermk
■ Etanercept: Anti-TNF agent approved for use in mod-severe psoriasis
' Others agents (if refractory to Soriatane or UV light): oral retinoids, methotrexate, cyclosporine

17. Which skin disorder matches the following statement?


• Pruritic, purple, polygonal papules
• Life threatening rash with bullae
• Pruritus associated with asthma
• Pruritic vesicles associated with celiac disease
Allergy to nickel
Thickened scar esp. around face/chest
Antibodies against epidermal basement membrane
Antibodies against cell-cell adhesions
• Parakeratotic scaling
• Keratin-filled cysts
• Sand-paper; predisposition to squamous cell cancer
Skin rash and proximal muscle weakness
Honey-crusting lesions common about the nose and lips
• Hyperkeratosis and koilocytosis
• Histology shows palisading nuclei

Copyright 3 2012. Doctors In Training.com. LLC. All Rights Reserved.


u est ion warm-
What are the Kubler-Ross stages of grief? (FAI I p6l)

What physiologic changes are sensed by peripheral chemoreceptors? By cen.


chemoreceptors? (FAI I p265) (FAQ p290)

Which oncogene is associated with colon cancer? Which is associated with small cell li
cancer? (FAI I p227) (FAQ p253)

Psychology Basics
8roin Organization
- Amygdala (Phys p7l9)
- Brainstem neurotransmitter changes with disease (FAII p442) (FAI2 p485)
Freud's structural theory of the mind (FAII p439)
Psychology Basics
- Personality
- Life stages
- Maslow's hierarchy of needs
- Suffering, hope, helplessness
- Review of stress
- List of stressful events by degree of stress induced

4. Erikson's Stages of Development (1968)

^ Stage in Life Ages Psychosocial Crisis


m
Infant Birth- 18m Basic Trust vs. Mistrust
»
>
Toddler 18m - 3yrs Autonomy vs. Shame
*
Preschooler 3yrs - 5yrs Initiative vs. Guilt
1 School-Age Child 6yrs - 12 yrs Industry vs. Inferiority
Adolescent 12yrs - 18yrs Identity vs. Role Confusion

> Young Adult 18 yrs - 35 yrs Intimacy vs. Isolation
k> Middle-Aged Adult 35 yrs - 55 to 65 yrs Generativity vs. Self-absorption
Older Adult 55 to 65 yrs - Death Integrity vs. Despair
»

Maslow's Hierarchy of Needs (1943)


1) Physiological Needs
2) Safety Needs
3) Love and Belonging
4) Esteem
5) Self-Actualization

Copyright (■) 2012. Doctors In Training.com. LLC. All Rights Reserved.


f fl M B B H R H
What psychosocial crisis are most 25-year-olds working through?

What is the order of human needs as described by Maslow?

What neurotransmitter changes do you see with the following diseases?


(FAII p442) (FAQp485)
• Anxiety disorders
• Depression
• Mania
• Alzheimer disease
• Huntington disease
• Schizophrenia
itjmi^jjiajijjjj^jy

Learning and Therapy


Phys: Chapter 58
Stages of learning
Types of therapy counseling

10. Four Stages of Learning

1) Unconscious Incompetence
2) Conscious Incompetence
3) Conscious Competence
4) Unconscious Competence

11. Bloom's Taxonomy of Learning Objectives

1) Knowledge
2) Comprehension
3) Application
4) Analysis
5) Synthesis
6) Evaluation

12. Cognitive Behavioral Therapy


• Treatment tools:
- Joumaling
- Challenging beliefs
- Mindfulness
- Relaxation
• Treatment goals:
- Recognize unrealistic perceptions and maladaptive behavior patterns
- Replace negative thoughts/behaviors with images/beliefs/actions that facilitate recovery

Copyright C'j 2012. Doctors In Training.com. LLC All Rights Reserved.


13. Psychoanalysis
• Analysis of dreams, fantasies, and associations, as well as the verbal and physical expression of thoughts
• Therapist helps the client recognize and confront the inner conflicts responsible for symptoms and
behavior problems

14. Axis
I) Psychiatric disorders
II) Intellectual disabilities and personality disorders
III) General medical condition
IV) Psychosocial
V) Global Assessment of Function

Which portion of the brain is responsible for delayed gratification?

16. Which portion of the brain is responsible for memory?

'hat term applies to the growth of new neurons?

Copyright © 2012. Doctors In Tmintng.com. LLC. All Rights Reserved.


What are some of the clinical effects of zinc deficiency!

What is the cause of anemia given the following statement?


(FAI I p349-353) (FAQ p380-385)
• Microcytic anemia + swallowing difficulty + glossitis
• Microcytic anemia + > 3.5% HbA2
• Megaloblastic anemia not correctable by Bl2 or folate
• Megaloblastic anemia along with peripheral neuropathy
• Microcytic anemia + basophilic stippling
• Microcytic anemia reversible with B6
• HIV positive patient with macrocytic anemia
• Normocytic anemia + red urine in the morning
• Normocytic anemia and elevated creatinine

Vhat type of cancer is associated with the following tumor suppressor genes?
(FAII p228) (FAQp253)

APC
WTI
BRCAI and BRCA2

Infants and Early Childhood


Regress/on in children (FAII p44l)
Child abuse (FAII p44l) (FAI2 p484)
Child neglect (FAII p44l) (FAI2 p484)
Infant deprivation effects (FAII p44l) (FAI2 p484)
Depression in infants - Anaclitic depression (FAII p44l)
Mental retardation
Trichotillomania
Childhood and early-onset disorders (FAII p442) (FAI2 p484)
ADHD medications (FAII p452) (FAI2 p497) (GG p303)

4. What are the most common causes of mental retardation?


• Down syndrome, Fragile X syndrome, Rett syndrome
• Congenital hypothyroidism
• Fetal alcohol syndrome and other prenatal toxin exposures (lead, mercury, valproate)
• Perinatal hypoxia or infection
• Postnatal causes: trauma/abuse, CNS hemorrhage, hypoxia (e.g., near-drowning), toxins, psychosocial
deprivation, malnutrition, intracranial infection, and CNS malignancy

Copyright © 2012. Doctors In Training.com. LLC. All Rights Reserved.


5. Trichotillomania
• Compulsive nervous hair pulling
• More common in young girls
• Examination reveals unusual patterns of broken hairs of varying length ■▶ "wire brush" feel
• Treatment: Education ■» Cognitive behavioral therapy ■» fluoxetine or clomipramine

6. Stimulants Used in ADHD


• Methylphenidate (Ritalin) and Dextroamphetamine (Adderall)
- Amphetamine, increases pre-synaptic norepinephrine vesicle release
• Atomoxetine (Strattera)
- Norepinephrine reuptake inhibitor
• Other clinical uses of stimulants
- Narcolepsy (modafinil), obstructive sleep apnea an excessive daytime sleepiness, major depressive
disorder

When a young monkey is separated from its mother; it becomes withdrawn, socially isolc
and grooms poorly, ft is thought that this behavior is the monkey equivalent of what hun
problem?

A 4-year-old girl complains of pain in her genitalia. On exam, a discharge is noticed and a
smear of the discharge shows N. gonorrhoeae. How was she infected?

What name is given to chronic compulsive hair pulling common in young girls?

What stimulant medications are commonly used in the treatment of ADHD?

Adolescence
H: Chapter 79
Pervasive developmental disorders (FAII p442) (FAI2 p485)
Depression in children
Bullying
Eating disorders (FAII p449) (FAI2 p493) (H p636)
Gender identity disorder (FAII p449) (FAI2 p493)

II. What are some of the characteristic features of autism?


• "Living in his own world"
• Symptoms evident prior to age 3
• Lack of responsiveness to others, poor eye contact, absence of social smile
• Impairments in communication, language delay, repetitive phrases
• Peculiar repetitive, ritualistic habits (e.g., spinning around, hand flapping)
• Fascination with specific, seemingly mundane objects (vacuum cleaners, sprinklers)
• Usually below normal intelligence

Copyright © 2012. Doctors lnTraining.com, LLC. All Rights Reserved.


12. What are the characteristic features of Rett disorder?
Females with normal birth and development for the first 6 months of life
• Stage I
- Deceleration of head growth between ages 5 months and 4 years
- Developmental arrest between 6-18 months
- Social withdrawal and communication dysfunction
• Stage II
- Loss of acquired purposeful hand skills between ages 6-30 months
- Stereotypic hand movements usually midline (e.g., washing, wringing, squeezing) and constant during
waking hours
- Gait and truncal apraxia between ages I -4 years
• Stage III
- Pseudostationary phase beginning between 2-10 years
- Some improvement in behavior, hand use, and communication skills
- More prominent motor dysfunction and seizures
• Stage IV
- Onset after age 10
- Increased rigidity, bradykinesia, reduced mobility

Eating Disorders (FAI I p449) (FAQ p493)


13. What electrolyte changes are seen in patients with excessive purging overtime?

14. Binge Eating Disorder


• Purging is not present
• Expression of deeper psychological problems
• Many actually have negative feelings toward food

15. Compulsive Eating Disorder


• Purging is not present
• Type of obsessive-compulsive disorder
• Much time is spent thinking and fantasizing about food

Which childhood psychiatric disorder matches the following statement7


• Females only, loss of previously acquired purposeful hand skills between 6-30 months
• Impairments in social interactions, communications, play, repetitive behaviors
• Impairment in social interaction (but not avoidance), no language delay
• Stereotyped hand movements
• Ignoring the basic rights of others
• Characterized by hostility, annoyance, vindictiveness, disobedience, and resentfulness
• Multiple motor and vocal tics
• Impulsive and inattentive
• 7-year-old that avoids going to school to stay home with parent

What is the difference between binge eating disorder and compulsive eating disorder?

A 15-year-old girl of normal height and weight for her age has enlarged parotid glands bi
no other complaints. The mother confides she found laxatives in the daughter's closet.

Copyright © 2012. Doctors In Training.com, LLC. All Rights Reserved.


Ebwwr^wi

What is the difference between a warm agglutinin and a cold agglutinin? (FAI I p353)
(FAQ p385)

What are the equations for sensitivity; specificity, positive predictive value, and negativ
predictive value? (FAI I p5l) (FAQ p53)

Delirium, Dementia, and Alcohol


H: Chapters 25, 371, 392
Orientation (FAII p443) (FAI2 p485)
Delirium (FAII p443) (FAI2 p486) (H pl96)
Dementia (FAII p443) (FAI2 p486) (H p3300)
Dissociative disorders (FAII p444) (FAI2 p488)
Substance dependence (FAII p449) (FAI2 p494) (H p3526)
Substance abuse (FAII p449) (FAI2 p494) (H p3526)
Substance withdrawal (FAII p449)
Stages of change (FAII p450) (FAI2 p494)
Signs and symptoms of substance abuse (FAII p450) (FAI2 p495)
- Alcohol, barbiturates, benzos (GG p6S6-657)
- Alcoholism (FAII p45l) (FAI2 p496) (H p3549)
- Delirium tremens (FAII p45l) (FAI2 p496)

4. Stages of Behavioral Change


1) Precontemplation: +/- awareness of problem, no thought of changing
2) Contemplation: develops a desire to change
3) Preparation: intention to change
4) Action: begins to change behavior consistently
5) Maintenance: consistently changed behavior for over 6 months

5. What is the difference between alcohol abuse and alcohol dependence?


(FAII p449) (FAQ p494)
• Alcohol abuse - repeated alcohol use despite recurrent adverse consequences
• Alcohol dependence - at least 3 of the following: tolerance; withdrawal; alcohol abuse; inability to quit,
cut-back, or regulate use; great amount of time spent obtaining, using, or recovering

6. What medications are effective in helping to prevent relapse in recovering alcoholics?


• is the tried-and-true best relapse prevention

Acamprosate (Campral)

Copyright © 2012. Doctors In Traimng.com. LLC. All Rights Reserved.


exam. When asked what he would do if he smelled smoke in a movie theater; he replie
that he would yell "fire." When asked what a table and chair have in common, he replie
that they both have 4 legs are made of wood. The family reports that for the past year
someone has had to stay with him at all times for his own safety and that he stays awake
all day and sleeps well at night. What is the most likely diagnosis in this patient?

A 72-year-old patient of yours is brought into the clinic by his family because of strange
behaviors over the past week. She has been very agitated recently, takes many naps
during the day, occasionally urinates on herself and has had a poor appetite. It is difficult
for you to administer the MMSE because she is unable to focus her attention on the
questions. What is the most likely diagnosis?

What are the stages of behavioral change?

Substance Abuse
H: Chapter 393
GG: Chapters 18, 24

Signs and symptoms of substance abuse (FAII p450) (FAI2 p495)


Opioids (H p3552) (GG p658)
Heroin addiction (FAII p45l) (FAI2 p496) (H p649)
Opioid analgesics (FAII p430) (FAI2 p47l) (GG p48l)
- Butorphanol (FAII p430) (FAI2 p472) (GG p509)
- Tramadol (FAII p430) (FAI2 p472) (GGp508)
Amphetamines and cocaine (GG p66l)
Caffeine and nicotine (GG p657)
PCP (GG p666)
LSD (GG p665)
Marijuana (GG p663)

Copyright © 2012. Doctors In Training.com. LLC All Rights Reserved.


What drug intoxication or withdrawal is causing the following symptoms in the followir
patients?
• Post-op constipation and/or respiratory depression
• Severe depression, headache, fatigue, insomnia/hypersomnia, hunger
• Pinpoint pupils, N/V, seizures
• Belligerence, impulsiveness, nystagmus, homicidal ideations, psychosis
• Headache, anxiety/depression, weight gain
• Anxiety/depression, delusions, hallucinations, flashbacks
• Euphoria, social withdrawal, impaired judgment, hallucinations
• Rebound anxiety, tremors, seizures, life-threatening
• Anxiety, piloerection, yawning, fever, rhinorrhea, nausea, diarrhea

What weak opioid agonist is commonly used in chronic pain control prior to prescribir
stronger opioids? (FAI I p430) (FAQ p472)

v i iat is the treatment for a patient that is overly intoxicated with each of the followin
substances? (FAI I p450) (FAQ p495)
Alcohol
1 Benzodiazepines |

Copyright © 2012. Doctors In Training.com. LLC. All Rights Reserved.


Which neoplasm is associated with the following statement!
• Nftrosamines
• Asbestos
• Naphthalene
• Arsenic
• EBV
• HPV
• Schistosoma haematobium

What is the cause of ITP? (FAI I p355) (FAQ p387)

What is the mechanism of action of the following drugs? (FAI I p362-364) (FAQ p396-
397)
• Streptokinase
• Aspirin
• Clopidogrel
• Abciximab
• Tirofiban
• Ticlopidine
• Enoxaparin
• Eptifibatide

cd Psychosis and Schizophrenia


H: Chapter 391
GG: Chapter 16
Hallucination vs. illusion vs. delusion vs. loose association (FAII p443) (FAI2 p486)
Hallucination types (FAII p444) (FAI2 p487)
Delusional disorder (FAII p444) (FAI2 p487)
Anatomy and physiology of schizophrenia
Schizophrenia (FAII p444) (FAI2 p487) (H p3542)
Keeping "schizo-"straight (FAII p449) (FAI2 p493)
Antipsychotics (FAII p453) (FAI2 p498) (GG p428) (H p3544, Table 391-14)
Atypical antipsychotics (FAII p4S3) (FAI2 p498) (GG p428) (H p3544, Table 391-14)

Copyright © 2012. Doctors In Training com, LLC. All Rights Reserved.


• Schizotypal
• Schizophrenia
• Schizoaffective
- Schizoid
Schizophreniform

What are some of the positive symptoms of schizophrenia? What are some of the neg
symptoms?

Categorize the following antipsychotics in the appropriate category as neuroleptics (low,


moderate, or high potency) or atypical antipsychotics: olanzapine, thioridazine, quetiapine,
molindone, chlorpromazine, haloperidol, fluphenazine, loxapine, risperidone, thiothixene,
trifluoperazine, clozapine, aripiprazole
i M M l M B U B t fl l

What are the clinical features of neuroleptic malignant syndrome? How is it treated?

You are on-call and receive a call from a nurse asking if she can give some sleep median
or diphenhydramine to an elderly patient with mild dementia who is currently hospitalizt

Mania and Bipolar Disorder


H: Chapter 391
GG: Chapter 16

Manic episode (FAII p445) (FAI2 p488)


Hypomanic episode (FAII p44S) (FAI2 p488)
Bipolar disorder (FAII p445) (FAI2 p488) (H p3539)
Lithium (FAII p454) (FA/2 p499) (GG p445)

i[»1im.9lllr4

9. What are the criteria for the diagnosis of mania?

I®, What is the milder form of bipolar disorder?

11. What mood stabilizers are used in the treatment of bipolar disorder?

B Y- T l ^ K ^ ^
.e potential BBI5igB^(oi!iiiii!iii'^

Copyright © 2012. Doctors In Trnining.com. LLC. All Rights Reserved.


What clinical presentation might lead you to suspect a patient may have lymphoma?
(FAI I p357) (FAQ p389)

What form of leukemia matches the following statement?


• Most common leukemia in children
Most common leukemia in adults in US
Characteristic Auer rods
Greater than 20% blasts in marrow
Leukemia with more mature cells and less than 596 blasts
PAS (+) acute leukemia
- Always positive for the Philadelphia chromosome (t 9;22)
Acute leukemia positive for peroxidase
- Solid sheets of lymphoblasts in marrow
• Always associated with the BCR-ABL genes

What histological change takes place in the trachea of a smoker?

Major Depressive Disorder (MDD)


H: Chapter 391

Major depressive episode (FAII p445) (FAI2 p489) (H p3536)


Atypical depression (FAII p446) (FAI2 p489)
Postpartum mood disturbances (FAII p446) (FAI2 p489)
Electroconvulsive therapy (ECT) (FAII p446) (FAI2 p489)
Risk factors for suicide completion (FAII p46) (FAI2 p490)

Copyright"; 2012. Doctors In Training.com. LLC. All Rights Reserved.


ick Quiz
What are the criteria for the diagnosis of major depressive episode?

at is atypical depression?

What is the milder form of depression?

A patient mentions that he has had thoughts of suicide. What questions should you
to help determine how high-risk he is?

A patient tries to commit suicide by slitting her wrists. After she is appropriately managed
in the ER, what question would you ask the patient in order to determine her level of
commitment of trying to take her own life?

A 28-year-old woman has symptoms of mild depression for 6 years. What's the
diagnosis?

Antidepressant Medications
GG: Chapter 15

SSRIs (FAII p4S5) (FAI2 p500) (GGp405)


SNRIs (FAII p455) (FAI2 pSOO) (GGp405)
Tricyclic antidepressants (FAII p455) (FAI2 pSOO) (GG p405)
Monoamine oxidase inhibitors (FAII p455) (FAI2 p500) (GGp405)
Atypical antidepressants (FAII p456) (FAI2 p50l)

Copyright © 2012. Doctors In Traimng.com. LLC. All Rights Reserved.


Two months after the loss of her spouse, a 42-year-old female is having trouble eating,
concentrating and sleeping. She sleeps only 2-3 hours each night. What would you do f
this patient?

What is the mechanism of action of the following medication class?


SSRI-

SNRI-

TCA-

MAOI-

Benzodiazepines-
Barbitu rates-

Typical neuroleptics-
Atypical neuroleptics-

Categorize the following antidepressants as either an SSRI, TCA, MAOI, NDRI, or SNFU.
(FAII p455-456)
nortriptyline, selegiline, bupropion, mirtazapine, fluvoxamine, doxepin, phenelzine, fluoxetine,
clomipramine, imipramine, amitriptyline, nefazodone, milnacipran, desipramine, sertraline, venlafaxine.
paroxetine, tranylcypromine, duloxetine, citalopram, desvenlafexine, trazodone
SSRI NDRI

TCA

MAOI Tetracyclic

13. Which antidepressant matches the following statement?


• SE priapism
• Lowers the seizure threshold
• Works well with SSRIs and increases REM sleep
• Appetite stimulant that is likely to result in weight gain
• Can be used for smoking cessation
• Can be used for bedwetting in children

14. What are the symptoms of TCA overdose?

15. What are the symptoms of serotonin syndrome?

Copyright © 2012. Doctors In Training.com. LLC. All Rights Reserved.


uestion Warm-Up
What are the signs of vitamin A deficiency?

What are the major signs of vitamin C deficiency (scurvy)?

If \ty$jiTar.(£i!l^

Anxiety Disorders
H: Chapter 391
Panic disorder (FAII p446) (FAI2 p490) (H p3529)
Specific phobias (FAII p446) (FAI2 p490) (H p3533)
Obsessive-compulsive disorder (FAII p447) (FAI2 p490) (H p3535)
Post-traumatic stress disorder (FAII p447) (FAI2 p49l) (H p3534)
Generalized anxiety disorder (FAII p447) (FAI2 p49l) (H p3530)
- Buspirone (FAII p454) (FAI2 p 499) (GGp349)
Malingering (FAII p447) (FAI2 p49l)
Factitious disorder (FAII p447) (FAI2 p49l)
Somatoform disorders (FAII p447) (FAI2 p49l) (H p354l)

.ViM^f^^'^fcifn-i.-cl-S-.l^i^l Irdlfr >','E(ia 'Sr«: ^Ir?- 'V.'^lrilnTon


because of claustrophobia. What can you do to help this patient?

A young woman that is anxious about receiving her first pap smear is told to relax and
imagine going through the steps of the exam. What does this process exemplify?

6. A woman has flashbacks about her boyfriend's death one month ago following a hit-
and-run accident. She often cries and wishes for the death of the culprit. What is the
diagnosis?

7. A nurse has episodes of hypoglycemia. Blood analysis reveals no elevation in C-proteir


What is her diagnosis?

8. What somatoform disorder matches the following description? . . .


• Unexplained pain
• Patient with normal anatomy is convinced a part of their anatomy is abnormal
• Unexplained loss of sensory or motor function (tests and PE are negative)

• Unexplained complaints in multiple organ systems


• False belief of being pregnant
I* Unwavering belief by the patient that she has a specific disease (despite medical reassurance)

Copyright© 2012. Doctors In Training.com. LLC. All Rights Reserved.


Ego Defense Mechanisms and Personality Disorders
H: Chapter 391

Anxiety self-treatment
- Ego defense mechanisms (FAII p440) (FAI2 p482)
- Alcohol
- Escapism (video games, TV)
Personality disorders (FAII p448) (FAI2 p492-493) (H p3542)
Review of psychiatric pharmacology (FAII p452) (FAI2 p497)

9. Which mature defense mechanism fits the following statement?


• Voluntarily choosing not to think about a piece of bad news
• Indiana Jones using humor to express feelings of discomfort
• Arsonist donates money to the fire department
• Using one's aggression to succeed in business ventures
• Realistically planning for future inner discomfort
• Consdously postponing an inner conflict until after a big project is completed
• Redirecting impulses to a socially favorable object

10. Which immature defense mechanism fits the following statement?


Not acknowledging a piece of bad news, as though it was never said

Involuntary withholding of a feeling from consdence awareness


A veteran that can describe horrific war details without any emotion
A child abuser was himself abused as a child

Man yells at his family when he has a bad day at work


A doset homosexual hates homosexuals because of the way they "make" him feel

Using intellectual processes to avoid affective expression; Dr Frasier Crane


Belief that people are either all good or all bad

Expressing aggression through passivity, masochism, and turning against self


Believing an external source is responsible for an unacceptable inner impulse
Changing ones character or personal identity to avoid emotional distress
Returning to an earlier level of maturation to avoid the conflict at the current maturational level
(stressed children wet the bed)

• Offering an explanation for an unacceptable attftude/belief/behavior

>s. • A thought that is avoided is replaced by an unconscious emphasis on the opposite


<D • Converting mental conflicts into bodily symptoms
X • Temporarily inhibiting thinking but continuing to build more tension

<
. • Avoiding interpersonal intimacy to resolve conflict and obtain gratification
- • Extreme forms can result in multiple personalities
__ • Chronically giving into an impulse to avoid tension from an unexpressed unconscious wish (tantrums)

u
>-
CO
Q_

[16] Copyright © 2012. Doctors In Traimng.com, LLC. All Rights Reserved.


11. What personality disorder fits the following statement?
• Excessive need to be taken care of, submissive and clinging behavior, low self-confidence, fears of
separation and losing support
• Grandiosity, feels he is entitled to things, lack of empathy
• Suicide attempts (■* 1596 mortality), unstable mood and behavior, sense of emptiness and loneliness,
impulsiveness
• Distrustful, suspicious, litigious
• Lifelong voluntary social withdrawal, no psychosis, emotional expression is limited (restricted range of
affect)
• Feelings of inadequacy, hypersensitive to rejection or criticism, socially inhibited, shy
• Constant mood of unhappiness and pessimism
• Odd appearance, thoughts, and behavior no psychosis; social awkwardness
• Controlling, perfectionistic, orderly, stubborn, indedsfve
• Criminality, unable to conform to social norms, disregard for others' rights
• Excessively dramatic emotional, and extroverted; sexually provocative behavior; unable to maintain
intimate relationships

-u
CO
- <
n
ON

>
D
X
£
Copyright© 2012. Doctors In Training.com. LLC. All Rights Reserved. [17]
What defense mechanism fits the following statement. , Wlffinmi
• Voluntarily choosing not to think about a piece of bad news
• Not acknowledging a piece of bad news, as though it was never said
• Involuntary withholding of a feeling from conscience awareness
• A veteran that can describe horrific war details without any emotion
• A child abuser was himself abused as a child
• Man yells at his family when he has a bad day at work

. A 65-year-old asks her husband to stay in the hospital overnight with her because she is
afraid to be alone. What defense mechanism is she exhibiting?

. Which defense mechanism underlies all other defense mechanisms?


1
. A 60-year-old man, admitted for chest pain, jumps out of bed and does 50 push-ups to
show the nurses he has not had a heart attack What defense mechanism is he using?

. A 40-year-old woman tells you during one of her office visits that she is in love with
you. You refer her to someone else, and she attempts suicide. What type of personality
disorder does this patient have?

17. A 30-year-old woman tells you during one of her office visits that you are the best

she threatens to change doctors because you do not feel a particular lab test is necessary.
(doctor she's ever
Additionally, youhad but several
notice that your nurse is very
symmetrical cutsdisrespectful. On a subsequent
on her left forearm visit,
which she attributes
to cat scratches. What type of personality disorder does this person have?

and black feather boa. She also is wearing an excessive amount of lipstick, and you notice
her having conversations with many of the other patients in the waiting room. What
type of personality disorder do you suspect in this patient?

A person demands only the best and most famous doctor in town. What is the
personality disorder does this person have?

What are the Cluster A personality disorders?

I A 55-year-old
What are thewoman
Clustercomes
B personality
to your disorders?
office wearing all black including a black miniskirt

12. What are the Cluster C personality disorders?

Copyright © 2012, Doctors In Training.com. LLC. All Rights Reserved.


uestion Warm-
What pathology fits the following phrase? p5l8-526) (FAQ p580-598)
Antiplatelet antibodies
- Bamboo spine on X-ray
' Webbed neck, short stature
j Painful, raised lesions on palms + fever

Dry eyes, dry mouth, arthritis


- Posterior cervical adenopathy
• Low serum ceruloplasmin

What type of antipsychotic is often the first line of treatment for psychosis? Which
antipsychotic should be reserved for severe, refractory cases because of agranulocyto
(FAII p453) (FAQp498)

Which skin disorder matches the following statement? (FAI I p388-389) (FAQ p424-427)
• Pruritic, purple, polygonal papules
• Pruritic vesicles assodated with celiac disease
• Thickened scar esp. around face/chest
• Parakeratotic scaling
• Keratin-filled cysts
• Skin rash and proximal muscle weakness
• Honey crusting lesions common about the nose and lips
• Hyperkeratosis and koilocytosis

Anatomy and Embryology


R: Chapter 20
Phys: Chapter 25
Anatomy
- Kidney anatomy and glomerular structure (FAII p458) (FAI2 p504)
- juxtaglomerular apparatus (jGA)
- Ureters: course (FAII p458) (FAI2 p505)
Embryology
- Kidney embryology (FAII pl32) (FAI2 pl40)
- Potter syndrome (FAII pi32) (FAI2 pi40)
- Horseshoe kidney (FAII pl32) (FAI2 pl40)
Fluid compartments (FAII p459) (FAI2 p505)

Copyright © 2012, Doctors In Training.com. LLC. All Rights Reserved.


What artery prevents a horseshoe kidney from ascending in the abdomen?

A 40-year-old patient of yours weighs 100 kg. What is her estimated plasma volume?
(FAI I p459) (FAQ p505) (FAQ p505)

What fundamental problem creates Potter syndrome? (FAII pl32) (FAQ pl40)

What cell type releases renin? (FAI I p463) (FAQ p5IO)

In what manner do the ureters cross the uterine artery which helps you identify these
structures during pelvic surgery? (FAI I p458) (FAQ p5IO)

Glomerulus
Phys: Chapters 25-26
GG: Chapter 25

Glomerular filtration barrier (FAII p459) (FAI2 p505)


Renal clearance (FAII p459) (FAI2 p506)
Glomerular filtration rate (GFR) (FAII p459) (FA/2 p506)
Effective renal plasma flow (ERPF) (FAII p459) (FA12 p506)
Filtration (FAII p460) (FAI2 p5l2)
Changes in glomerular dynamics (FAII p460) (FAI2 p507)
Calculation of reabsorption and secretion rate (FAII p460) (FAI2 p507)
Glucose clearance (FA 11 p460) (FA 12 p507)
Amino acid clearance (FAII p460) (FAI2 p507)

9. How will the following changes affect RBF, GFR, and filtration fraction?
(FAII p458) (FAI2p507)
GFR RBF FF (GFR/RBF)
Constriction of the afferent arteriole
Constriction of the efferent arteriole
Dilation of the afferent arteriole
Dilation of the efferent arteriole
Increase in serum protein
Ureter stone obstruction
ACE inhibitors
Indomethacin, Naprosyn, ibuprofen

Copyright © 2012. Doctors In Training.com. LLC All Rights Reserved.


What effect will a renal stone that obstructs the ureter have on GFR and FF?

tat is the maximal serum glucose concentration at which glucose can be absorbed ii
the tubules?

What vitamin deficiency results from Hartnup disease? (FAI I p460) (FAQ p507)

What substances can be used to estimate GFR? What substances can be used to
estimate renal plasma flow? (FAI I p459) (FAQ p506)

What is the equation for the renal clearance of any substance? (FAI I p459) (FAQ p50

Copyright © 2012. Doctors In Trainmg.com. LLC. All Rights Reserved.


What agents can be used to treat osteoporosis.

A child exhibits proximal muscle weakness and enlarged calves. What is the disease a.
how is it inherited? (FAI I p87) (FAQ p9l)

3. What are 4 types of epithelial cell junctions? What are 4 proteins involved in non-
epithelial adhesion mechanisms? (FAI I p370) (FAQ p404)

Nephron Physiology
Phys: Chapters 27-28
GG: Chapter 25

Nephron physiology (FAII p46l) (FAI2 p508)


Proximal tubule
Relative concentrations along proximal tubule (FAII p462) (FAI2 p509)
Thin descending loop ofHenle
Thick ascending loop ofHenle
Early distal convoluted tubule
Collecting tubule

4. Proximal Tubule (first half)

HC03-
carbonlc
anhydrase

C02 + H20
3HC03"
HCO3-,
Lumen C02 + H20

where X is glucose, amino acids, Pi, or lactate

Copyright ©2012. Doctors In Training.com, LLC. All Rights Reserved.


5. Proximal Tubule (second half)

CI" _ _
Na+

r T

Lumen Interstitiu

Anion"
J
J 1| ▶
a-X
CI".__J
Na+ f

6. Proximal Tubule (anions and cations)

organic anion organic anion organic anion


-' alpha-ketoglutarate

Lumen Interstitii

organic cation organic cation

7. Thick Ascending Limb (TAL)

Lumen carbonic' Interstiti


anhydrasal

Ca++'

Copyright © 2012. Doctors In Training.com. LLC. All Rights Reserved.


8. What class of drugs inhibits the Na72CI7K+ symporter in the thick ascending limb? (FAI I p473)

9. Early Distal Tubule

Lumen

H20

10. What determines how much water is reabsorbed in the distal tubules and the
collecting ducts?

11. What two types of cells compose the collecting duct and the last segment of the distal
tubule? What do they do?
• Principle cells -
• Intercalated cells -

12. What are the two types of intercalated cells?

in
.y 13. What class of diuretic directly affects principle cells?
i

.z> 14. What affect does aldosterone have on the intercalated cells and principle cells of the
O collecting duct?
! ' • Intercalated cells-
• Principle cells-

15. What drug antagonizes aldosterone's action on the principle cells of the collecting duct,
, thereby promoting Na+ excretion and inhibiting K+ excretion?

16. What are the critical steps involved in excreting dilute urine?
• Dilution of fluid in the thick ascending segment (to 100 mOsm/kg H2O) as solute is reabsorbed and
water remains in lumen (due to the impermeability of water in the thick ascending limb)
• The absence of ADH renders the distal tubule and cortical collecting duct impermeable to water
• Tubular fluid is diluted even more as solute is removed from the tubular fluid in the distal tubule and
cortical collecting duct but water remains
• Because of the low fluid osmolality in the collecting duct and the slight permeability of the medullary
collecting duct to urea, urea enters the tubule from the medullary interstitium thereby keeping the
osmolality of the medullary interstitium low
Copyright © 2012, Doctors In Training.com. LLC. All Rights Reserved.
17. What are the critical steps involved in excreting concentrated urine?
1) Dilution of fluid in the thick ascending segment (to 100 mOsm/kg H2O) as solute is reabsorbed
and water remains in lumen (due to the impermeability of water in the thick ascending limb)
- The reabsorption of solute without water in the thick ascending limb helps to increase the
osmolality in the interstitium
2) The presence of ADH renders the distal tubule and collecting duct permeable to water
- As water leaves the tubular fluid to an interstitium of high osmolality, the osmolality of the tubular
fluid increases (and equals the osmolality of the interstitium)
3) The presence of ADH increases the permeability of the last portion of the medullary collecting
duct to urea
- As the tubular fluid has a high concentration of urea (due to the reabsorption of water in the initial
segments of the collecting duct and impermeability of those segments to urea), urea enters the
interstitium as it goes down its concentration gradient from the last portion of the medullary
collecting duct
- As urea leaves the last portion of the medullary collecting duct (to go into the interstitium) and
enters the loop of Henle (from the interstitium), it becomes more and more concentrated within
the interstitium thereby increasing the osmolality of the interstitium
- This high osmolality serves to concentrate the urine in the collecting ducts (which are permeable
to water which allows the fluid in the cortical collected duct to achieve the same osmolality as the
fluid in the medullary interstitium)

What segment of the renal tubule matches the following statements? (FAI I p46l) (FAQ p508)
• Reabsorbs 67% of the fluid and electrolytes filtered by the glomerulus
• Segment responsible for concentrating urine
Site of secretion of organic anions and cations
- Always impermeable to water
• Permeable to water only in the presence of ADH

Site of the Na+/2C17K+ co-transporter


Site of isotonic fluid reabsorption
Site responsible for diluting urine
Only site where glucose and amino acids are reabsorbed
Water reabsorption in the Loop of Henle

What are the two main cell types of the collecting duct?

20. What affect does aldosterone have on the intercalated cells and principle cells of the

• Intercalated cells-
• Principle cells-

What class of drugs inhibits the Na+/2Q"/K+ symporter in the thick ascending limb?
(FAII p473)
I collecting (FAQ p524)
duct?

Copyright© 2012. Doctors In Training.com. LLC. All Rights Reserved.


Diuretics
GG: Chapter 25
Phys: Chapter 31
Site of action (FAII p473) (FA 12 p524)
Mannitol (FAII p474) (FAI2 p526)
Acetazolamide (FAII p474) (FAI2 p526)
Furosemide (FAII p474) (FAI2 p526)
Ethacrynic acid (FAII p474) (FAI2 p525)
Hydrochlorothiazide (FAII p474) (FAI2 p526)
K*-sparing diuretics (FAII p47S) (FAI2 p526)
Diuretics: electrolyte changes (FAII p475) (FAI2 p527)

What type of diuretic is the following drug? (FAI I p473-475) (FAQ p524-526)
• Triamterene • Mannitol
• Acetazolamide • Metolazone
• Hydrochlorothiazide • Chlorthalidone
• Bumetanide • Furosemide
• Spironolactone • Amiloride
• Chlorothiazide • To r s e m i d e
• Ethacrynic acid

. What diuretic or class of diuretic would be most useful in the following situation?
(FAI I p474-475) (FAQ p524-526)
• Acute pulmonary edema
■•• Idiopathic hypercalciuria (■» calcium stones)
• Glaucoma
• Mild to moderate CHF with expanded ECV
• In conjunction with loop or thiazide diuretics to retain K+
• Edema associated with nephrotic syndrome
• Increased intracranial pressure
• Mild to moderate hypertension
• Hypercalcemia
• Altitude sickness
• Hyperaldosteronism

What is the site of action of mannitol? (FAI I p474) (FAQ p525)

What is the site of action of the thiazides? (FAI I p474) (FAQ p474)

A patient with heart failure exacerbation needs medical diuresis but has a sulfa allergy.
What diuretic can be used?

Copyright © 2012. Doctors In Trainmg.com, LLC. All Rights Reserved.


ues
A patient with recent kidney transplant on cyclosporin for immunosuppression requii
an antifungal agent for candidiasis. What antifungal agent would result in cyclosporin
toxicity?

What is the most common cause of the following? (FAI I p529) (FAQ p594-596)
• Hypoparathyroidism
• Metastatic disease to brain
• Lysosomal storage disease
• Myocarditis

For what type of information is the following thalamic nucleus a relay station?
• Ventral posterior lateral
• Lateral geniculate
• Ventral posterior medial
• Ventral anterior

Renal Metabolic Basics


R: Chapter 20
Phys: Chapters 29-30
H: Chapter 45, 47

Kidney endocrine functions (FAII p463) (FAI2 pSIl)


Hormones acting on kidney (FAII p464) (FAI2 p5l2)
Potassium shifts (FAII p464) (FAI2 pSI3)
Electrolyte disturbances (FAII p464) (FAI2 pSI3)
Acid-base physiology (FAII p465) (FA/2 p5l4)

4. Plasma osmolality = 2[Na+]Piasma + [glucose]/!8 + [BUNJ/2.8

5. Potassium Shifts (FAI 1 p464) (FAQ p5l2)


K* shift out of cells ■» Hyperkalemia IC1" shift into cells ■» Hyperkalemia
• Low insulin • Insulin
• Beta-blockers • 8-agonist
• Acidosis • Alkalosis
• Digoxin • Cell creation proliferation
• Cell lysis (i.e. leukemia)

Copyright © 2012. Doctors In Training.com. LLC. All Rights Reserved.


for status asthmaticus?

What factors/substances cause hyperkalemia?

8. What factors/substances cause hypokalemia?

A patient presents with hypertension, hypokalemia, metabolic alkalosis, and low plasma
renin. What is the diagnosis, and how do you treat it?

What are the actions of angiotensin II? (FAI I p462) (FAQ p5IO)

Which electrolyte disturbance fits the following presentation? (FAI I p464) (FAQ p5l3)
• Correcting too rapidly may result in central pontine myelinosis
• Peaked T waves
• Tetany
• Arrhythmias
• Decreased deep tendon reflexes
• Flattened T waves, U waves on EKG

pH and ADH
Phys: Chapters 29-30
H: Chapter 45, 47
Acidosis/alkalosis (FAII p463) (FAI2 p5l3)
Renal tubular acidosis (RTA) (FAII p466) (FAI2 p5l4)
Diabetes insipidus (FAII p300) (FAI2 p300)
SIADH (FAII p300) (FAI2 p328)
Demeclocycline (FAII p305) (FAI2 p333)

Copyright ■'»> 2012. Doctors In Training.com. LLC. All Rights Reserved.


Normal Ranges (use for the question below)
pH 7.35-7.45
pCC>2 35-45 mmHg
(45 x 2 = 90) p02 75-105 mmHg
(45/2 = 22.5) HCCV 22-28 mEq/L

12. Determine what is wrong in patients with the following lab values:
1 H ' HC03- I pC02 i Type of Acid-Base Disorder

(Please note that the above values only reflect high and low values and may not accurately reflect
values in appropriate compensatory mechanisms.)

mi' M»T k^T:+> i [• 11 M.*i 'lr


13. How does acidosis/alkalosis affect extracellular K+ concentrations?

Determine what is wrong in a patient with the following lab values:


pH HCOr pCOl Problem
750 35 42
7.33 1
3 28
7.20 18 40
7.66 36 30
7.47 14 22
7.10 1
5 50

What are the causes of acidosis with an elevated anion gap? (FAI I p465) (FAQ p5l4)

ostic features of diabetes Ml p300)(FAI

Copyright :) 2012. Doctors In Traimng.com. LLC. All Rights Reserved.


What nerve is damaged when a patient presents with the following symptom (upper
extremity)? (FAI I p374) (FAQ p4IO)
• Scapular winging
• Loss of forearm pronation
• Cannot abduct or adduct fingers
• Weak lateral rotation of arm
• Unable to abduct arm beyond 10 degrees

What are the most common causes of hypocalcemia?

Label the following Gram positive algorithm.

Gram (+) Algorithm

Copyright © 2012. Doctors In Training.com, LLC. All Rights Reserved.


Nephritic Syndrome
R: Chapter 20
H: Chapter 283
Nomenclature of glomerular disorders (FAII p466) (FAI2 p5l5)
Nephritic syndrome (FAII p467) (FAI2 p5l6)

What are the defining features of nephritic syndrome? (FAI I p467) (FAQ p467)

Why is it important to treat Strep throat in children or in those that could transmit St
pyogenes to a child?

Which glomerular disease would you suspect most in a patient with the following findings:
(FAI I p466-468) (FAQ P5I5-5I8)
• Granular pattern of immune complex deposition; LM: hypercellular glomeruli
• Linear pattern of immune complex deposition
• Deposition of IgG, IgM, IgA, and C3 in the mesangium
• Anti-GBM antibodies, hematuria, hemoptysis
• Nephritis, deafness, cataracts
• Crescent formation in the glomeruli
• Wire-loop appearance

Nephrotic Syndrome
R: Chapter 20
H: Chapter 283

Nephrotic syndrome (FAII p468) (FAI2 p468)

Copyright•;-•) 2012. Doctors In Trainmg.com. LLC. All Rights Reserved.


Which glomerular disease would you suspect most in a patient with the following findin
(FAI I p466-468) (FAQ P5I5-5I8)
• Most common nephrotic syndrome in children
• IF: granular pattern of immune complex deposition; LM: diffuse capillary thickening
• Kimmelstiel-Wilson lesions (nodular glomerulosclerosis)
• Most common nephrotic syndrome in adults
• EM: loss of epithelial foot processes
• Nephrotic syndrome associated with hepatitis B
• Nephrotic syndrome associated with HIV
• EM: subendothelial humps and tram-tack appearance
• LM: segmental sclerosis and hyalinosis
• Purpura on back of arms and legs, abdominal pain, IgA nephropathy
• Apple-green birefringence with Congo-red stain under polarized light
• EM: spiking of the GBM due to electron dense subepithelial deposits

Glomerular histology reveals multiple mesangial nodules. This lesion is indicative of what
disease?

A teenager presents with nephrotic syndrome and hearing loss. What is the disease?

A 4-year-old boy presents with facial edema and proteinuria. What is the appropriate
treatment?

Copyright C 2012. Doctors In Tr.iimng.com. LLC. All Rights Reserved.


arm-
A patient taking lisinopril complains of new onset, constant coughing. To what medication
class should this patient be switched?

What pathology is associated with the following high-yield phrase? (FAI I p518-522) (
p580-584)
• "Worst headache of my life"
• Waxy casts in urine
• Neuropathy + AV nodal block
• Port-wine stain in the ophthalmic division of the trigeminal nerve
• Urethritis, conjunctivitis, arthritis
• Painless jaundice

To what drug category does the following drug belong?


Azathioprine 6-mercaptopurine
Probenecid Rofecoxib
Primaquine Carmustine
Cefprozil Doxycydine
Lamivudine Timolol
Tobramycin Methotrexate
Losartan Cimetidine
dinavir Mefloquine

Other Renal Pathology I


R: Chapter 20
H: Chapters 94, 285, 287, 288
Casts in urine (FAII p466) (FAI2 pSIS)
Kidney stones (FAII p469) (FAI2 p5l9)
Renal cell carcinoma (FAII p469) (FAI2 p520)
Wilms tumor (FAII p469) (FAI2 p520)
Transitional cell carcinoma (FAII p470) (FAI2 p520)
Pyelonephritis (FAII p470) (FAI2 pS2l)
Drug-induced interstitial nephritis (FAII p470) (FAI2 p470)

4. Under what circumstances would you see the following type of cast?
(FAII p466) (FAI2p5l5)
• RBC cast
• WBCcast
• Bacterial cast
• Epithelial cell cast
• Waxy cast
• Fatty cast
• Granular cast

Copyright © 2012. Doctors In Training.com. LLC. All Rights Reserved.


5. Acute Interstitial Nephritis (AIN) (FAII p470) (FAQ p52l)
• Results in acute renal failure (ARF)
• Classic presentation: fever, rash, eosinophilia, and azotemia
• Most common cause is drug-induced (NSAIDs, PCN/cephalosporins (esp. methicillin)), sulfonamides
(e.g. TMP-SMX, furosemide), ciprofloxacin, cimetidine, allopurinol, PPIs, indinavir, mesalamine)
• Rx: 2 weeks of corticosteroids

Other Renal Pathology 2


R: Chapter 20
H: Chapters 279-282, 284
Diffuse cortical necrosis (FAII p470) (FAI2 p52l)
Acute tubular necrosis (FAII p470) (FAI2 p52l-522)
Renal papillary necrosis (FAII p470) (FA 12 p522)
Acute renal failure (FAII p47l) (FAI2 p522)
Consequences of renal failure (FAII p47l) (FAI2 p522)
Renal cysts (FAII p472) (FAI2 pS23)

10. Causes of ATN (FAI I p47l) (FAQ p523)


• Drugs: aminoglycosides, cephalosporins, polymyxins
• Radiograph contrast dye (prevent with N-acetylcysteine, fluids, NaHC03)
• Rhabdomyolysis/myoglobinuria
- Due to muscle breakdown from seizure disorder, cocaine, or crush injuries
- Findings: 4+ blood in urine, no RBC on urine cell count, renal failure, elevated CPK

Copyright © 2012. Doctors In Trainnig.com. LLC. All Rights Reserved.


ssion Quiz
A patient reports a long-term history of acetaminophen use. What is she at increase
risk for?

2 What changes will be seen in a basic metabolic panel in a patient with renal failure?
(FAII p47l) (FAQp523)

A CT scan reveals massively enlarged kidneys bilaterally. What is the diagnosis?

14. Renal pathology rapid review: (FAI I p53l) (FAQ p592-596)


• Most common tumor of the urinary tract system
• Most common renal malignancy of early childhood (ages 2-4)
• Histologic appearance of renal cell carcinoma
• Histological appearance of chronic pyelonephritis
• Fever + rash + hematuria + eosinophilia
• Cancer associated with Schistosoma haematobium
• Treatment for cystine kidney stones

Copyright© 2012, Doctors In Training.com. LLC. All Rights Reserved.


I. What personality disorder fits the following statement? (FAI I p447-448) (FAQ p492-493)
1 Excessive need to be taken care of, submissive and clinging behavior
j Low self-confidence, fears of separation and losing support
' Grandiosity, feels he is entitled to things, lack of empathy
j Suicide attempts (■▶ 15% mortality), unstable mood and behavior
' Sense of emptiness and loneliness, impulsiveness
Odd appearance, thoughts, and behavior; no psychosis; social awkwardness
Controlling, perfectionistic, orderly, stubborn, indecisive
Criminality, unable to conform to social norms, disregard for others' rights

What pathology matches the following statements?


Antiepfthelial cell antibodies
Anti-basement membrane
Cough, conjunctivitis, coryza, fever
Councilman bodies
Green/yellow pigment just within the corneoscleral margin
Anticentromere antibodies
Dementia + eosinophilic inclusions in neurons
Anti-dsDNA antibodies (ANA antibodies)

What are the different causes of post-op fever? (FAI I pi78) (FAQ pl98)

Reproductive Anatomy
R: Chapters 21. 22
H: Chapters 346, 347
Male reproductive anatomy (FAII p479) (FAI2 p53l) (H p30IO)
Female reproductive anatomy (FAII p478) (FAI2 p530) (H p3028)
Female reproductive histology (FAII p478) (FAI2 p530)
Innervation of the male and female genitals (COA p4ll)
Pudendal nerve block (COA p433)
Autonomic innervations of the male sexual response (FAII p479) (FAI2 p53l)
Gonadal venous drainage (FAII p478) (FAI2 p530)
Inguinal canal (FAII p3l5) (FAI2 p344)
Hernias (FAII p3l6) (FAI2 p345)

Copyright <<s 20I2. Doctors In Training.com, LLC. All Rights Reserved.


What cell type lines the uterus? (FAI I p478) (FAQ p530)

What ligament of the female pelvis matches the following description?


(FAII p478) (FAQ p478)
• Connects the cervix to the pelvic side wall
• Connects the ovaries to the lateral pelvic wall
• Connects the uterus, fallopian tubes and ovaries to the pelvic side wall
• Contains the uterine vessels
• Contains the ovarian vessels

What is the landmark used for the pudendal nerve block? (COA p433)

To where does testicular cancer first metastasize? (FAI I p478) (FAQ p530)

Reproductive Embryology
COA: Chapter 3
H: Chapter 349
Genital embryology (FAIIpl33) (FA12 pi42)
SRYgene (FAII p486) (FAI2 pl42)
Bicornuate uterus (FAII pl33) (FAI2 pl4l)
Male/female genital homologues (FAII pl34) (FAI2 pl42)
Congenital penile abnormalities (FAII pl34) (FAI2 pl43)
Descent of testes and ovaries (FAII pl34) (FAI2 pl43) (COA p205)

8. Exstrophy of the bladder - congenital gap in the anterior bladder wall and abdominal
wall in front of it ■▶ exposure of the bladder interior to the outside world

What structures develop from the mesonephric duct system? (FAI I pl32) (FAQ pl4l-l42)

10. What is the male homologue to the following female structure? (FAI I pl34) (FAQ pl43)

• Labia minora
• Bartholin glands
• Urethral and paraurethral glands (of Skene)

What gene product comes from the SRY gene that underlies male genital development?
(FAII p486) (FAQ pl42)
i* Vestibular bulbs

4) (FAQ pi

Copyright @ 2012. Doctors In Training.com. LLC. All Rights Reserved.


What is the most common: (FAI I p528) (FAQ p580) (H pl929, Table 237-1)
• Cause of DIC
• Heart murmur
• Coronary artery involved in thrombosis
• Cause of death in lupus patients
• Congenital heart anomaly

What are the positive symptoms of schizophrenia? What are the negative symptoms
schizophrenia? (FAI I p444) (FAQ 487,493) (H p3542)

What is the WAGR complex? (FAI I p469) (FAQ p520) (R p440,479)

Androgens
H: Chapter 346
Phys: Chapter 80
Seminiferous tubules (FAII p480) (FAI2 p532)
Spermatogenesis (FAII p48l) (FAI2 p533)
Regulation of spermatogenesis (FAII p48l) (FA/2 p533)
Control of reproductive hormones (Male) (FAII p497) (FAI2 p553)
Androgens (FAII p482) (FAI2 p534)
Testosterone (methyltestosterone) (FAII p497) (FAI2 p553)
Antiandrogens (FAII p498) (FAI2 p553) (GG pl204)
Androgen insensitivity syndrome (FAII p486) (FA/2 p540) (H p305l)
S-alpha reductase deficiency (FAII p486) (FAI2 p540)
Pseudohermaphroditism (FAII p486) (FAI2 p539)
True hermaphrodite (FAII p486) (FAI2 p540) (H p3050)

Copyright ©2012. Doctors In Training.com, LLC. All Rights Reserved.


How many carbon mole
(FAI I p29l) (FAQ p292) (Phys p980 Fig. 80-8)

What is the difference between androgen insensrtivity and 5-alpha-reductase deficiency?


(FAII p486) (FAQ p540)

What is the role of Sertoli cells and Leydig cells in spermatogenesis?


(FAII p48l)(FAI2p48l,533)

,'. What reproductive pathology matches the following statement?


• Female with short stature + no Barr body
• Chromosomal XXY
• Chromosomal XO
• Presence of ovaries, but male genitalia
• Unable to generate DHT
• Both ovarian and testicular tissues are present
• Webbing of the neck
• Male with Barr body in PMNs
• Ambiguous genitalia until puberty, then masculinization

Testes Pathology
H: Chapters 96, 346
Epididymitis (R p986)
Torsion (R p987) (H pl097)
Cryptorchidism (FAII p495) (FAI2 p55l) (R p984)
Testicular germ cell tumors (FAII p496) (FAI2 p552) (R p987)
Testicular non-germ cell tumors (FAII p496) (FA12 p552) (R p992)
Tunica vaginalis lesions (FAII p496) (FAI2 p552) (R p993)

8. Epididymitis
• Inflammation of the epididymis
• Dx: support of the testes ■» some relief
• Rx <35 = GC/Chlamydia
- Ceftriaxone IM then Doxycycline xlOd
• Rx >35 or h/o anal intercourse = Enterobacteriaceae
• Fluoroquinolone xl0-l4d

9. Testicular Torsion
• Twisting of the spermatic cord ■▶ ischemia
• Dx: support of testis ■» no relief; US
• Rx: surgical detorsion with bilateral orchiopexy within 6hrs

10. Cryptorchidism (FAI I p495) (FA p55l)


• Failure of testis to descend into scrotum
• Usually unilateral
• Descent usually complete in I st year of life
• 35x increased risk of malignant tumor in the undescended testicle (usually a germ cell tumor)

Copyright <n 2012, Doctors In Traimng.com. LLC. All Rights Reserved.


d of Session Quiz

(FAII p495)(FAI2p55l)

What testicular tumor is described by the following statement?


• Composed of cytotrophoblasts and syncytiotrophoblasts
• May present initially with gynecomastia
• Elevated AFP
• Elevated B-hCG
• Most common testicular tumor
• Most common testicular tumor in infants and children up to 3yrs of age
• Most common testicular tumor in men over age 60
• Histologic appearance similar to koilocytes (cytoplasmic clearing)
• Histologically may have alveolar or tubular appearance sometimes with papillary convolutions
• Composed of multiple tissue types
• Histologic endodermal sinus structures (Schiller-Duval bodies)
• 25% have cytoplasmic rod-shaped crystalloids of Reinke
• Androgen producing and associated with precocious puberty

Copyright© 2012. Doctors In Traming.com. LLC All Rights Reserved.


What pathology is associated with the following key wo.
(FAII p518-519) (FAQp580-584)
• Bilateral hilar lymphadenopathy
• Cherry-red spot on macula
• Slapped cheeks rash on child
• Organism associated with dog or cat bite
• Facial muscle spasm upon tapping the cheek
• Cough, conjunctivitis, coryza
• Nephritis, hearing loss, cataracts

When performing a lumbar puncture for anesthesia administration, where is the


anesthesia dosed? Where is CSF found? (COA p 505-506)

What is the female homologue to the following male structure?


(FAI I pi34) (FAQ pi42) (H p3049, Fig 349-3)
• Scrotum
• Prostate gland
• Glans penis
• Corpus spongiosum
• Bulbourethral glands
Vent

60 Penis
o
-pr COA: Chapter
Chapter 80 3
X Phys:
+=> H: Chapter 48
rd
0_ Sexual dysfunction (FAII p6l) (FAI2 p63) (H p374)
nj Erectile dysfunction (COA p427) (H p375)
-= Sildenafil, vardenafil (FAII p499) (FAI2 p556) (H p377)
■ Penile pathology (FAII p496) (FAI2 p552)
- Bowen disease (R p983)
! - Bowenoid papulosis (R p496)
~~ Condyloma acuminatum (R p983)
Balanitis (R p982)

Q 4. Erectile Dysfunction

b* Failure • to
Failure to fill(psychogenic,
initiate (atherosclerosis)
endocrinologic or neurogenic)
• Failure to store adequate blood volume within the lacunar network
• Diabetic, atherosclerotic and drug related causes account for >80% of cases of ED in older men

Q_ 5. Condyloma acuminatum
• Benign genital wart
• Caused by HPV 6 and 11

Copyright © 2012. Doctors In Training.com. LLC. All Rights Reserved.


lick Quiz
What is the mechanism of action of sildenafil? (FAI I p499) (FAQ p556)

What are the side effects of sildenafil? (FAI I p499) (FAQ p556)

What organism is commonly implicated in balanitis?

Prostate
H: Chapter 95
R: Chapters 21, 22
Prostatitis (FAII p494) (FAI2 p550) (R p993) (H p239l)
Benign prostatic hyperplasia (FAII p495) (FAI2 pSSI) (R p994)
Tamsulosin (FAII p499) (FAI2 p556) (GGp307)
Prostatic adenocarcinoma (FAII p495) (FAI2 p55l) (R p996) (H p799)

9. BPH - Benign Prostatic Hyperplasia (R p994)


• Present in 80% of men over age 80 y/o
• Diagnosis based on the following symptoms: sensation of incomplete voiding, increased urinary
frequency (less than q2hrs), straining to void, intermittent or weak urine stream, urgency, nocturia (at
least 2-3 times a night)
• Palpable prostate size may not correlate with degree of obstruction or symptom severity
• Alternative Medicine: Isoflavones, saw palmetto
• Medical Intervention
- Nonselective a-blockers: Doxazosin, Prazosin, and Terazosin
• Decrease prostate smooth muscle tone ■» immediate improvement in urine flow
• SE: dizziness, postural hypotension, fatigue, asthenia. To reduce SE, dose qHS and titrate dose
upward slowly overtime (weekly)
• Tamsulosin (Flomax) (selective a-1A—blocker) - fewer SE than non-selectives, has no
antihypertensive effects
- 5a-reductase inhibitors: Finasteride, Dutasteride
• slowly reduces dihydrotestosterone levels ■▶ 20% decrease in prostate volume over 3-6m
• SE: decreased libido, ejaculatory disorder, impotence
• Surgical Intervention

Copyright© 2012, Doctors In Trainmg.com. LLC. All Rights Reserved.


fi T F i i
1 How does flutamide differ from finasteri
use? (FAI I p498) (FAQ p 554)

A 55-year-old man undergoing treatment for BPH has increased testosterone and
decreased DHT as well as gynecomastia and edema. What is his medication?

kmvjilu are the diagnostic symptoms of BPH? (FAI I p495) (FAQ p55l)

Copyright © 2012. Doctors In Trainmg.com. LLC. All Rights Reserved.


Question Warm-Up
i?IH|ll»<'|["<*/-|iliV/*vm*JK*t*jlM»<Wllll^JPJHSJlkif*lfSli

What nerve damage causes carpal tunnel syndrome? (FAI I p372) (FAQ p407) (R pi266)

To what drug categories do the following drugs belong?


• Cholestyramine • Clarithromydn
• Venlafaxine
Venlafaxine • Famotidine
Fluphenazine Fluvoxamine
Phentolamine 6-mercaptopurine
Captopril Selegiline
Trazodone Terbinafme
Carteolol Clozapine

Female Reproductive Cycle


Phys: Chapter 81
H: Chapter 347
Control of reproductive hormones (female) (FAII p497) (FAI 2 p535)
Estrogen (FAII p482) (FAI2 p535) (GGp 11 6 3 ) [
Progesterone (FAII p483) (FAI2 p535) ~D
Menstrual cycle (FAII p483) (FAI2 p536)
Ovulation (FAII p484) (FAI2 p537) {
Oogenesis (FAI I p484) (FAI2 pS37) [
Pregnancy (FAII p484) (FAI2 p537) .
hCG(FAII p485) (FAI2 p538) )
( I l

4. Outline the hormone sequence of the female reproductive cycle. '.


(FAI I p483) (FAQ p536) (Phys p995) (
FSH ■» follicle maturation ■▶ production of estradiol ■▶ production of LH surge ■▶ ovulation and *_
production of progesterone (along with estradiol) ■▶ inhibition of FSH and LH production ■▶ decline of _^
corpus luteum ■» no production of estradiol and progesterone ■» loss of FSH inhibition ■» increase in
FSH (repeat step I) '
r~
5. Corpus Luteum (Phys p995)
• Formed after ovulation, produces progesterone and estrogen in the luteal phase q-
• Lifespan 13-14 days (luteal phase) '
• If BhCG from the placenta is present, the lifespan will extend to 6-7 weeks until the placenta is able to
produce its own progesterone.

6. What are the layers of the endometrium? Which layers are shed during menstruation? ~D

7. Which hypothalamic nucleus is involved in ovulation?

8. When does the basal body temperature increase occur in relation to ovulation?

Copyright© 2012. Doctors In Trainmg.com. LLC. All Rights Reserved


What is the two-cell theory of estradiol production? (FAI I p482) (FAQ p535)

What are the target cells of LH? What cells respond to FSH? (FAI I p48l) (FAQ p534)

State whether the following statement describes estrogen or progesterone.


(FAII p482-483) (FAQp535)
• Production of thick mucus that inhibits entry of sperm into the uterus
• Induces LH surge
• Uterine smooth muscle relaxation
• Follicle growth
• Maintenance of pregnancy
• Hepatic synthesis of transport proteins
• Withdrawal leads to menstruation

When does 3-hCG appear in the urine during pregnancy? (FAI I p483) (FAQ p537)

Hormonal Birth Control and Menopause


H: Chapters 347, 348
GG: Chapter 40
Hormonal birth control
- Oral contraception (FAII p499) (FAI2 p555) (Phys plOOl) (GG pi 185)
- Estrogens (FAII p498) (FAI2 p554) (GG p 1163)
- Progestins (FAII p498) (FAI2 p555) (GGpll87)
- Medroxyprogesterone (GG p 1187)
- lUDs
Menopause (FAII p485) (FAI2 p538) (R pl027)
Menopausal hot flashes
Hormone replacement therapy (FAII p498) (FAI2 p555) (GG pi 175)

13. Medroxyprogesterone (Depo-Provera) (GG p 1187)


• IM injection dosed q3m
• Associated with bone mineral density loss especially if long-term, therefore not ideal for >2yr of use
• Good choice of contraception for patients with MR

14. Intrauterine Device (IUD)


• Copper (Paragard) ■» 10 year
• Progesterone (Mirena) ■▶ 5 year, prevents menstruation, used as Rx for menorrhagia
• Small risk of uterine rupture when placing
• Contraindicated if high risk of STD

Copyright © 2012, Doctors In Training.com. LLC. All Rights Reserved.


15. Menopausal Hot Flashes (R pi027) (Phys p999)
• Occur in 75% of menopausal women
• Presentation: starts in face/chest then generalizes, lasts 2-4min, associated with diaphoresis and
palpitations, followed by chills and shivering
• May cause sleep disturbances
• Rx: Estrogen replacement > SSRI or SNRI (venlafaxine) > clonidine orgabapentin
• Herbal Rx: soy isoflavonesf, red cloverf, black cohoshf, vitamin E

f potential estrogenic effect on the breast, much like estrogen replacement

16. What are the pros and cons of oral contraceptive pill use?
(FAI I p499) (FAQ p554) (GG pi 187)

Why is p

What is the best option for birth control in a mentally retarded patient?

What hormonal changes are seen during menopause? (FAI I p485) (FAQ p536)

A 23-year-old female who is on rifampin forTB prophylaxis and on birth control (estrogen)
gets pregnant Why?

Copyright © 2012. Doctors In Training.com, LLC. All Rights Reserved


What pathology matches the following statement?
• Lens-shaped lesion on head CT
• Common underlying cause of intussusception
• No milk production in the postpartum period
• Pigmented hamartomas in the iris
• Howell-Jolly bodies
• Cancer associated with asbestos
• Owl's eye inclusions
• Owl's eye nucleus
• Owl's eye protozoan
• 50-year-old male with new, unexplained skin yellowing and no other symptoms

In which causes of vaginal discharge/vaginitis will the pH be high? In which will the pH be
low? (FAII pl8l)(FAI2p20l)

What structures arise from the paramesonephric ducts?


(FAII pl33) (FAQ pl4l) (H p3048)

Vaginal and Cervical Pathology


R: Chapter 22
H: Chapter 97
Female reproductive anatomy (FAII p478) (FAI2 p530)
Vaginismus and vestibulitis (COA p434)
Benign vulvar cysts
- Bartholin gland cysts (COA p433) (R plOII)
- Skene glands
Vulvar intraepithelial neoplasia (VIN) and vaginal intraepithelial neoplasia (VAIN) (R plOI2, 1016)
Vaginal carcinoma (FA II p492) (FAI2 p547) (R plOI6)
Cervical pathology (FAII p488) (FAI2 p543
Endometriosis (FA p489) (FAI2 pS43) (R pl028)

4. Female Reproductive Anatomy

Lippmcou vvutiams & vvnKins Alias 01 Anatomy Vs/zuuo vvoncrs Muwer neaiui. inc. /mi ngnts reserved.

Copyright © 2012, Doctors In Training.com. LLC. All Rights Reserved.


5. Vulvar Intraepithelial Neoplasia (VIN) (RplOQ)
• Very similar to CIN except vulvar location
• Grades I, II, III
• Associated with HPV (esp. 16, 18, and 31)
• Koilocytosis: squamous cell w/ perinuclear cytoplasmic clearing
• Precursor to vulvar carcinoma (FAI I p492) (FAQ p547)

What are the risk factors for cervical cancer? (FAI I p488) (FAQ p543)

What is the classic histological appearance of a vaginal epithelial cell infected with HP\
(FAII p488) (FAQ p543)

Who is at risk of clear cell adenocarcinoma? (FAI I p492) (FAQ p547)

Uterine Pathology
H: Chapter 97
R: Chapter 22

Endometrial proliferation (FAII p489) (FA12 p544) (Phys p995)


- Endometrial hyperplasia
- Endometrial carcinoma
Myometrial tumors (FAII p489) (FAI2 p544) (R pl03l)
- Leiomyoma
- Leiomyosarcoma
Leuprolide (FAII p497) (FA/2 p554) (GGpll22)
Gynecologic tumor epidemiology (FAII p489) (FAI2 p544) (H p8l3)

What are the risk factors for endometrial carcinoma?

What uterine pathology matches the following description? (FAI I p488) (FAQ p544,
• Excess unopposed estrogen is the main risk factor
• Menorrhagia with an enlarged uterus and no pelvic pain
• Pelvic pain that is present only during menstruation
• Diagnosed by endometrial biopsy in clinic
• Definitive diagnosis and treatment is by laparoscopy
• Menstruating tissue within the myometrium
• Malignant tumor of the uterine smooth muscle
• Most common gynecologic malignancy

l tev M" 1-•'■'■ 5 !2£ing of leuprolide affect its physiological impact7 (FAI I p497) (FAQ p5
1 n ISA^'SS

Copyright © 2012. Doctors In Training.com, LLC. All Rights Reserved.


What organism is associated with the following statement?
(FAI I pi54) (FAQ pi67) (R p358, Table 8-8)
• Cat scratch
• Dog/cat bite
• Cat feces
• Puppy feces
• Animal urine

To what classes of medication do the following drugs belong?


• Primaquine • Te m a z e p a m
• Saquinavir • Desipramine
• Betaxolol • Captopril
• Prazosin • Busulfan
• Thiopental • Moxifloxacin
• Tranylcypromine • Zanamivir
• Sertraline • Miconazole

Which defense mechanism fits the following description?


(FAI I p440) (FAQ p482) (H p3529)
• Involuntary withholding of a feeling from conscience awareness
• A veteran that can describe horrific war details without any emotion
• A child abuser was himself abused as a child
• Underlies all other defense mechanisms
• May lead to multiple personalities
• Adult whining, bedwetting, crying

Ovarian Pathology
H: Chapter 97
R: Chapter 22

Ovarian histology (R pi041)


Premature ovarian failure (FAII p489) (FAI2 p544)
Most common causes of anovulation (FAII p489) (FAI2 p544) (R pl026)
Polycystic ovarian syndrome (FAII p490) (FA12 p545)
Clomiphene (FAII p498) (FAI2 p555) (GGpll79)
Ovarian cysts (FAII p490) (FAI2 p545) (R pl038)

Copyright © 2012. Doctors In Training.com, LLC. All Rights Reserved.


Quick Quiz
What is the underlying cause o
treatment? (FAI I p490) (FAQ p545)

A patent with polycystic ovarian disease is most at risk for developing which type of cance

Under what circumstances would you expect to see an elevated LH?

What drug would you give to inhibit prolactin secretion? (FAI I p290) (FAQ p3l7)

Ovarian Tumors
H: Chapter 97
R: Chapter 22
Ovarian non-germ cell tumors (FAII p492) (FAI2 p547)
Ovarian germ cell tumors (FAII p49l) (FAI2 p546) (R pl047)

8. What are the 4 main categories of ovarian tumors? (FAI I p491 -492) (FAQ p546-547)
• Epithelial (65% of ovarian tumors, 90% of ovarian cancers)
• Germ cell
• Stromal
• Metastatic (Gl, breast, endometrium)

9. What are the main types of epithelial cell ovarian tumors (which account for 65% of
ovarian tumors and 90% of ovarian cancers)?
Serous, mucinous, endometrioid, clear cell, Brenner, mixed
(hint My Med Students Consistently Beat Exams.)

10. What are the main types of germ cell ovarian tumors?
Teratoma, dysgerminoma, endodermal sinus, choriocarcinoma

11. What are the main types of stromal/sex cord ovarian tumors?
Granulosa-theca cell, Sertoli-Leydig cell, fibroma

12. What ovarian tumor matches the following statement? (FA p49l-492) (FAQ p546-547)

Lined with fallopian tube-like epithelium


Psammoma bodies
Intraperitoneal accumulation of mucinous material
Resembles bladder epithelium
arm rp{\ ti imnrc:

Multiple different tissue types


Elevated B-hCG
Produces AFP
Ovarian tumor + ascites + pleural effusions
Estrogen-secreting, leading to precocious puberty
Call-Exner bodies
Testosterone-secreting, leading to virilization

Copyright© 2012, Doctors In Training.com. LLC. All Rights Reserved.


What ovarian tumor matches the following statement? (FAI I p490-492) (FAQ p546-547)
• Produces AFP
• Estrogen-secreting, leading to precocious puberty
• Intraperitoneal accumulation of mucinous material
• Testosterone-secreting, leading to virilization
• Psammoma bodies
• Multiple different tissue types
• Lined with fallopian tube-like epithelium
• Ovarian tumor + ascites + hydrothorax
• Call-Exner bodies
• Resembles bladder epithelium
• Elevated B-hCG

What are the risk factors for ovarian cancer?

An obese woman presents with amenorrhea and increased levels of serum testosterone.
What is the most likely diagnosis?

Copyright :. 2012. Doctors In Training.com. LLC. All Rights Reserved.


Question Warm-Up
What pathology fits the following high-yield phrase?
• Thyroid cells with optically clear nudei
• Anemia with hypersegmented neutrophils
• Branching rods on oral infection
• Eczema + recurrent infections + thrombocytopeni
• Hemosiderinuria + thrombosis

What are the stages of behavioral change? (FAI I p450) (FAQ p494)

Which primary bone tumor fits the following description? (FAI I p38l) (FAQ p
• Most common malignant primary bone tumor of children
• Most common benign bone tumor
• 11 ;22 translocation
• Soap-bubble appearance on X-ray
• Onion-skin appearance of bone
• Codman's triangle on X-ray

Pregnancy Complications
H: Chapter 7

Gravidity and parity


Twinning (FAII pl2l) (FAI2 pl28)
Amniocentesis and chorionic villus sampling (CVS)
Placental development (FAII pl22) (FAI2 pl29)
Umbilical cord (FAII pl22) (FAI2 pl29)
Hydatidiform mole (FAII p487) (FAI2 p540)
Common causes of recurrent miscarriages (FAII p487)
Infertility
Physiologic changes in pregnancy

4. In monozygotic twins, twin placentation is determined by timing of egg division:


• Diamniotic / dichorionic placentation- division occurs prior to morula stage (within 3 days of
fertilization)
• Diamniotic / monochorionic placentation- division occurs 4-8 days post-fertilization (blastocyst)
• Mono/mono (one placenta) occurs with division 8-12 days after fertilization (epiblast/hypoblast)
- One yolk sac and 2 fetal poles = monoamnionicity
- Cord entanglement = monoamnionicity
• Division at or after 13 days ■▶ conjoined twins

Copyright ©2012. Doctors In Training.com. LLC. All Rights Reserved


5. Amniocentesis
• Performed at 15-17 weeks for genetic evaluation (in third trimester for lung maturity eval)
• Indications:
- Evaluation of lung maturity (lecithin: sphingomyelin ratio a 2.5)
- > 35-year-old (offered for eval of possible trisomy/genetic defects)
- Abnormal maternal serum triple or quad screen
- In Rh-sensitized pregnancy to detect fetal blood type or fetal hemolysis
• Risks: I -2% maternal/fetal hemorrhage, 0.5% fetal loss

6. Chorionic Villus Sampling (CVS)


• Performed at 10-12 weeks
• Risks: 1% fetal loss, 1% inability to diagnose NTD, limb defects if done < 9 weeks

7. What are some of the normal physiologic changes that take place during pregnancy?
• Cardiac output increases 30-50%
• Plasma volume increases 50%, RBC volume increases 30%
• BP decreases in early pregnancy ■» nadir at 16-20wks ■» return to pre-pregnancy levels by term
• Increased minute ventilation ■» decreased PACO2 and PaC02, mild respiratory alkalosis ■» CO2
transferred more easily from fetus to mother
• Increased procoagulation factors ■» hypercoagulable state
• Increased GFR ■» decreased BUN and Cr
• Normal TSH and free T4
• Increased peripheral resistance to insulin (due to human placental lactogen) that worsens throughout
pregnancy ■▶ hyperinsulinemia, hyperglycemia, hyperlipidemia

>^
1 :8.and
A pregnant woman
hypertension. at 16abnormality
What weeks of gestation
might bepresents
seen onwith an test,
blood atypically large is
and what abdomen
the
disorder?

A 15-year-old female patient of yours that normally comes with her parents presents
alone this time. She states that she is sexually active but that she knows she is not
pregnant because she has never menstruated. What would be the appropriate next step
in managing this patient?

Monozygotic twins are delivered. One is pale and has a hematocrit of 15%, and the other
is flushed with a hematocrit of 55%. What is the cause of these features?

Copyright © 2012. Doctors In Trainmg.com, LLC. All Rights Reserved.


Labor and Delivery
H: Chapter 7

Pregnancy-induced hypertension (preeclampsia and eclampsia) (FAII p487) (FAI2 p54l)


Pregnancy complications (FAII p488) (FAI2 p541, 542)
Amniotic fluid abnormalities (FAII p488) (FAI2 p542)
OB safe medications
Labor
- Dinoprostone (FAII p499) (GGp952)
- Ritodrinelterbutaline (FAII p499) (FAI2 pSSS) (GG p293)
Pregnancy termination
- Mifepristone (RU-486) (FAII p498) (FAI2 p555) (GG pi 184)

II. OB Safe Medications


• Category A: nystatin (vaginal),
• Category B: acetaminophen, diphenhydramine, ondansetron, meclizine, PCN/ampicillin/cephalosporins/

piperacillin, macrolides (erythromycin, azithromycin), nitrofurantoin, metronidazole, H2-blockers/PPIs,


calcium antacids, insulin, metformin, methyldopa,
• Category C: pseudoephedrine, promethazine, demerol/morphine/hydrocodone, nystatin (oral),

hydralazine, nifedipine, labetalol, docusate sodium, heparin

Category Description
A Safety established in human studies
E1 Presumed safety based on animal studies
C No human or animal studies show an adverse effect/uncertain safety
C) Human risk, but benefit may outweigh risk
X Contraindicated, risk clearly outweighs benefit

What is the difference between placental previa, abruptio, and accreta?


(FAII P488)(FAI2p54l-542)

A pregnant woman with previous c-section is at increased risk for what?

14. What are some of the conditions that can result in polyhydramnios?
FAII p488) (FAQp542)

What is the mechanism of action of mifepristone? (FAI I p498) (FAQ p555)

What agents are commonly used as tocolytics? (FAI I p499) (FAQ p334,537)

Copyright Q 2012. Doctors In Trainmg.com. LLC. All Rights Reserved.


[20] Copyright © 2012. Doctors In Trainmg.com. LLC. All Rights Reserved.
Trisomies and Mental Retardation
R: Chapter 5
H: Chapter 62

Fragile X syndrome (FAII p87) (FAI2 p92) (R pi69)


Autosomal trisomies (FAII p88) (FAI2 p93)
- Down syndrome (FAII p88) (FAI2 p93)
- Robertsonian translocation (FAII p89) (FAI2 p93)
- Edwards syndrome (FAII p88) (FAI2 p93)
- Patau syndrome (FAII p88) (FAI2 93)
Mental retardation
Overview of causes of mental retardation

us may nave uown synaromer


(FAI I p88) (FAQ p93)

What is the most common event that causes Down syndrome: meiotic nondisjunctio
mosaicism, or Robertsonian translocation? (FAI I p88) (FAQ p93)

What gene is affected in Fragile X syndrome? (FAI I p87) (FAQ p92)

What are the clinical features of Fragile X syndrome? (FAI I p87) (FAQ p92)

Other Chromosomal Disorders


H: Chapters 61, 62
R: Chapter 5
Sex chromosome disorders (FAII p485) (FAI2 p485) (R pl64-l67)
Cri-du-chat syndrome (FAII p485) (FAI2 p94)
Williams syndrome (FAII p89) (FAI2 p94)
22qll deletion syndromes (FAII p89) (FAI2 p94) (R pi62)

What are the clinical features of Williams syndrome? (FAI I p89) (FAQ p94)

>■, ,iat are the possible defects in cases of chromosome 22ql I deletion?
(FAII p89) (FAQp94)

What are the distinguishing characteristics of Klinefelter syndrome compared to Turner


syndrome? (FA p483) (FAQ p485)

Copyright © 2012. Doctors In Training.com. LLC. All Rights Reserved.


(FAI I p498) (FAQ p555) (GG pi 179)

2. What nerve is damaged when a patient presents with the following upper extremity
symptom®? (FAI I p374) (FAQ p409) (COA p729)
• Loss of forearm pronation
• Loss of arm and forearm flexion
• Trouble initiating arm abduction
• Unable to raise arm above horizontal

What pathology fits the following description?


• Smudge cell
• Port-wine stain in ophthalmic division of trigeminal nerve
• S3 heart sound
• Adrenal hemorrhage associated with meningococcemia
• Ferruginous bodies
• Subepithelial humps on EM
• Spike and dome on EM
• Myocyte disarray
• Currant jelly stool
• Sacroiliitis
\dverse

Benign Breast Conditions


R: Chapter 23
Breast feeding (H p588, 756) (Phys pi023)
Common breast conditions (FAII p494) (FAI2 p550)
- Acute mastitis (FAII p494) (FAI2 p550) (R pi069)
- Fibrocystic change (FAII p494) (FAI2 p550)
- Fat necrosis (FAII p494) (FAI2 pSSO)
- Gynecomastia (FAII p494) (FAI2 p550) (H p30l9) (R pl093)
Benign breast tumors (FAII p492) (FAI2 p548) (R pl07l)
- Fibroadenoma (FAII p492) (FAI2 p548) (R pl09l)
- Intraductal papilloma (FAII p492) (FAI2 p548) (R pl072)
- Phyllodes tumor (FAII p492) (FAI2 p548) (R pl092)
- Adenoma (R pi071)

4. Benign Epithelial Lesions (FAI I p492,494) (FAQ p548)


Non-proliferative breast (fibrocystic) changes
• Fibrosis -
• Cysts-_
• Adenosis: fibroadenoma -

Proliferative breast disease without atypia


• Sclerosis adenosis -
• Epithelial hyperplasia -
• Complex sclerosing lesion -

Copyright © 2012. Doctors In Training.com. LLC. All Rights Reserved.


uick Quiz
What are some of the causes of gynecomastia? SHlSSsiR?n2 p550)

What organism is most commonly responsible for acute mastitis?


(FAI I p494) (FAQ p550)

What is the most common cause of breast lumps in women of childbearing age?
(FAI I p494) (FAQ p550)

What is the most common breast tumor in women under age 25?
(FAII p492) (FAQp548)

What is the classic presenting complaint in a patient with intraductal papilloma?


(FAII p492) (FAQp548)

Breast Tumors and Cancer


R: Chapter 23

Malignant breast tumors (FAII p493) (FAI2 p549)


SERAIS (tamoxifen and raloxifene) (FAII p498) (FAI2 p555) (GG pi 177)
Anastrozolelexemestane (FAII p498) (FAI2 p555) (GG pl76l)

10. In Situ Breast Carcinoma (15-30%)


Ductal carcinoma in situ (DCIS, Intraductal Carcinoma) (FAI I p493) (FAQ p549)
• Comedocarcinoma -
• Solid
■ Cribiform

Papilary
• Micropapillary
Lobular carcinoma in situ (LCIS) -

Invasive Breast Cancer (FAI I p493) (FAQ p549)


• Inflammatory carcinoma (70-80%) -
• Invasive ductal carcinoma -
• Invasive lobular carcinoma -
• Medullary carcinoma -
• Tubular/cribiform carcinoma
• Mucinous carcinoma
• Papilary carcinoma
• Metaplastic carcinoma

Copyright© 2012. Doctors In Training.com. LLC. All Rights Reserved.


What breast pathology fits the following description? (FAI I p492-494) (FAQ p548-550)
• Most common breast tumor in women under 25
• Most common breast mass in postmenopausal women
• Most common breast mass in premenopausal women
• Most common form of breast cancer

• Small, mobile, firm mass with sharp edges in 24-yr-old woman


• Histological "leaf-like projections"
• Signet ring cells
• Loss of e-cadherin cell adhesion gene on chrom 16
• Always ER(+) and PR(+)

• Commonly presents with nipple discharge


• Eczematous patches on nipple
• Multiple bilateral fluid-filled lesions with diffuse breast pain
• Firm, fibrous mass in a 55-yr-old woman

A 58-year-old postmenopausal woman is on tamoxifen. What is she at increased risk of


acquiring?

Copyright ir; 2012. Doctors In Training.com, LLC. All Rights Reserved.


• Signet-ring cells
• Nutmeg liver
• Maternal elevations of AFP
• RBC casts in urine
• Currant-jelly sputum
• Dog or cat bite

Which glomerular disease would you suspect most in a patient w'rth the following findin
(FAII p468)(FAI2p5l7-5l8)
• IF: granular pattern of immune complex deposition; LM: diffuse capillary thickening
• IF: granular pattern of immune complex deposition; LM: hypercellular glomeruli
• IF: linear pattern of immune complex deposition
• IF: deposition of IgG, IgM, IgA, and C3 in the mesangium
• EM: subendothelial humps and tram-track appearance
• Nephritis, deafness, cataracts
• LM: crescent formation in the glomeruli
• LM: segmental sclerosis and hyalinosis
• Purpura on back of arms and legs, abdominal pain, IgA nephropathy
• EM: spiking of the GBM due to electron dense subepithelial deposits

For which organisms can the following antibiotic classes be used?


(FAI I pi86, FAI I pi88) (FAQ p206, FAQ p209)
uaitrjaOTHH-si

Micro by Systems I
H: Chapters 126, 257, 381
R: Chapters 15, 26. 28
Normal flora: dominant (FAII pl75) (FAI2 pl95)
Common causes of pneumonia (FAII pl76) (FAI2 pl96)
Common causes of meningitis (FAII pl77) (FAI2 pl97)
CSF findings of meningitis (FAII pl77) (FAI2 pl97)
Osteomyelitis (FAII pl77) (FAI2 pl97)

Copyright'':-) 2012. Doctors lnTr.iining.com, LLC. All Rights Reserved.


4. Which infectious agent fits the following description?
Common cause of pneumonia in immunocompromised patients
Most common cause of atypical / walking pneumonia
Most common fungal infection of the lung in the Texas/Gulf Coastal region
Common causative agent for pneumonia in alcoholics
Can cause an interstitial pneumonia in bird handlers

Often the cause of pneumonia in a patient with a history of exposure to bats and bat droppings
Fungal cause of pneumonia in a patient who has recently visited South California, New Mexico, or
West Texas
Pneumonia associated with "current jelly" sputum
Q fever

Associated with pneumonia acquired from air conditioners


Most common cause of pneumonia in children I-year-old or younger
Most common cause of pneumonia in the neonate (birth-28d)
Most common cause of pneumonia in children and young adults (including college students, military
recruits, and prison inmates)
Common cause of pneumonia in patients with other health problems
Most common cause of viral pneumonia

Causes a wool-sorter's disease (a life-threatening pneumonia)


Endogenous flora in 20% of adults
Common bacterial cause of COPD exacerbation
Common pneumonia in ventilator patients and those with cystic fibrosis
Pontiac fever

£
<L>

H
CO

>
LU

[2] Copyright •© 2012. Doctors In Training.com. LLC. All Rights Reserved.


5. An adolescent presents with cough and rust-colored sputum. What does the gram
of the sputum show?

'■>. An older male patient has blood in his urine and renal stones. What organism is most
likely responsible for this patient's stones?

What organism is the most common cause of osteomyelitis? (FAI I p!77) (FAQ pl97)

What organism would you suspect as the cause of a septic arthritis in a 20-year-old m
(FAII pl77) (FAQ pi97)

(FAII pl76) (FAQpl96)


• 6w-l8y
• I8y-40y
I What are the most common causes of pneumonia for the following patient populatior
• 40y-65y
• Elderly

10. What findings would help you distinguish bacterial meningitis from fungal meningitis an
viral meningitis? (FAI I pl77) (FAQ pl97)

What CSF findings would you see in a patient with TB meningitis?


(FAII p 177) (FAQp 197)

Micro by Systems 2
H: Chapters 130-131
R: Chapter 22

Sexually transmitted diseases (FAII pl8l) (FAI2 p20l)


Vaginal dischargelvaginitis
Pelvic inflammatory disease (FAII pl8l) (FAI2 p202)
Fever in the post-op patient
Nosocomial infections (FAII pl8l) (FAI2 p202)
TORCHES infections (FAII pl79) (FAI2 pl99)

Copyright© 2012. Doctors In Training.com. LLC. All Rights Reserved.


12. What are the features of congenital syphilis?
Earlv manifestations Cfirst 5 weeks of lifel
• Hepatosplenomegaly, elevated LFTs
• Hemolytic anemia, jaundice
• Rash followed by desquamation of hands and feet
• Snuffles (blood-tinged nasal secretions)
• Radiographic changes at birth: metaphyseal dystrophy and periostitis

• Hutchinson teeth (notching or blunting of the upper incisors)


• Saddle nose deformity
• Frontal bossing
• Saber shins (anterior bowing of the tibia)

13. Vaginal Discharge/Vaginitis

pH Historical Clues Wet Prep


Physiologic discharge
Candidiasis

Trichomonas

Bacterial vaginosis

14. Fever in the Post-Op Period


• Wind
• Water
• Wound
• Walking
• Wein
• Wonder drugs

What are the TORCH infections? (FAI I pl78) (FAQ pl99)

50-year-old patient recovered from abdominal surgery performed 2 days ago and has
had an internal catheter in place since that time. He now has a fever of 100° F. What is the
most likely cause of his fever?

Which STD matches the following statement? (FAI I pl8l) (FAQ p20l)
• Clue cells
• Painless genital ulcer
• Flagellated cells

Copyright S 2012. Doctors In Training.com, LLC. All Rights Reserved.


What are 5 examples of encapsulated bacteria? What test can be used to detect
encapsulated bacteria? (FAII pl39) (FAQ pl49)

What causes a steeple sign on X-ray? What causes a thumb sign on X-ray?

What pathology matches the following statement? (FAI I p52l-53l) (FAQ p580-588)
• Signet ring cells in the ovary
• Signet rings in RBCs
• Smudge cell
• Spike and dome of glomerulus on EM
• Tram track of glomerulus on light microscopy
• Strawberry tongue
Most common location of tophi

Drug Side Effects


GG: Chapter 4

Drug reactions (FAII p244-245) (FAI2 p27l-272)

4. What drugs have the following potential side effects? (FAI I p244) (FAQ p27l)

Anticholinergic SE (dryness, confusion)


Coronary vasospasm
Cutaneous flushing
Dilated cardiomyopathy
Torsades de pointes

Agranulocytosis
Aplastic anemia
Thrombocytopenia
Gray baby syndrome
Hemolysis in G6PD-deficient patients
Thrombosis

Pulmonary fibrosis
Cough
Focal to massive hepatic necrosis

Hepatitis
Pseudomembranous colitis
Adrenocortical insufficiency

Gynecomastia
Hot flashes

Hypothyroidism

Copyright © 2012. Doctors In Traming.com. LLC. All Rights Reserved.


5. What drugs have the following potential side effects? (FAI 1 p245) (FAQ p272)
Gingival hyperplasia
Gout

Osteoporosis
Photosensitivity
Stevens-Johnson syndrome
Drug-induced SLE
Tendon rupture/cartilage damage
Fanconi syndrome
Interstitial nephritis

Hemorrhagic cystitis
Dizziness, nausea, headache, vision changes, tinnitus

Nephrogenic diabetes insipidus


Seizures
Tardive dyskinesia
Disulfiram-like reaction

Nephro + neurotoxicity
Nephro + ototoxicity
■ 1

What medication causes cardiotoxicity and bone marrow suppression?

A patient tries to commit suicide by overdosing on digitalis. What is the most important
step in the management of this patient?

What drugs induce the P450 system, and what effect this will have on other drugs?

What drugs inhibit the P450 system, and what effect this will have on other drugs?

. An African-American male that goes to Africa develops anemia after taking prophylactic
medicine for primary disease prevention. What enzyme is this patient deficient in?

A 65-year-old male patient taking multiple medications presents with gynecomastia.


Which of the following meds is most responsible for this patient's gynecomastia? digitalis,
cimetidine, or spironolactone

. A patient presents with tinnitus, dizziness, headaches, and Gl distress. What drug is
causing these symptoms?

drug-indu

Copyright © 2012, Doctors In Traming.com. LLC. All Rights Reserved.


Antidotes
GG: Chapter 4

Specific antidotes (FAII p243) (FAI2 p270)

End of Session Quiz


14. What are the antidotes to the following toxins?
• Acetaminophen
• Salicylates
• Amphetamines
• Anticholinesterases, organophosphates
• Antimuscarinic, anticholinergic agents
• B-blockers (or verapamil)
• Digitalis
• Iron (FAN p224) (FAI2 p270)
• Lead
• Arsenic, mercury, gold
• Copper; arsenic, gold
• Cyanide
• Methemoglobin
• Carbon monoxide
• Methanol, ethylene glycol (antifreeze)
• Opioids
• Benzodiazepines
• Tricyclic Antidepressants
• Heparin
• Warfarin
• t-PA, streptokinase
• Theophylline

Copyright © 2012. Doctors In Training.com. LLC. All Rights Reserved.


What pathology matches the following statement? (FAI I p524-52_,
• Focal myocardial inflammation with multinucleate giant cells
• Eosinophilic cytoplasmic globules in liver near nucleus
• Desquamated epithelial casts in sputum
• "Onion-skin" periosteal reaction
• Pseudopalisading tumor cell arrangement
• Elevated serum uric acid

What are the most common causes of meningitis in the following age ranges?
(FAII pl77) (FAQ pl97)

3. What structures form Hesselbach's triangle? (FAI I p316) (FAQ p345)

A Autosomal Dominant Diseases Part I


^ R : Chapter 5
q Autosomal dominant diseases (FAII p86) (FAI2 p90)
en

What genetic defect is responsible for achondroplasia? (FAQ p90)

A genetic defect in which proteins results in hereditary spherocytosis? (FAQ p90)

6. What are the Cs of Huntington disease? (FAQ p90)

Copyright © 2012, Doctors In Training.com. LLC. All Rights Reserved.


Autosomal Dominant Diseases Part 2 and Trinucleotide Repeats
R: Chapter 5
Autosomal dominant diseases (FAII p86) (FAI2 p90)
Neurofibromatosis (FAII p427) (FAI2 p467)
Von Hippel-Lindau disease (FAII p427) (FAI2 p467)
Trinucleotide repeat diseases (FAII p88) (FAI2 p92)

What autosomal dominant disease fits the following statement?


• Associated with floppy mitral valve, dissecting aortic aneurysm, berry aneurysm
• Associated with mitral valve prolapse, liver disease, berry aneurysms
• Neural tumors and pigmented iris hamartomas
• Very strong assodation with colon cancer
• Ml before age 20
• Hemangioblastomas of retina/cerebellum/medulla
• Increased MCHC, hemolytic anemia
• Bilateral acoustic neuromas
• Fadal lesions, seizure disorder, cancer risk
• Caudate atrophy, dementia
• Cystic medial necrosis of the aorta
• Defect of fibroblast growth factor (FGF) receptor 3

What are some of the disorders resulting from trinucleotide repeats? (FAQ p92)

Copyright (£) 2012. Doctors In Traimng.com. LLC. All Rights Reserved.


What high-yield path fits the following statement. p520-52l) (FAQ p582-583)
• Large "bull's eye" rash
• Strawberry tongue
• Resting tremor, rigidity
• Rash on palms and soles

What are the toxic side effects of tricyclic antidepressant use? (FAI I p455) (FAQ p500)

3. In which glomerular disease would you expect to see the following changes?
(FAI I p467-468) (FAQ p516-517)
• Anti-GBM antibodies (immunofluorescence)
• Kimmelstiel-Wilson lesions (light microscope)
• "Spike and dome" appearance (electron microscope)
• 'Tram track" appearance of subendothelial humps (electron microscope)
umps (electron microscope)

Autosomal-Recessive Diseases and Lysosomal Storage Diseases


-j-, R : Chapter 5

ty Autosomal-recessive diseases (FAII p86) (FAI2 p9l)


Lysosomal storage diseases (FAII pill) (FAI2 pi 16-117)

ss 4. Cystic Fibrosis

Blood Lumen

HC03

carbonic!
anhydrasel
HCO3

Copyright (£9 2012. Doctors In Tratnmg.com, LLC. All Rights Reserved.


Lysosomal Storage Diseases (FAI I pi 11) (FAQ pi 16-117)
5. Which lysosomal storage disease is associated with renal failure?

6. What are the only two X-linked recessive lysosomal storage diseases? What is the
method of inheritance of the others?

7. What is the most common lysosomal storage disease?

8. Which lysosomal storage diseases are associated with an early death (usually by age 3)?

9. Which lysosomal storage disease is a demyelinating disease?

10. How might corneal clouding and mental retardation help distinguish between the
mucopolysaccharidoses?
• Hurler syndrome:
• Hunter syndrome:
• Scheie syndrome:
• l-cell disease: _

11. Which lysosomal storage disease is characterized by the following enzyme deficiency? <
• a-L-iduronidase m
• Iduronate sulfatase ^
• Arylsuffatase A ^
• a-galactosidase A "^
• G a l a c t o c e r e b r o s i d a s e (■* g a l a c t o c e r e b r o s i d e a c c u m u l a t i o n ) I
• p - g l u c o c e r e b r o s i d a s e ( ■▶ g l u c o c e r e b r o s i d e a c c u m u l a t i o n ) Q )
• Hexosaminidase fl>
• Sphingomyelinase (■* sphingomyelin accumulation) 3

12. Characterized by an accumulation of GM2 ganglioside. n


D
to
13. Characterized by an accumulation ofdermatin sulfate. pj
0)
to

14. Which are particularly common among Ashkenazi Jews? }>

Q_
15. Which has characteristic "crinkled paper cytoplasm"? \s
i

5*
16. What is the differential diagnosis for a cherry-red spot on the retina? Q_

Copyright © 2012. Doctors In Trainmg.com. LLC. All Rights Reserved. [II]


. What is the name of the ge e the fits the following description?
• Macroorchidism and autism
• Endocardial cushion defects are common
• Recurrent pulmonary infections, steatorrhea
• Multiple fractures, easily confused with child abuse
• Associated with Alzheimer disease after age 35
• Excess fibro-fatty tissue deposits amongst muscle

'8. What gene is defective in cystic fibrosis? (FAQ p9l)

What is the traditional test used to diagnose cystic fibrosis? (FAQ p9l)

X-Linked Recessive Disorders


TD & Chapter 5
QJ
V X-linked recessive disorders (FAII p86) (FAI2 p9l)
C
~T 20. Hint for X-linked diseases: Fabry's Tale: Duke the Muscular Hunter Brutally Lysed the
X Albino Gopher without being aWAre it was a Fragile Hemophiliac!

Copyright <C 2012. Doctors In Trainmg.com. LLC. All Rights Reserved


Session Quiz
Classify the following disorders as either autosomal dominant autosomal recessive, o,
X-linked recessive.
Glycogen storage diseases, fragile X, polycystic kidney disease (adult and infant), PKU, hereditary
spherocytosis, Duchenne muscular dystrophy, familial adenomatous polyposis, Lesch-Nyhan,
Bruton, Huntington, thalassemias, sickle cell, Wiskott-Aldrich, Von Recklinghausen, Von-Hippel
Lindau, hemophilia, mucopolysaccharidosis, familial hypercholesterolemia, Marfan, cystic fibrosis,
hemochromatosis, G6PD'-deficiency

Autosomal Dominant Autosomal Recessive X-Linked Recessive

Copyright <9 2012. Doctors In Training.com. LLC. All Rights Reserved.


What is the antidote for the following toxin? WXmmmfSTtsm
• Copper, gold, arsenic
• Arsenic, mercury, gold
• t-PA, streptokinase
• Digitalis

What drugs have the following side effects? (FAI I p244-245) (FAQ p27l-272)
• Agranulocytosis
• Osteoporosis
• Pulmonary fibrosis
• Gynecomastia
• Photosensitivity
• Drug-induced lupus

What pathology fits the following high-yield phrase? (FAI I p518-520) (FAQ p580-582)
• Hypertension + hypokalemia + metabolic alkalosis
• Fever + night sweats + weight loss
• Adrenal hemorrhage due to meningococcemia
• Blue sclera
• Cs of Huntington disease
• Hyperphagia, hypersexuality, hyperoralfty, hyperdocilfty
• Nystagmus, intention tremor; scanning speech
• Lower extremity purpura, arthralgias, renal disease

Pediatric Review Part I


Apgar score (FAII p60) (FAI2 p62)
Cerebral palsy
Low birth weight (FAII p60) (FAI2 p62)
Review of neonatal infections and antibiotics
SIDS
Early developmental milestones (FAII p60) (FAI2 p62)
Tanner stages of sexual development (FAII p60) (FAI2 p535)
Puberty

4. What developmental structure matches the following description?


• Supplies oxygenated blood to the fetus
• Removes nitrogenous waste from the fetal bladder
• Fetal placental structure that secretes hCG
• Maternal component of the placenta
• Returns deoxygenated blood from the fetal internal iliac arteries

5. Sudden Infant Death Syndrome (SIDS)


• Usually occurs at 2-4 months old
• Usually occurs while infant is sleeping
• Maternal risk factors: low SES, age <20, drugs/cigarettes during pregnancy
• Infant risk factors: low birth weight, female, premature, prior sibling with SIDS
• Preventive measures: "back to bed," pacifier when sleeping, fan in the room

Copyright s> 2012. Doctors In Training.com. LLC. All Rights Reserved.


Development & Physiology (FAI I p60) (FAQ p62)
6. Complete the following chart of developmental milestones
Gross motor Verbal Fine motor Self-care
3m Rolls over Hands together
6m
12m 1 -3 words Drinks from cup
15m Walk backward, run
18m 4-cube tower
2y Jump up Half understandable Washes/dries hands
3y
4y
5y Copies square

7. A mother presents with her I-year-old child that can stand alone, has just learned
to walk, and has a 5-word vocabulary. She would like to know if her child is
developmentally normal and when she can begin toilet training.

8. A 2-year-old child speaks in short sentences of 2-3 words but cannot identify colors or
recite his ABCs. The mother is concerned that he is not developing normally. What
do you tell the mother?

9. A girl can speak in complete sentences, has an imaginary friend, and considers boys to
be "yucky". How old is she?

10. Tanner Stages


Boy genitalia development Girl breast development Pubic Hair
Stage 1 Prepubertal Prepubertal Prepubertal 73
Stage 2 Enlargement of scrotum and
testes
Bud with elevation of breast
and papilla Areola enlarges
Sparse long,
slightly pigmented
5
hair
Stage 3 Enlargement of penis (length Further enlargement Darker, coarser,
first) and more curled Ln
Stage 4 Penis - growth in breadth and Areola and papilla form a Adult hair in type
development of glans secondary mound above the but covering a
level of the breast smaller area a?
Testes - enlarge
Scrotum - larger and darker %
Stage 5 Adult Mature - only papilla projects Adult in type and n
as areola recesses quantity

Female development Breast development (I I) ■» Growth spurt (12) ■▶ Menarche (13)


2
Male development Stage 2 (12) ■* Growth spurt (14-15)

Copyright © 2012. Doctors In Training.com. LLC. All Rights Reserved. [15]


What factors are taken i

What is the definition of low birth weight? What complications are associated with low
birth weights?

What are the features of 21-hydroxylase deficiency? (FAI I p29l) (FAQ p3l8)

Other than fragile bones, what are the features of osteogenesis imperfect? (FAI I p80)
(FAI2p83)

What genetic defect results in Fragile X syndrome? (FAI I p87) (FAQ p92)

What might you see in a first trimester ultrasound of a fetus with Down syndrome?
(FAII p88) (FAQ p93)

lat are the features of Williams syndrome? (FAI I p89) (FAQ p94)

. Based on the following milestones, how old are the following children?
• Jumps up, 6 cube tower, eats with spoon, 2-3 word sentences
• Regards face, responds to sound, not yet able to roll over
• Stands with support, 1-3 words, stranger anxiety, drinks from a cup
• 'Rides tricycle, understandable sentences, plays board-games

. A 16-year-old female patient presents with amenorrhea. It is later discovered that this
patient lacks a uterus and uterine tubes, and there are two round structures in the
midline just superior to the labia majora. What is most likely the cause of this patient's
amenorrhea?

. A child presents with cleft-lip. Which embryonic process failed?

ow does the presentation of a branchial cleft cyst (FAI I pi 27) (FAQ pi 36) differ fro
that of athyroglossal duct cyst? (FAI I pl30) (FAQ pl38)

Copyright © 2012. Doctors In Training.com, LLC. All Rights Reserved.


Pediatric Review Part 2
Diagnosing disorders of sex hormones (FAII p486) (FAI 2 p539)
Kallmann syndrome (FAII p486) (FAI2 p540)
Review of ethics - consent for minors
Review of pediatric infections already discussed
- Otitis media
Review of pediatric psychiatric disorders
Review of pediatric neoplasms

22. Neoplasms in Children


• ALL
• Astrocytoma
• Neuroblastoma
• Hemangioma
• Wilms tumor
• Hepatoblastoma
• Retinoblastoma
• Rhabdomyosarcoma
• Ewing sarcoma
• Osteogenic sarcoma
• Lymphoma (lymphoblastic)
• Teratoma

73
<
m

Ln

r?

n"

i
cD

Copyright© 2012. Doctors In Training.com. LLC. All Rights Reserved.


[17]
111 r« ■•i M-i^a w i K9i' i F

23. Congenital most commons:


• Most common cause of early cyanosis
• Most common cause of late cyanosis
• Most common cause of primary amenorrhea
• Most common chromosomal disorder
• Most common cause of congenital mental retardation
• Second most common cause of congenital mental retardation
• Most common lethal genetic disease of Caucasians
• Most common cause of congenital malformations in US

. What are some of the main distinguishing features in autistic disorder, Rett disorder, and
Asperger syndrome? (FAI I p442) (FAQ p485)

. vVhat pathology is associated with the following statement? (FAI I p53l) (FAQ p596)
• Most common testicular tumor
Most common tumor of infancy
Most common primary cardiac tumor in children

26. Complete the following chart of developmental milestones (FAI I p60) (FAQ p62)

Gross motor Verbal Fine motor Self-care


Rolls over Hands together

Jump up Half Washes/dries


understandable hands

Copyright ■:■;• 2012. Doctors In Training.com. LLC. All Rights Reserved.


The Wisdom and Art of Medicine

70
m
<
m

ON
I
31
D

fl>

Copyright ©2012. DoctorslnTraining.com. LLC. All Rights Reserved.


rL "i9i
' J
"Realize how much time there is, [and] how long the day is. You have sixteen waking
hours, three or four of which at least should be devoted to making a silent conquest of
your mental machinery.... A few hours out of the sixteen will suffice, only let them be
hours of daily dedication.... The failure to cultivate the power of peaceful
concentration is the greatest single cause of mental breakdown."
Sir William Osier

Listen to the Exhortation of the Dawn!


Look to this Day!
For it is Life, the very Life of Life.
In its brief Course lie all the
Varieties and Realities of your Existence:
The Bliss of Growth,
The Glory of Action,
The Splendour of Beauty;
For Yesterday is but a Dream
And Tomorrow is only a Vision;
But Today well lived makes
Every Yesterday a Dream of Happiness,
And every Tomorrow a Vision of Hope.
Look well therefore to this Day!
Such is the Salutation of the Dawn!

[20] Copyright © 2012. Doctors In Training.com. LLC. All Rights Reserved.


Founder and Chief Educator Brian Jenkins, MD and our other MD physician educators
teach the Doctors In Training Step I Review Course. All of the Doctors In Training
physician lecturers pull insight from their own clinical experience and pair this with their
extensive knowledge of the USMLE and COMLEX. This vital combination of clinical
experience and Basic Science knowledge allows our physicians to teach students the
material needed not only to perform well on board exams, but also to be well-rounded
physicians and make the world a better place.

Michael Mclnnis, MD
The Doctors In Training 2012 Step I Review Course, taught by Doctors in Training Chief Educator,
Brian Jenkins, MD, and additional MD educators focuses on high-yield information in an active-learning
format. This course format keeps medical students engaged and maximizes retention through the use
of quick quizzes, entertaining learning components and mnemonics. The 2012 course is broken down
into approximately 30-45 minute video lecture segments, which allow students to easily digest the
high-yield testing material.

With success around the world, over 14,000 medical students have utilized the Doctors In Training
Step I Review Course from over 150 U.S. and 110 International medical schools.

■/W-z-'im. IKis is ike kiakesttj-iefl source outtkere.


- Case Western reserve- University >cwf of HWtoirxe- SWent

SUCCESSFUL STEP 1
SYKAPTOGENESIS!
D\\ isa.area,tresource-1+ Ke|psr«e*9uou.4^use<sTa*v?S"bw(li-ii^«flf^Wv**4-
A veiu e-Tn«e*\t Wait to prepare-. ft
- pj TTbpkirvs iWersitj- VtMeftt

pur vnarafa is oiwm'ml mJ( wveJl r*e iMfcensefu- ikrotuik tke "toua k tir*es of stuX-ir^ -for ike ewvy.
- TVlLsfwes'tem An-z-ojvx University Swof of Osteopath 7he<$dr\e Vtu^ent

I feft[ikeI WJla.sfyJLyarbwwitk DILI Akofu-teL\weJIittwJtai»-so4W1 "took +J\ea>Urse-!


- ^rtkeastDkio %JUi University SWent

job awp JUI a. supew ioW II kere is w? Way. I Wou|| kive scored as kiak witkout tke- DlTa>Urse!
- Uftiversit* of rUvaA>d\cofoi^e4iw^ >We*\t

I kstaotrrvu-llSl^tv^u^lcy. resuftsWtkaivfl s«r^e-)i1nwmw4-l>stwanWt» W wtl


wif pe-recowv\en??itt4 Dll toeveruoKeamwif^etwteLWSi«r\ih^Up -forike.ytep"^ course.TWkuou.a^airv
- wJMefpkia Cojtae of L\teopatki^7ne^idy\e > Went

I (bvM tke structure of accw^w^ave- (ea.rni.vj. I ke <aui-z--7.es, ^laarajns an| supp(er*eniaru. rwateriaf rvvUe

evenjtWa so simple <*W eastf if uwerstW. lr* DlTaXisors oreaWa stuX p|an -for r*e <WWere tkere -for
i*e even*, step of ine wau.. ikank uou tc DM I
- letkwtM Israef ms+itcte SWent

Visit DoctorslnTraining.com to see our additional medical education lecture series.

DOCTORS IN TRAINING
BETTER DOCTORS. BETTER WORLD.

You might also like